A08 Logarithms

Download as pdf or txt
Download as pdf or txt
You are on page 1of 89

LOGARITHMS

26 JANUARY 2024
REVISION: 836

AZIZ MANVA
AZIZMANVA@GMAIL.COM

ALL RIGHTS RESERVED


Get all the files at: https://bit.ly/azizhandouts
Aziz Manva (azizmanva@gmail.com)

TABLE OF CONTENTS
1.5 Power Rule 23
TABLE OF CONTENTS ................................. 2 1.6 Change of Base 40
0.1 Formula Summary 3 1.7 Change of Base: Reciprocals 46

1. ALGEBRA OF LOGARITHMS .................. 4 2. APPLICATIONS ...................................... 55


1.1 Evaluating Logarithms 4 2.1 Graphs 55
1.2 Exponential and Logarithmic Form 7 2.2 Counting and Probability 62
1.3 Domain 15 2.3 Applications-II 76
1.4 Product and Quotient Rules 18 2.4 Hyperbolic Functions 82
Get all the files at: https://bit.ly/azizhandouts
Aziz Manva (azizmanva@gmail.com)

0.1 Formula Summary


A. Cheat Sheet

Definition
log
⏟ 𝑦𝑥 =𝑧 ⇔ ⏟𝑧 = 𝑥
𝑦 , 𝑥 > 0, 𝑦 > 0, 𝑦 ≠ 1
𝑳𝒐𝒈𝒂𝒓𝒕𝒉𝒎𝒊𝒄 𝑭𝒐𝒓𝒎 𝑬𝒙𝒑𝒐𝒏𝒆𝒏𝒕𝒊𝒂𝒍 𝑭𝒐𝒓𝒎
𝐶𝑜𝑚𝑚𝑜𝑛 𝐿𝑜𝑔𝑠: log 𝑥 𝑥 = 1, log 𝑥 1 = 0

Basic Rules
𝑃𝑟𝑜𝑑𝑢𝑐𝑡 𝑅𝑢𝑙𝑒: log 𝑥 𝑚𝑛 = log 𝑥 𝑚 + log 𝑥 𝑛
𝑚
𝑄𝑢𝑜𝑡𝑖𝑒𝑛𝑡 𝑅𝑢𝑙𝑒: log 𝑥 = log 𝑥 𝑚 − log 𝑥 𝑛
𝑛
𝑃𝑜𝑤𝑒𝑟 𝑅𝑢𝑙𝑒: log 𝑥 𝑎𝑛 = 𝑛 log 𝑥 𝑎

Advanced Rules
𝑚 𝑚
𝑃𝑜𝑤𝑒𝑟 𝑅𝑢𝑙𝑒 𝐸𝑥𝑡𝑒𝑛𝑠𝑖𝑜𝑛: log 𝑎𝑛 𝑥 𝑚 = log 𝑎 𝑥 𝑛 = log 𝑎 𝑥
𝑛
log 𝑚
𝐶ℎ𝑎𝑛𝑔𝑒 𝑜𝑓 𝐵𝑎𝑠𝑒: log 𝑥 𝑚 =
log 𝑥
𝐸𝑥𝑝𝑜𝑛𝑒𝑛𝑡𝑖𝑎𝑡𝑖𝑜𝑛: 𝑎log𝑎 𝑥 =𝑥

Common logs to the Base 10

log 2 log 3 log 5 log 7


0.3010 ≈ 0.3 0.4771 0.7 0.84

P a g e 3 | 89
Get all the files at: https://bit.ly/azizhandouts
Aziz Manva (azizmanva@gmail.com)

1. ALGEBRA OF LOGARITHMS
1.1 Evaluating Logarithms
A. Log 𝑩𝒂𝒔𝒆 𝟏𝟎
When working with exponents, we learn properties like:
𝑎𝑚 × 𝑎𝑛 = 𝑎𝑚+𝑛
Exponents are very common in some situations, and writing them out in superscripts is not very comfortable.
In these kinds of situation, it is useful to have a mathematical technique which will you work with the exponents
directly.

This is the base of the chapter logarithms.

1.1: Definition
⏟𝑥 = 𝑛
𝑎 ⇔⏟
log 𝑎 𝑛 = 𝑥 , 𝑎 > 0, 𝑎 ≠ 1, 𝑛 > 0
𝑬𝒙𝒑𝒐𝒏𝒆𝒏𝒕𝒊𝒂𝒍 𝑳𝒐𝒈𝒂𝒓𝒊𝒕𝒉𝒎𝒊𝒄
𝑭𝒐𝒓𝒎 𝑭𝒐𝒓𝒎

We look at some examples:


10𝟐 = 100 ⇒ log10 100 = 𝟐
2𝟑 = 8 ⇒ log 2 8 = 𝟑

Example 1.2: Base 10


Evaluate:

Integers Decimals 1000


H. log10 0.01
A. log10 1,000 E. log10 0.001 0.01𝑎
B. log10 10,000 1 I. log10 0.001𝑏
F. log10 1,000,000
C. log10 10𝑎 × 100 100
D. log10 100𝑏 × 10 G. log10 0.001

Integers 1 1
3 log = log 6 = log 10−6 = −6
log 1,000 = log 10 = 3 1,000,000 10
2
log 10,000 = log 104 = 4 100 10
log = log −3 = log102−(−3) = log 105 = 5
Decimals 0.001 10
log 0.001 = log 10−3 = −3 0.01𝑎 (10−2 )𝑎
log10 = log10 = −2𝑎 + 3𝑏
0.001b (10−3 )b

Example 1.3: Base 10 with Radicals

Radicals in the Number Radicals in the Base


A. log10 √10 D. log √10 10
0.01 E. log 3√10 √10
B. log10 √1000
F. log 𝑎√10 √10
𝑛 0.1𝑎
C. log10 √100𝑏

Radicals in the Number

P a g e 4 | 89
Get all the files at: https://bit.ly/azizhandouts
Aziz Manva (azizmanva@gmail.com)

1 1
log10 √10 = log10 102 =
2
0.01 5 5
log10 √ = log10 √10−5 = log10 10−2 = −
1000 2

𝑛 0.1𝑎 𝑛 (10−1 )𝑎 𝑛 −𝑎−2𝑏 𝑎 + 2𝑏


log10 √ 𝑏
= log 10 √ 2 𝑏
= log 10 √ 10 −𝑎−2𝑏 = log 10 𝑛
10 =−
100 (10 ) 𝑛
Radicals in the Base
log √10 10 = log 1 10 = 2
102
log 3√10 √10 = log 1 √10 = 3
103
log 𝑎√10 √10 = log 1 √10 = 𝑎
10𝑎

Example 1.4: Integer Bases


Evaluate:

Integers Multi-Step
A. log 2 16 (125)(625)
J. log 5 (AHSME 1950/25)
25
B. log 3 81
K. If 𝑥 = (log 8 2)log2 8 , then log 3 𝑥 equals:
C. log 7 49
(AHSME 1966/9)
D. log 5 125
Decimals
E. log 8 512
L. log 2 0.125
F. log 9 81
M. log 2 0.0625 (AHSME 1959/11)
G. log 3 729
Radicals
H. log 2 2048 4 3
I. log 5 15625 N. log 3 27 √9 √9(AHSME 1953/22)

Integers 1
log 3 𝑥 = log 3 = log 3 3−3 = −3
log 2 16 = 4 27
log 3 81 = 4 Decimals
log 7 49 = 2 First convert the expression from decimals to
log 5 125 = 3 exponents:
log 8 512 = 3 Part L
log 9 81 = 2 125 1
0.125 = =
log 3 729 = 6 1000 8
log 2 2048 = 11 1
log 2 0.125 = log 2 = log 2 2−3 = −3
log 5 15625 = 6 8
Part M
Multi-Step
625 54 1
Part J 0.0625 = = 4 = = 2−4
(125)(625) 10000 2 × 54 24
log 5 = log 5 (5)(625) log 2 0.0625 = log 2 2−4 = −4
25
= log 5 5 ∙ 54 = log 5 55 = 5 Radicals
2 2 1 2 1 1
Part K log 3 33 34 33 = log 3 33+2+3 = log 3 346 = 4
6
1 3 1
𝑥 = (log 8 2)log2 8 = ( ) =
3 27

Example 1.5: Fractional and Radical Bases

P a g e 5 | 89
Get all the files at: https://bit.ly/azizhandouts
Aziz Manva (azizmanva@gmail.com)

➢ If we have fractions in either the number, or the base, we need to “move” the exponent from the
denominator to the numerator. This results in negative numbers.
➢ If, both the number and its base are fractions, then we do not need to move, and the answer remains
positive.
➢ If a number is not a perfect power, then we may still attempt to get a fractional answer. In this case, reduce
all numbers to powers of prime numbers.

Evaluate all parts below. E. log 1 625 L. log121 1331


5
Fractional Bases M. log 64 2048
1 Fractional Answers
A. log 3 3 Radical Bases
F. log 8 4
N. log √2 2
B. log 1 4 G. log 9 27
2
H. log16 8 O. log 3 √3
C. log 1 81 3
3 I. log 81 27 P. log 7 √7
1
D. log 1 J. log 512 128 Q. log √11 112
9 729
K. log 25 125 R. log 2√3 144

3
log 81 27 =
Fractional Bases 4
1 7
= log 3 3−1 = −1
log 3 log 512 128 =
3 9
1 𝑥 3
log 1 4 = 𝑥 ⇒ ( ) = 4 ⇒ 2−𝑥 = 22 ⇒ 𝑥 = −2 log 25 125 =
⏟ 2 ⏟2 2
𝐸𝑥𝑝𝑜𝑛𝑒𝑛𝑡𝑖𝑎𝑙
3
𝐿𝑜𝑔𝑎𝑟𝑖𝑡ℎ𝑚𝑖𝑐 log121 1331 =
𝐹𝑜𝑟𝑚 𝐹𝑜𝑟𝑚 2
log 1 81 = −4 11
3 log 64 2048 =
1 6
log 1 =3 Radical Bases
9 729 Part N
log 1 625 = −4 𝑥 𝑥 𝑥
5 log √2 2 ⇒ (√2) = 2 ⇒ 22 = 21 ⇒ =1⇒𝑥=2
2
Parts O-Q
Fractional Answers
1
Part F log 3 √3 =
2
Observe that 1
3
23 = 8, 22 = 4 log 7 √7 =
3
Hence, if we find the cube root of 8, and then square log √11 112 = 4
it, we will get 4. Part R
2 4
log 8 4 = log 2√3 144 = log 2√3 16 × 9 = log 2√3 24 × (√3)
3
2 4
log 8 4 = 𝑥 ⇒ 8𝑥 = 4 ⇒ 23𝑥 = 22 ⇒ 3𝑥 = 2 ⇒ 𝑥 = = log 2√3 (2√3) = 4
3 If you can directly observe the answer, it will save you
Parts G-M
3 many steps.
log 9 27 = 2 4
2 (2√3) = 12 ⇒ (2√3) = 122 = 144 ⇒ log 2√3 144
3 =4
log16 8 =
4

1.6: Important Values

log 2 log 3 log 5 log 7

P a g e 6 | 89
Get all the files at: https://bit.ly/azizhandouts
Aziz Manva (azizmanva@gmail.com)

0.3010 ≈ 0.3 0.4771 0.7 0.84

B. Natural Logarithms

1.7: Natural Logarithm (Log Base 𝒆)


The number 𝑒 has great importance in mathematics.
𝑒 ≈ 2.71

The log of a number 𝑥 to the base 𝑒 is written:


log 𝑒 𝑥 = ln 𝑥

𝑒 is a constant like 𝜋

Example 1.8: Irrational Bases


Evaluate:

1 3
Basics C. ln 𝑒 2 Radicals F. ln √𝑒
A. ln 𝑒 2 D. log 𝜋 𝜋 2 E. ln √𝑒 𝜋
G. ln √𝑒 𝑒
B. ln 𝑒 5

Basics 1 1
2 log 𝑒 √𝑒 = log 𝑒 𝑒 2 =
log 𝑒 𝑒 = 2 2
log 𝑒 𝑒 5 = 5 3 1
log 𝑒 √𝑒 =
1 1 3
log 𝑒 𝑒 2 = 𝜋 𝑒
𝑒
2 ln √𝑒 =
log 𝜋 𝜋 2 = 2 𝜋
Radicals

1.2 Exponential and Logarithmic Form


A. Basics

1.9: Exponential and Logarithmic Form


⏟𝑥 = 𝑛
𝑎 ⇔⏟
log 𝑎 𝑛 = 𝑥 , 𝑎 > 0, 𝑎 ≠ 1, 𝑛 > 0
𝑬𝒙𝒑𝒐𝒏𝒆𝒏𝒕𝒊𝒂𝒍 𝑳𝒐𝒈𝒂𝒓𝒊𝒕𝒉𝒎𝒊𝒄
𝑭𝒐𝒓𝒎 𝑭𝒐𝒓𝒎

Every exponential expression has a logarithmic version, and vice versa. With practice, it is possible to move from
one to the another. This kind of fluency is important in being able to simplify complicated equations.
43 = 64 log 4 64 = 3
5 ⇔

2 = 32 log
⏟ 2 32 = 5
𝑬𝒙𝒑𝒐𝒏𝒆𝒏𝒕𝒊𝒂𝒍 𝑭𝒐𝒓𝒎 𝑳𝒐𝒈𝒂𝒓𝒊𝒕𝒉𝒎𝒊𝒄 𝑭𝒐𝒓𝒎

Example 1.10:
Convert as indicated below.

Convert to logarithmic form A. 42 = 16 B. 211 = 2048

P a g e 7 | 89
Get all the files at: https://bit.ly/azizhandouts
Aziz Manva (azizmanva@gmail.com)

C. 6𝑦 = 216 G. 𝑒 2𝑥 = 21 K. log 2 64 = 𝑥
D. 𝑦 4 = 81 Convert to exponential form L. log 𝑎 𝑏 = 𝑐
E. 𝑎𝑏 = 𝑐 H. log 2 64 = 6 M. ln(2𝑥) = 3.1
F. 𝑥 = 𝑒3 I. log 𝑥 64 = 6
J. log 2 𝑥 = 6

Convert to logarithmic form Convert to exponential form


log 4 16 = 2 26 = 64
log 2 2048 = 11 𝑥 6 = 64
log 6 216 = 𝑦 26 = 𝑥
log 𝑦 81 = 4 2𝑥 = 64
log 𝑎 𝑐 = 𝑏 𝑎𝑐 = 𝑏
ln 𝑥 = 3
2𝑥 = ln 21 2𝑥 = 𝑒 3.1

Example 1.11: Simple Equations


Find the value of 𝑥 in each case below:

Integer Values G. log 𝑥 27 = 2 3


K. log 64 𝑥 = 2
A. log 𝑥 128 = 7 H. log 𝑥 1024 = 3 1
L. log 𝑥 6 = log 5
B. log 𝑥 81 = 2 Fractional Values √5
C. log 𝑥 729 = 6 I. log 𝑥 4 =
1 Natural Logarithm Related
2
D. log 𝑥 729 = 3 3 M. log 𝑥 𝑒 3 = 4
E. log 𝑥 729 = 2 J. log 81 𝑥 =4
F. log 𝑥 12 = 2

Integer Values Fractional Values


Part A 1
4 = 𝑥 2 ⇒ 𝑥 = 16
log 𝑥 128 = 7 ⇒ 𝑥 7 = 128 ⇒ 𝑥 7 = 27 ⇒ 𝑥 = 2 3
Part B 𝑥 = 814 = 33 = 27
3
log 𝑥 81 = 2 ⇒ 𝑥 2 = 81 ⇒ 𝑥 = ±√81 = ±9 𝑥 = 642 = 83 = 512
However, since 𝑥 is the base of the logarithm, 1 1 1 −2
𝑥 > 0 ⇒ 𝑅𝑒𝑗𝑒𝑐𝑡 𝑥 = −9 ⇒ 𝑥 = 9 log 𝑥 6 = − ⇒ 𝑥 −2 = 6 ⇒ (𝑥 −2 ) = 6−2
2
Parts C,D,E 1
log 𝑥 729 = 6 ⇒ 𝑥 = 3 𝑥 = 6−2 =
36
log 𝑥 729 = 3 ⇒ 𝑥 = 9 Natural Logarithm Related
log 𝑥 729 = 2 ⇒ 𝑥 = 27 Part M
Parts F,G,H 3
log 𝑥 𝑒 3 = 4 ⇒ 𝑥 4 = 𝑒 3 ⇒ 𝑥 = ±𝑒 4
log 𝑥 12 = 2 ⇒ 𝑥 2 = 12 ⇒ 𝑥 = √12 = 2√3
log 𝑥 27 = 2 ⇒ 𝑥 2 = 27 ⇒ 𝑥 = √27 = 3√3 But
log 𝑥 1024 = 3 ⇒ 𝑥 3 = 1024 = 210 3
1 10 1 1 𝑥 > 0 ⇒ 𝑥 = 𝑒4
23 3
3
𝑥= (210 )3 = 23 = = 23 × 23 = 8√2

Example 1.12: Inequalities


A. log 2 𝑥 > 3
B. log 𝑥 2 > 3

P a g e 8 | 89
Get all the files at: https://bit.ly/azizhandouts
Aziz Manva (azizmanva@gmail.com)

C. log 2 (𝑥 + 2) > 7

Part A
log
⏟ 2𝑥 > 3 ⇒ 𝑥⏟> 23 ⇒ 𝑥 > 8 ⇒ 𝑥 ∈ (8, ∞)
𝑳𝒐𝒈𝒂𝒓𝒊𝒕𝒉𝒎𝒊𝒄 𝑬𝒙𝒑𝒐𝒏𝒆𝒏𝒕 𝑭𝒐𝒓𝒎
𝑭𝒐𝒓𝒎
Part B
3 3
log 𝑥 2 > 3 ⇒
⏟ 2 > 𝑥3
⏟ ⇒ 𝑥 3 < 2 ⇒ 𝑥 < √2 ⇒ 𝑥 ∈ (0,1) ∪ (1, √2)
𝑳𝒐𝒈𝒂𝒓𝒊𝒕𝒉𝒎𝒊𝒄 𝑬𝒙𝒑𝒐𝒏𝒆𝒏𝒕 𝑭𝒐𝒓𝒎
𝑭𝒐𝒓𝒎
Part C
𝑥 + 2 > 27 ⇒ 𝑥 > 126

Example 1.13
Find the value of 𝑥 in the form 𝑝𝑎 𝑞𝑏 , where 𝑝 and 𝑞 are prime numbers:
4
√3
log √𝑥 72 = log 3√3 5
√3

Work with the RHS first:


1
34 1 1 1
log 3√3 1 = log 3√3 34−5 = log 1 320 = 𝑦
⏟ 33
35 𝐿𝑜𝑔𝑎𝑟𝑖𝑡ℎ𝑚𝑖𝑐 𝐹𝑜𝑟𝑚
1 𝑦 1 𝑦 1 𝑦 1 3
(33 ) = 320 ⇒ 33 = 320 ⇒ = ⇒𝑦=
⏟ 3 20 20
𝐸𝑥𝑝𝑜𝑛𝑒𝑛𝑡𝑖𝑎𝑙 𝐹𝑜𝑟𝑚
3 40
3 1 20 3 3 3 40 3 40 40
log √𝑥 72 = ⇒ (𝑥 2 ) = 72 ⇒ 𝑥 40 = 72 ⇒ (𝑥 40 ) = 72 3 ⇒ 𝑥 40× 3 = 72 3
⏟ 20 ⏟
𝑳𝒐𝒈𝒂𝒓𝒊𝒕𝒉𝒎𝒊𝒄 𝑬𝒙𝒑𝒐𝒏𝒆𝒏𝒕𝒊𝒂𝒍
𝑭𝒐𝒓𝒎 𝑭𝒐𝒓𝒎
40 80
𝑥 = (23 × 32 ) 3 = 240 × 3 3

Example 1.14
If log 2𝑥 216 = 𝑥, where 𝑥 is real, then 𝑥 is:
A. A non-square, non-cube integer
B. A non-square, non-cube, non-integral rational number
C. An irrational number
D. A perfect square
E. A perfect cube (AHSME 1960/24)

Convert from logarithmic to exponential form:


(2𝑥)𝑥 = 216
2 𝑥 𝑥 𝑥 = 63 = 23 × 33
𝑥=3

Example 1.15: Change of Subject


Make 𝑥 the subject in the following equations:
A. 𝑦 = log 5 (𝑥 + 4) + 3
B. 𝑦 = 4 ln(√𝑥 + 3)
C. 𝑦 = √ln(𝑥 2 + 2) − 2

P a g e 9 | 89
Get all the files at: https://bit.ly/azizhandouts
Aziz Manva (azizmanva@gmail.com)

Part A 𝑦 2
(𝑒 4 − 3) = 𝑥
Subtract 3 from both sides:
𝑦 − 3 = log 5 (𝑥 + 4) Part C
Convert to exponential form:
5𝑦−3 = 𝑥 + 4 𝑦 2 = ln(𝑥 2 + 2) − 2
Subtract 4 from both sides: 𝑦 2 + 2 = ln(𝑥 2 + 2)
2
5𝑦−3 − 4 = 𝑥 𝑒 𝑦 +2 = 𝑥 2 + 2
2
Part B 𝑒 𝑦 +2 − 2 = 𝑥 2
𝑦 2
= ln(√𝑥 + 3) ±√𝑒 𝑦 +2 − 2 = 𝑥
4
𝑦
𝑒 4 = √𝑥 + 3

Example 1.16
A. Find the domain of 𝑦 = √ln(𝑥 2 + 2) − 2

The expression for which we are finding the logarithm must be positive:
𝑥 2 + 2 > 0 ⇒ 𝐴𝑙𝑤𝑎𝑦𝑠 ℎ𝑜𝑙𝑑𝑠
The expression inside the square root must be non-negative:
ln(𝑥 2 + 2) − 2 ≥ 0
ln(𝑥 2 + 2) ≥ 2
Convert to exponential form:
𝑥2 + 2 ≥ 𝑒2
Subtract 2 from both sides:
𝑥2 ≥ 𝑒2 − 2
Use the property that 𝑥 2 ≥ 𝑎 ⇒ 𝑥 > √𝑎 𝑂𝑅 𝑥 < −√𝑎:
𝑥 ≥ √𝑒 2 − 2 𝑂𝑅 𝑥 ≤ −√𝑒 2 − 2

1.17: Absolute Value Equations


|𝑥| = 𝑎, 𝑎 ∈ ℝ
𝑎 > 0 ⇒ 𝑥 = ±𝑎
𝑎=0⇒𝑥=0
𝑎<0⇒𝑥∈𝜙

|𝑥| = 5 ⇒ 𝑥 = ±5
|𝑥| = 0 ⇒ 𝑥 = 0
|𝑥| = −5 ⇒ 𝑥 ∈ 𝜙

Example 1.18: Absolute Value


Solve for 𝑥 in each part.

Absolute Value in the Number


A. log 5 |𝑥 + 2| = 2
Absolute Value in the Answer
B. log 3 𝑥 = 𝑦, |𝑦| = 2
Absolute Value in the Base
C. log |𝑥| 4 = 3
Absolute Value Terms

P a g e 10 | 89
Get all the files at: https://bit.ly/azizhandouts
Aziz Manva (azizmanva@gmail.com)

D. |log 𝑥 4| = 2

Absolute Value in the Number


log 5|𝑥 + 2| = 2
Convert to exponential form
|𝑥 + 2| = 52 = 25
𝑥 + 2 = ±25
𝑥 ∈ {−27,23}
Absolute Value in the Answer
log 3 𝑥 = 𝑦, |𝑦| = 2
1
𝑥 = 3𝑦 = 3±2 ⇒ 𝑥 ∈ { , 9}
9
Absolute Value in the Base
log |𝑥| 4 = 3
|𝑥|3 = 4
Take cube roots:
3
|𝑥| = √4
3
𝑥 = ± √4
Absolute Value Terms
log 𝑥 4 = ±2
Convert to exponential form:
𝑥 ±2 = 4
𝑥 2 = 4 ⇒ 𝑥 = {+2, −𝟐} ⇒ 𝑥 = 2
1 1 1
2
= 4 ⇒ 𝑥2 = ⇒ 𝑥 = ±
𝑥 4 2

Since is the base, 𝑥 > 0:


1
𝑥=
2

Example 1.19: Absolute Value


𝑎
A. The product of the solutions to log 4|𝑥 + 1| = −7 can be written in the form 𝑏 for integers 𝑎, 𝑏 with no
common factor other than 1. Find 𝑏 − 𝑎.
B. Find the sum of the solutions to |log 2 𝑥| + |log 3 𝑥| + ⋯ + |log 2022 𝑥| = −2022

Part A
log 4 |𝑥| = −7
1 1
|𝑥 + 1| = 4−7 = 7 = 14
4 2
1
𝑥 + 1 = ± 14
2
1
𝑥 = ± 14 − 1
2
1 1 − 214
𝑥 = 14 − 1 =
2 214
1 1 + 214
𝑥 = − 14 − 1 = −
2 214

P a g e 11 | 89
Get all the files at: https://bit.ly/azizhandouts
Aziz Manva (azizmanva@gmail.com)

1 + 214 1 − 214
𝑥 ∈ {− , }
214 214
(1 + 214 ) (1 − 214 )
𝑃𝑟𝑜𝑑𝑢𝑐𝑡 = − ×
214 214
28 28
1−2 2 −1
= =
228 228
𝑏 − 𝑎 = 228 − (228 − 1) = 1
Part B
𝐿𝐻𝑆 ≥ 0
𝑅𝐻𝑆 < 0
𝐶𝑜𝑛𝑡𝑟𝑎𝑑𝑖𝑐𝑡𝑖𝑜𝑛
𝑥 ∈ {𝜙}

Sum of empty set is zero (by convention)


𝐴𝑛𝑠𝑤𝑒𝑟 = 0

Example 1.20: Absolute Value


If |𝑥 − log 𝑦| = 𝑥 + log 𝑦 where 𝑥 and log 𝑦 are real, then
A. 𝑥 = 0
B. 𝑦 = 1
C. 𝑥 = 0 and 𝑦 = 1
D. 𝑥(𝑦 − 1) = 0
E. None of these (AHSME 1972/8)

𝑥 − log 𝑦 = ±(𝑥 + log 𝑦)


Case I:
𝑥 − log 𝑦 = 𝑥 + log 𝑦
− log 𝑦 = log 𝑦
2 log 𝑦 = 0
log 𝑦 = 0
𝑦=1
Case II:
𝑥 − log 𝑦 = −𝑥 − log 𝑦
𝑥 = −𝑥
2𝑥 = 0
𝑥=0
We combine the two cases to get:
𝑦 = 1 𝑂𝑅 𝑥 = 0
𝑦 − 1 = 0 𝑂𝑅 𝑥 = 0
And multiply the two equations to get:
𝑥(𝑦 − 1) = 0

Example 1.21
A. (𝐴𝑛𝑠𝑤𝑒𝑟 𝑖𝑛 𝑠𝑖𝑚𝑝𝑙𝑒𝑠𝑡 𝑟𝑎𝑑𝑖𝑐𝑎𝑙 𝑓𝑜𝑟𝑚) If log 6 𝑥 = 2.5, the value of 𝑥 is: (AHSME 1953/5)
B. (𝑄𝑢𝑎𝑑𝑟𝑎𝑡𝑖𝑐𝑠) The values of 𝑎 in the equation: log10(𝑎2 − 15𝑎) = 2 are: (AHSME 1951/22)
C. If log10 𝑥 2 − 3𝑥 + 6 = 1, the value(s) of 𝑥 is/are: (AHSME 1953/21)
Part A
Convert to exponential form:
𝑥 = 62.5 = 62 × 60.5 = 36√6

P a g e 12 | 89
Get all the files at: https://bit.ly/azizhandouts
Aziz Manva (azizmanva@gmail.com)

Part B
Convert to exponential form:
102 = 𝑎2 − 15𝑎
(𝑎 − 20)(𝑎 + 5) = 0
𝑎 ∈ {−5,20}
Part C
Convert to exponential form:
𝑥 2 − 3𝑥 + 6 = 10
𝑥 2 − 3𝑥 − 4 = 0
(𝑥 − 4)(𝑥 + 1) = 0
𝑥 ∈ {−1,4}

Example 1.22: Equations Reducible to Quadratics


A. Find the value(s) of 𝑥 that satisfy (log 𝑥)2 − 11 log 𝑥 + 10 = 0
B. (𝐶ℎ𝑎𝑙𝑙𝑒𝑛𝑔𝑒) Find the last three digits of the product of the positive roots of √1995𝑥 log1995 𝑥 = 𝑥 2 . (AIME
1995/2)

Part A
Change of variable.
Substitute 𝑦 = log 𝑥:
𝑦 2 − 11𝑦 + 10 = 0 ⇒ 𝑦 = {1,10}
Change back to the original variable
log 𝑥 = {1,10} ⇒ 𝑥 = {10,1010 }
Part B
Change of variable.
Let 𝑦
⏟= log1995 𝑥 ⇒ ⏟1995𝑦 = 𝑥 . Then
𝐿𝑜𝑔𝑎𝑟𝑖𝑡ℎ𝑚𝑖𝑐 𝐸𝑥𝑝𝑜𝑛𝑒𝑛𝑡𝑖𝑎𝑙
𝐹𝑜𝑟𝑚 𝐹𝑜𝑟𝑚
√1995(1995𝑦 )𝑦 = (1995𝑦 )2
Combine using properties of exponents:
1 2
19952+𝑦 = 19952𝑦
Bases are same, the exponents must be same:
1 1
+ 𝑦 2 = 2𝑦 ⇒ 𝑦 2 − 2𝑦 + = 0
2 2
1
Apply the quadratic formula: 𝑎 = 1, 𝑏 = −2, 𝑐 = 2:
1
2 ± √4 − (4)(1) (2) 2 ± √2
𝑦= =
2 2
Change back to the original variable
2 ± √2 2±√2
log1995 𝑥 = ⇒ 𝑥 = {1995 2 }
⏟ 2 ⏟
𝐿𝑜𝑔𝑎𝑟𝑡ℎ𝑚𝑖𝑐 𝐹𝑜𝑟𝑚 𝐸𝑥𝑝𝑜𝑛𝑒𝑛𝑡𝑖𝑎𝑙 𝐹𝑜𝑟𝑚
Both solutions are positive. Product is:
2+√2 2−√2 4
1995 2 × 1995 2 = 19952 = 19952
Last three digits are the remainder when 19952 is divided by 1000:
19952 ≡ 9952 ≡ (−5)2 ≡ 25 (𝑚𝑜𝑑 1000)
Alternately, you can get the last three digits as:
19952 = (2000 − 5)2

P a g e 13 | 89
Get all the files at: https://bit.ly/azizhandouts
Aziz Manva (azizmanva@gmail.com)

= 20002 − (2)(−5)(2000) + 25
The first two terms do not matter:
𝐹𝑖𝑛𝑎𝑙 𝐴𝑛𝑠𝑤𝑒𝑟 = 25

1.23: Nested Logs


Nested logs are best solved inside out. They look intimidating but can be solved with practice.

Example 1.24: Nested Logs


For each part below, find the value (if it is an expression), or solve for the variable (if it is an equation).

Double Nesting
A. log 5 [log10 𝑥] = 0
B. log 3 [log 4 𝑥] = 0
C. [log10(5 log10 100)]2 (AHSME 1964/1)
Triple Nesting
D. log 3 {log 2[log10 10512 ]}
E. log 4 [log 2 (log 3 𝑥)] = 0
F. log 1 [log 1 (log 27 𝑥)] = 0
2 3

G. log 1 [log 1 (log 1 𝑥)] = 0


3 4 5
Quadruple Nesting
H. log 𝜋2 +𝑒 {log 1 [log 4√2(log 7 2401)] + 4}
2

Double Nesting
log10 𝑥 = 1 ⇒ 𝑥 = 10
log 4 𝑥 = 1 ⇒ 𝑥 = 4
[log10(5 × 2)]2 = [log10 (10)]2 = [1]2 = 1
Triple Nesting
log 3 {log 2[512]} = log 3 9 = 2
log 2 (log 3 𝑥) = 1 ⇒ log 3 𝑥 = 2 ⇒ 𝑥 = 9
1 1
log 1 (log 27 𝑥) = 1 ⇒ log 27 𝑥 = ⇒ 𝑥 = 273 = 3
3 3
1
1 1 4 1
log 1 (log 1 𝑥) = 1 ⇒ log 1 𝑥 = ⇒ 𝑥 = ( ) = 5−4
4 5 5 4 5
Quadruple Nesting
log 𝜋2 +𝑒 {log 1 [log 4√2 (4)] + 4} = log 𝜋2 +𝑒 {log1 [8] + 4} = log 𝜋2 +𝑒 {1} = 0
2 2

Example 1.25
A. If log 2(log 2 (log 2 𝑥)) = 2, then how many digits are in the base-ten representation for x? (AHSME 1993/11)
1
B. If log 7(log 3 (log 2 𝑥)) = 0, then 𝑥 −2 equals (AHSME 1983/12)
C. If log 2(log 3 (log 4 𝑥)) = log 3 (log 4 (log 2 𝑦)) = log 4 (log 2(log 3 𝑧)) = 0, then the sum 𝑥 + 𝑦 + 𝑧 is equal to
(AHSME 1971/21)

Part A
log 2 (log 2 𝑥) = 22 = 4
log 2 𝑥 = 24 = 16

P a g e 14 | 89
Get all the files at: https://bit.ly/azizhandouts
Aziz Manva (azizmanva@gmail.com)

𝑥 = 216 = 210 ∙ 26 = 1024 ∙ 64 ≈ 64000


5 𝐷𝑖𝑔𝑖𝑡𝑠
Part B
log 3 (log 2 𝑥) = 1
log 2 𝑥 = 3
𝑥=8
1 1 1 1 1
𝑥 −2 = 8− 2 = 1 = =
82 √8 2√2
Part C
log 3(log 4 𝑥) = log 4(log 2 𝑦) = log 2 (log 3 𝑧) = 1
𝑥 = 43 = 64
𝑦 = 24 = 16
𝑧 = 32 = 9
𝑥 + 𝑦 + 𝑧 = 64 + 16 + 9 = 89

Example 1.26
A. log 7 |2 log 5 𝑥| = 0

1 1
|2 log 5 𝑥| = 1 ⇒ 2 log 5 𝑥 = ±1 ⇒ log 5 𝑥 = ± ⇒ 𝑥 ∈ { , √5 }
2 √5

1.3 Domain
A. Uniqueness of Log Function

1.27: Uniqueness of Logarithm


log 𝑎 𝑥 = 𝑛
For a specific base 𝑎, 𝑥 is unique for unique values of 𝑛.

1.28: Using Uniqueness of Logarithm


We can use the uniqueness of logarithm to conclude:
log 𝑎 = log 𝑏 ⇔ 𝑎 = 𝑏

Example 1.29: No Solutions


Find the value(s) of 𝑥 that satisfy log(𝑥 + 1) − log 𝑥 = 0

log(𝑥 + 1) = log 𝑥
𝑥+1 = 𝑥 ⇒1 = 0⇒ 𝑥 ∈𝜙
B. Variables in the Base

1.30: Restrictions on Base


log 𝑎 𝑛 = 𝑥, 𝑎 > 0, 𝑎 ≠ 1, 𝑛 > 0

𝑎 is the base of the logarithm


𝑎 > 0 because for a negative number, the answer alternates between positive and negative as the power to which it
is raised is even or odd.
𝑎 ≠ 1 since 1 to any power is 1 itself.

P a g e 15 | 89
Get all the files at: https://bit.ly/azizhandouts
Aziz Manva (azizmanva@gmail.com)

1.31: Restrictions on Number


log 𝑎 𝑛 = 𝑥, 𝑎 > 0, 𝑎 ≠ 1, 𝑛 > 0
Logarithms of negative numbers are not defined.

Why is 𝑛 not allowed to be negative? As an example, consider:

log10 −5 ⇒ 10𝑥 = −5 ⇒ 𝐻𝑎𝑠 𝑛𝑜 𝒓𝒆𝒂𝒍 𝑠𝑜𝑙𝑢𝑡𝑖𝑜𝑛


We have to check two things:
➢ Base should be positive, and not equal to 1.
➢ Number of which we are taking the base should be positive.

Example 1.32
Are the following logarithms defined:
A. log 3 5
B. log1 1
1
C. log 1
3 2
D. log 4 (−2)
E. log (−2) (4)

Part A 1
𝑁𝑜. =
> 0 ⇒ 𝐴𝑙𝑙𝑜𝑤𝑒𝑑
𝑁𝑜. = 5 > 0 ⇒ 𝐴𝑙𝑙𝑜𝑤𝑒𝑑 2
𝐵𝑎𝑠𝑒 = 3 > 0, 3 ≠ 1 ⇒ 𝐴𝑙𝑙𝑜𝑤𝑒𝑑 1 0,1
𝐵𝑎𝑠𝑒 = > ≠ 1 ⇒ 𝐴𝑙𝑙𝑜𝑤𝑒𝑑
𝐷𝑒𝑓𝑖𝑛𝑒𝑑 3 3
Part B 𝐷𝑒𝑓𝑖𝑛𝑒𝑑
𝑁𝑜. = 1 ⇒ 𝐴𝑙𝑙𝑜𝑤𝑒𝑑 Part D
𝐵𝑎𝑠𝑒 = 1 ⇒ 𝑁𝑜𝑡 𝐴𝑙𝑙𝑜𝑤𝑒𝑑 𝑁𝑢𝑚𝑏𝑒𝑟 = −2 < 0 ⇒ 𝑁𝑜𝑡 𝑉𝑎𝑙𝑖𝑑
𝑁𝑜𝑡 𝐷𝑒𝑓𝑖𝑛𝑒𝑑 Part E
Part C 𝐵𝑎𝑠𝑒 = −2 < 0 ⇒ 𝑁𝑜𝑡 𝑉𝑎𝑙𝑖𝑑

Example 1.33: Finding Domain


Find the domain of the following logarithms:

Basics C. log 𝑥−2(𝑥 + 5)


A. log 5 𝑥 3 2
D. log 2𝑥+√2 (− 𝑥 + )
5 3
B. log 𝑥 5 7

Linear Expressions

Basics 𝐵𝑎𝑠𝑒: 𝑥 − 2 > 0 ⇒ 𝑥


⏟> 2
Part A 𝑉𝑎𝑙𝑖𝑑 𝐷𝑜𝑚𝑎𝑖𝑛
𝑓𝑜𝑟 𝐵𝑎𝑠𝑒
𝑥 > 0 ⇒ 𝑥 ∈ (0, ∞) 𝐵𝑎𝑠𝑒: 𝑥 − 2 = 1 ⇒ 𝑥
⏟= 3
Part B 𝑁𝑜𝑡 𝐴𝑙𝑙𝑜𝑤𝑒𝑑
𝑥 > 0, 𝑥 ≠ 1 ⇒ (0,1) ∪ (1, ∞) 𝑓𝑜𝑟 𝐵𝑎𝑠𝑒

Linear Expressions Combine the three conditions:


Part C 𝑥 > 2, 𝑥 ≠ 3
𝑁𝑢𝑚𝑏𝑒𝑟: 𝑥 + 5 > 0 ⇒ ⏟ 𝑥 > −5 𝑥 ∈ (2,3) ∪ (3, ∞)
𝑉𝑎𝑙𝑖𝑑 𝐷𝑜𝑚𝑎𝑖𝑛 {𝑥|𝑥 > 2, 𝑥 ≠ 3}
𝑓𝑜𝑟 𝑁𝑢𝑚𝑏𝑒𝑟 Part D
The number must be greater than zero:

P a g e 16 | 89
Get all the files at: https://bit.ly/azizhandouts
Aziz Manva (azizmanva@gmail.com)

3 2 3 2 2 5 10 2 2 7
− 𝑥+ >0⇒− 𝑥>− ⇒𝑥< × ⇒𝑥< 𝑥 + √2 = 1 ⇒ 𝑥 = 1 − √2 ⇒ 𝑥 = (1 − √2)
5 3 5 3 3 3 9 7 7 2
Which we can estimate as:
The base must be greater than zero: 7
≈ − (0.41)
2 2 7 2
𝑥 + √2 > 0 ⇒ 𝑥 > −√2 ⇒ 𝑥 > − √2 Combine the three conditions:
7 7 2
The base must not be equal to 1: 7 7 7 10
(− √2, − (1 − √2)) ∪ (− (1 − √2), )
2 2 2 9

Example 1.34: Finding Domain of Quadratic Expressions


Find the domain of log 𝑥 2 +5𝑥+6 (𝑥 2 − 7𝑥 + 12)

The number must be greater than zero:


𝑥 2 − 7𝑥 + 12 > 0 ⇒ (𝑥 − 3)(𝑥 − 4) > 0 ⇒ 𝑥 ∈ ℝ \[3,4]
The base must be greater than zero:
𝑥 2 + 5𝑥 + 6 > 0 ⇒ (𝑥 + 2)(𝑥 + 3) > 0 ⇒ 𝑥 ∈ ℝ \[−3, −2]
The base must not be equal to 1:
−5 ± √52 − (4)(1)(5) −5 ± √5
𝑥 2 + 5𝑥 + 6 = 1 ⇒ 𝑥 2 + 5𝑥 + 5 = 0 ⇒ 𝑥 = =
2(1) 2
−5 ± √5
𝑥 ∈ ℝ\ {[−3, −2] ∪ [3,4] ∪ { }}
2

Example 1.35: Equations


Solve the following:
A. log 𝑥 (3𝑥 2 + 10𝑥) = 3
B. log 𝑥+2(3𝑥 2 + 4𝑥 − 14) = 2

Part A Convert from logarithmic form to exponential form:


Convert from logarithmic form to exponential form: 3𝑥 2 + 4𝑥 − 14 = (𝑥 + 2)2
3𝑥 2 + 10𝑥 = 𝑥 3 2𝑥 2 − 18 = 0
𝑥 − 3𝑥 2 − 10𝑥 = 0
3
𝑥2 = 9
𝑥(𝑥 2 − 3𝑥 − 10) = 0 𝑥 = {−3,3}
𝑥(𝑥 − 5)(𝑥 + 2) = 0 Check −3:
𝑥 = {−𝟐, 𝟎, 5} 𝑥 + 2 = −3 + 2 = −1 ⇒ 𝑅𝑒𝑗𝑒𝑐𝑡 − 3
But the base must be 𝑥 > 0, 𝑥 ≠ 1. Hence, Check 3:
𝑥=5 𝑥 + 2 = 3 + 2 = 5 ⇒ 𝑉𝑎𝑙𝑖𝑑
Substitute 𝑥 = 5 in the number 3𝑥 2 + 10𝑥: 3(3)2 + 4(3) − 14 = 27 + 12 − 14 ⇒ +𝑣𝑒
3(52 ) + 10(5) = +𝑣𝑒 𝑥=3
Part B

Example 1.36: Domain


Number
A. Linear
B. Radical
C. Quadratic
D. Polynomial
E. Exponential

P a g e 17 | 89
Get all the files at: https://bit.ly/azizhandouts
Aziz Manva (azizmanva@gmail.com)

Base
A. Linear
B. Radical
C. Quadratic
D. Polynomial
E. Exponential

C. Review

Example 1.37
Is the solution below correct?
1
log 𝑥 𝑒 2 = 4 ⇒ 𝑥 4 = 𝑒 2 ⇒ 𝑥 = ±(𝑒 2 )4 = ±√𝑒

𝑥 𝑖𝑠 𝑏𝑎𝑠𝑒 ⇒ 𝑥 > 0 ⇒ 𝑥 ≠ −√𝑒 ⇒ 𝑆𝑜𝑙 𝑖𝑠 𝑖𝑛𝑐𝑜𝑟𝑟𝑒𝑐𝑡.

1.4 Product and Quotient Rules


A. Product Rule
Exponentiation lets us convert multiplication into addition:
𝑥 𝑝 × 𝑥 𝑞 = 𝑥⏟
⏟ 𝑝+𝑞
𝑴𝒖𝒍𝒕𝒊𝒑𝒍𝒊𝒄𝒂𝒕𝒊𝒐𝒏 𝑨𝒅𝒅𝒊𝒕𝒊𝒐𝒏
Since logarithms are defined based on exponents, every exponent property has a corresponding logarithm
property. We start by looking the logarithm property corresponding to the exponent property above.

log 10 + log 100 = 1 + 2 = 3 = log 1000

1.38: Product Rule: Log of a product equals the sum of logs


Log of a product equals the sum of the logs:
log 𝑎 𝑥𝑦
⏟ =⏟
log 𝑎 𝑥 + log 𝑎 𝑦
𝑀𝑢𝑙𝑡𝑖𝑝𝑙𝑖𝑐𝑎𝑡𝑖𝑜𝑛 𝐴𝑑𝑑𝑖𝑡𝑖𝑜𝑛
Applies to any valid base

Let:
log 𝑎 𝑥 = 𝑚 ⇒ 𝑎𝑚 = 𝑥, log 𝑎 𝑦 = 𝑛 ⇒ 𝑎𝑛 = 𝑦
Then:
𝑅𝐻𝑆 = log 𝑎 𝑥 + log 𝑎 𝑦 = 𝑚 + 𝑛
𝑚 𝑛
Substitute 𝑎 = 𝑥 and 𝑎 = 𝑦:
𝐿𝐻𝑆 = log 𝑎 𝑥𝑦 = log 𝑎 𝑎𝑚 𝑎𝑛 = log 𝑎 𝑎𝑚+𝑛 = 𝑚 + 𝑛 = 𝑅𝐻𝑆
Shorter Version
Let 𝑥 = 𝑎𝑚 , and 𝑦 = 𝑎𝑛 for some 𝑚 and 𝑛.
log 𝑎 𝑥𝑦 = log 𝑎 𝑎𝑚 𝑎𝑛 = log 𝑎 𝑎𝑚+𝑛 = 𝑚 + 𝑛 = log 𝑎 𝑎𝑚 + log 𝑎 𝑎𝑛 = log 𝑎 𝑥 + log 𝑎 𝑦

Example 1.39: Expressions

Combine into a single term: C. log10 125 + log10 8 Use the product rule to expand:
A. log 20 10 + log 20 2 D. 1 + log 5 4 F. log 2 20
B. log 6 12 + log 6 3 E. 2 + log 2 5 G. log10 125 + log10 4

P a g e 18 | 89
Get all the files at: https://bit.ly/azizhandouts
Aziz Manva (azizmanva@gmail.com)

Combine into a single term: logarithmic form before we can apply the property.
Parts A, B and C 1 + log 5 4 = log 5 5 + log 5 4 = log 5 20
log 20 10 + log 20 2 = log 20 10 × 2 = log 20 20 = 1 2 + log 2 5 = log 2 4 + log 2 5 = log 2 20
log 6 12 + log 6 3 = log 6 36 = 2 Use the product rule to expand:
log10 125 + log10 8 = log10 1000 = 3 log 2 20 = log 2 (5 × 4) = log 2 5 + log 2 4 = log 2 5 + 2
Parts D and E log10 125 + log10 4 = log10 500
Here, some parts of the expression are not in = log10 100 + log10 5 = 2 + log10 5
logarithmic form. So, we need to convert them into

Example 1.40
A. Find the sum of log 1 + log 2 + ⋯ + log 𝑛
B. Find the sum of eight terms of ln 𝑥 + ln 𝑥 2 𝑦 + ln 𝑥 3 𝑦 2 + ⋯
1 2 3 1946
C. Simplify 𝑋𝑎 = log 𝑎 2 + log 𝑎 3 + log 𝑎 4 + ⋯ + log 𝑎 1947 , 𝑎 > 0, 𝑎 ≠ 1

Part A
log 1 + log 2 + ⋯ + log 𝑛 = log(1 × 2 × … × 𝑛) log 𝑛!
Part B
Use the property that ln 𝑎 + ln 𝑏 = ln 𝑎𝑏
𝑥 × 𝑥2𝑦 × 𝑥3𝑦2 × …
ln ⏟
8 𝑇𝑒𝑟𝑚𝑠
ln(𝑥 1+2+⋯+8 × 𝑦 0+1+⋯+7 )
8×9 7×8
ln (𝑥 2 ×𝑦 2 )

ln(𝑥 36 𝑦 28 )
Part C
This is a telescoping series, which we will able to collapse once we combine:
1 2 1946
𝑋𝑎 = log 𝑎 × × … ×
2 3 1947
Telescope:
1
𝑋𝑎 = log 𝑎
1947

Example 1.41: Equations


Solve for 𝑥:
1
(2 log 4 + 2 log 2) = log 𝑥
2

log 4 + log 2 = log 𝑥


log 8 = log 𝑥
8=𝑥

Example 1.42: Equations


A. log(𝑥 + 5) = log 3 + log 5
1
B. If log(𝑥) + log 2 = log 2 + log 1024, then find log 2 𝑥.
𝑥 𝑥
C. log (2) + log (3) = log(𝑥) + log 3
D. If log10 𝑚 = 𝑏 − log10 𝑛, then 𝑚 = (AHSME 1950/26)
E. If log10 𝑎 + log10 𝑏 = 𝑐 − log10 𝑑, then 𝑎 =

P a g e 19 | 89
Get all the files at: https://bit.ly/azizhandouts
Aziz Manva (azizmanva@gmail.com)

F. If all the logarithms are real numbers, the equality log(𝑥 + 3) + log(𝑥 − 1) = log(𝑥 2 − 2𝑥 − 3) is satisfied
for: (AHSME 1968/23)

Part A 𝑚𝑛 = 10𝑏
log(𝑥 + 5) = log 15 ⇒ 𝑥 + 5 = 15 ⇒ 𝑥 = 10 10𝑏
Part B 𝑚=
𝑛
log(2𝑥) = log 512 Part E
2𝑥 = 512 log10 𝑎𝑏𝑑 = 𝑐
𝑥 = 256 Convert from logarithmic form to exponential form:
log 2 256 = 8 𝑎𝑏𝑑 = 10𝑐
10𝑐
𝑎=
Part C 𝑏𝑑
𝑥2 Part F
log ( ) = log(3𝑥) Using the product rule:
6
𝑥2 log(𝑥 + 3)(𝑥 − 1) = log(𝑥 2 − 2𝑥 − 3)
= 3𝑥 log(𝑥 2 + 2𝑥 − 3) = log(𝑥 2 − 2𝑥 − 3)
6
2
𝑥 = 18𝑥 𝑥 2 + 2𝑥 − 3 = 𝑥 2 − 2𝑥 − 3
Since 𝑥 ≠ 0: 4𝑥 = 0
𝑥 = 18 𝑥=0
Part D However:
log10 𝑚 + log10 𝑛 = 𝑏 log(𝑥 − 1) = log(0 − 1) = log −1
log10 𝑚𝑛 = 𝑏 Hence, there are no real solutions.
Convert from logarithmic form to exponential form: 𝑥∈𝜙

Example 1.43: Freshman’s Dream


𝑀𝑎𝑟𝑘 𝑡ℎ𝑒 𝑐𝑜𝑟𝑟𝑒𝑐𝑡 𝑜𝑝𝑡𝑖𝑜𝑛
log 𝑝 + log 𝑞 = log(𝑝 + 𝑞) only if:
A. 𝑝 = 𝑞 = 𝑧𝑒𝑟𝑜
𝑞2
B. 𝑝 = 1−𝑞
C. 𝑝 = 𝑞 = 1
𝑞
D. 𝑝 =
𝑞−1
𝑞
E. 𝑝 = (AHSME 1952/18)
𝑞+1

log 𝑝𝑞 = log(𝑝 + 𝑞)
𝑝𝑞 = 𝑝 + 𝑞
𝑝𝑞 − 𝑝 = 𝑞
𝑝(𝑞 − 1) = 𝑞
𝑞
𝑝=
𝑞−1

Example 1.44: Inequalities


A. log 2 𝑥 + log 2 5 > 3
B.

Part A
log 2 5𝑥 > 3
Convert from logarithmic form to exponential form:

P a g e 20 | 89
Get all the files at: https://bit.ly/azizhandouts
Aziz Manva (azizmanva@gmail.com)

5𝑥 > 23
5𝑥 > 8
8
𝑥>
5
8
𝑥 ∈ ( , ∞)
5

B. Quotient Rule
First, look at the exponent rule for division.
𝑥𝑚
=⏟ 𝑥𝑚 ÷ 𝑥𝑛 = 𝑥
⏟𝑚 × 𝑥 −𝑛 = ⏟𝑚−𝑛
𝑥
𝑥⏟𝑛 𝐷𝑖𝑣𝑖𝑠𝑖𝑜𝑛 𝑀𝑢𝑙𝑡𝑖𝑝𝑙𝑖𝑐𝑎𝑡𝑖𝑜𝑛 𝑆𝑢𝑏𝑡𝑟𝑎𝑐𝑡𝑖𝑜𝑛
𝐷𝑖𝑣𝑖𝑠𝑖𝑜𝑛

1.45: Quotient Rule


Log of a quotient equals the difference of the logs.
𝑥
log = ⏟
log 𝑥 − log 𝑦
⏟𝑦 𝑆𝑢𝑏𝑡𝑟𝑎𝑐𝑡𝑖𝑜𝑛
𝐷𝑖𝑣𝑖𝑠𝑖𝑜𝑛

Let 𝑥 = 𝑎𝑚 , 𝑦 = 𝑎𝑛 :
𝑥 𝑎𝑚
log 𝑎 = log 𝑎 𝑛 = log 𝑎 𝑎𝑚−𝑛 = 𝑚 − 𝑛 = log 𝑎 𝑎𝑚 − log 𝑎 𝑎𝑛 = log 𝑥 − log 𝑦
𝑦 𝑎

True or False 1.46:


log 𝑥
log 𝑥 − log 𝑦 =
log 𝑦

False

Example 1.47: Expressions


Simplify:
5
A. log 2 4
B. log 2 20 − log 2 5
C. log 7 686 − log 7 2
𝑎 𝑏 𝑐 𝑎𝑦
D. log 𝑏 + log 𝑐 + log 𝑑 − log 𝑑𝑥 (AHSME 1957/5)
𝑎 𝑏 𝑦
E. log 𝑧 + log 𝑧 + ⋯ + log 𝑧 − log 𝑧 𝑎
𝑏 𝑐 𝑧

Parts A-C
5
log 2 = log 2 5 − log 2 4 = log 2 (5) − 2
4
20
log 2 20 − log 2 5 = log 2 = log 2 4 = 2
5
686
log 7 686 − log 7 2 = log 7 = log 7 343 = 3
2
Parts D-E (Telescoping)
𝑎 𝑏 𝑐 𝑑𝑥 𝑥
log × × × = log
𝑏 𝑐 𝑑 𝑎𝑦 𝑦

P a g e 21 | 89
Get all the files at: https://bit.ly/azizhandouts
Aziz Manva (azizmanva@gmail.com)

𝑎 𝑏 𝑦 1 1
log 𝑧 × × … × × = log 𝑧 = log 𝑧 𝑧 −1 = −1
𝑏 𝑐 𝑧 𝑎 𝑧

Example 1.48
log 𝑏 10 = 𝑋, log 𝑏 5 = 𝑌, log 𝑏 3 = 𝑍
Write the following expressions in terms of 𝑋, 𝑌 and 𝑍, if possible. Explain, for the expressions which cannnot be
written in terms of 𝑋, 𝑌 and 𝑍 why they cannot be so written.
50
A. log 𝑏 ( 9 )
20
B. log 𝑏 ( )
3
C. log 𝑏 2
D. log 𝑏 17

Part A
50 5 × 10
) = log 𝑏 ( 2 ) = log 𝑏 5 + log 𝑏 10 − log 𝑏 (32 ) = 𝑌 + 𝑋 − 2𝑍
log 𝑏 (
9 3
Where in the last step we made use of:
log 𝑏 3 = 𝑍 ⇒ 𝑏 𝑍 = 3 ⇒ 𝑏 2𝑍 = 32 ⇒ log 𝑏 (32 ) = 2𝑍
Part B
20 100 102
log 𝑏 ( ) = log 𝑏 ( ) = log 𝑏 ( ) = log 𝑏 102 − log 𝑏 3 − log 𝑏 5 = 2𝑋 − 𝑍 − 𝑌
3 15 3×5
Part C
10
log 𝑏 2 = log 𝑏 = log 𝑏 10 − log 𝑏 5 = 𝑋 − 𝑌
5
Part D
Note that
log 𝑏 3 = 𝑍 ⇒ 3 𝑖𝑠 𝑝𝑟𝑖𝑚𝑒
log 𝑏 5 = 𝑌 ⇒ 5 𝑖𝑠 𝑝𝑟𝑖𝑚𝑒
log 𝑏 2 × 5 = 𝑋 ⇒ 2,5 𝑎𝑟𝑒 𝑝𝑟𝑖𝑚𝑒
The prime numbers at our disposal are:
{2,3,5}
17 is a prime number not in the above set. Hence log 𝑏 17 cannot be written in terms of only 𝑋, 𝑌 and 𝑍.

Example 1.49: Equations


A. Solve for 𝑥: log 3 𝑥 − log 3 4 = log 3 12
1
B. Solve for 𝑥:log 3 𝑥 − log 3 10 = 2 + log 3 5
𝑠
C. If 𝑃 = (1 + 𝑘)𝑛 then 𝑛 equals: (AHSME 1958/12)

Part A
𝑥 𝑥
log 3 = log 3 12 ⇒ = 12 ⇒ 𝑥 = 48
4 4
Part B
𝑥 𝑥
log 3 = log 3 45 ⇒ = 45 ⇒ 𝑥 = 4.5
0.1 0.1
Part C
𝑠
(1 + 𝑘)𝑛 =
𝑃
Take logs to the base 1 + 𝑘:
𝑠
log1+𝑘 (1 + 𝑘)𝑛 = log1+𝑘
𝑃
P a g e 22 | 89
Get all the files at: https://bit.ly/azizhandouts
Aziz Manva (azizmanva@gmail.com)

𝑠
𝑛 = log1+𝑘
𝑃
𝑠
(1 + 𝑘)𝑛 =
𝑃
𝑛
𝑠
log(1 + 𝑘) = log
𝑃
𝑠
𝑛 log 1 + 𝑘 = log
𝑃
𝑠
log
𝑛= 𝑃 = log 𝑠 − log 𝑃
log(1 + 𝑘) log(1 + 𝑘)

Example 1.50: Concept Check


Of the following:
1. 𝑎(𝑥 − 𝑦) = 𝑎𝑥 − 𝑎𝑦
2. 𝑎 𝑥−𝑦 = 𝑎 𝑥 − 𝑎 𝑦
3. log(𝑥 − 𝑦) = log 𝑥 − log 𝑦
log 𝑥
4. log 𝑦
= log 𝑥 − log 𝑦
5. 𝑎(𝑥𝑦) = 𝑎𝑥 × 𝑎𝑦

A. Only 1 and 4 are true


B. Only 1 and 5 are true
C. Only 1 and 3 are true
D. Only 1 and 2 are true
E. Only 1 is true (AHSME 1950/18)

2(𝑥 + 𝑦) = 2𝑥 + 2𝑦

𝑎(𝑥𝑦) = 𝑎𝑥 × 𝑎𝑦 = 𝑎2 𝑥𝑦

2(3 × 4) = 2(12) = 24
2(3 × 4) = (2 × 3 ) × (2 × 4) = 6 × 8 = 48
3(𝑥𝑦) = 3𝑥 × 3𝑦 = 9𝑥𝑦 ⇒ 𝑊𝑟𝑜𝑛𝑔
Option E.
Only 1 is true.

1.5 Power Rule


A. Power Rule
The power rule in exponentiation tells us that we can convert exponentiation into multiplication of exponents.
( 𝑥 𝑚 )𝑛
⏟ = 𝑥
⏟ 𝑚𝑛

𝐸𝑥𝑝𝑜𝑛𝑒𝑛𝑡𝑖𝑎𝑡𝑖𝑜𝑛 𝑀𝑢𝑙𝑡𝑖𝑝𝑙𝑖𝑐𝑎𝑡𝑖𝑜𝑛
The same rule is applicable for logarithms. Exponents can be converted into multiplication of logs.

1.51: Power Rule


Log of the 𝑛𝑡ℎ power of a quantity is 𝑛 times the log of the quantity:
⏟ 𝑥𝑛
log = 𝑛⏟log 𝑥
𝑬𝒙𝒑𝒐𝒏𝒆𝒏𝒕𝒊𝒂𝒕𝒊𝒐𝒏 𝑴𝒖𝒍𝒕𝒊𝒑𝒍𝒊𝒄𝒂𝒕𝒊𝒐𝒏

P a g e 23 | 89
Get all the files at: https://bit.ly/azizhandouts
Aziz Manva (azizmanva@gmail.com)

Let
𝑚 = log 𝑏 𝑥

Convert to exponential form:


𝑏𝑚 = 𝑥

Raise both sides to power 𝑛


𝑏 𝑛𝑚 = 𝑥 𝑛

Take log both sides:


𝑛𝑚 = log 𝑥 𝑛

Substitute 𝑚 = log 𝑏 𝑥:
𝑛(log 𝑏 𝑥) = log 𝑥 𝑛

Which is what we wished to prove

Example 1.52

Evaluate F. log 0.25 ÷ log 8


A. log 𝑎 𝑎3 , 𝑎 > 0, 𝑎 ≠ 1 Combine into a single logarithm
B. log 9 273 G. 2 log 3 + 3 log 2
1 1 1
C. log 8 29 H. log 49 + 3 log 125
1 2
D. log 8 ÷ log 8 (AHSME 1961/6) I.
1 1
2 log 3 + 3 log 5 − log 125 − log 9
log 27 2 3
E. log 9

1
log 0.25 log 4 log 2−2 −2 log 2 2
Evaluate = 3
= 3
= =−
log 𝑎 𝑎3 = 3 log 8 log 2 log 2 3 log 2 3
9 Combine into a single logarithm
log 9 273 = log 9 (33 )3 = log 9 39 = 2 log 3 + 3 log 2 = log 9 + log 8 = log 72
2
log 8 29 = log 8(23 )3 = log 8 (8)3 = 3 1 1 1 1 5
log + log 125 = log + log 5 = log
2 49 3 7 7
3 2
1 log 8 log 8 log 32 + log 53 − log 52 − log 33
log 8 ÷ log = = = −1
8 log 8 −1 − log 8 32 × 53 4 3
log 33 3 log 3 3 = log 2 3 = log 3 × 5
3 2
= = 33 × 52
log 32 2 log 3 2

1.53: Approximation
➢ Currently, logarithms are important for their application in Calculus and other areas of math (modelling,
etc).
➢ But, historically, logarithms were used to carry out calculations for which the values of logarithms from a
log table were important.
➢ Even now, exam questions can still check your ability to work with logarithms

Example 1.54
If you are given log 8 ≈ 0.9031 and log 9 ≈ 0.9542, then the only logarithm that cannot be found without the use of
tables is:

P a g e 24 | 89
Get all the files at: https://bit.ly/azizhandouts
Aziz Manva (azizmanva@gmail.com)

A. log 17
5
B. log
4
C. log 15
D. log 600
E. log .4 (AHSME 1951/45)

Convert the values given to have a prime number:


0.9031
log 8 ≈ 0.9031 ⇒ log 23 ≈ 0.9031 ⇒ 3 log 2 ≈ 0.9031 ⇒ log 2 ≈
3
0.9542
log 9 ≈ 0.9542 ⇒ log 3 =
3

Note that you can calculate:


5 10
log = log = log 10 − log 8 = 1 − log 8
4 8
10
log 15 = log 3 + log 5 = log 3 + log = log 3 + log 10 − log 2 = log 3 + 1 − log 2
2
log 600 = log 6 × 100 = log 2 + log 3 + log 100 = log 2 + log 3 + 2
4
log 0.4 = log = log 4 − log 10 = (2 log 2) − 1
10
However
log 17 ⇒ 17 𝑖𝑠 𝑎 𝑝𝑟𝑖𝑚𝑒 𝑡ℎ𝑎𝑡 𝑤𝑒 𝑑𝑜 𝑛𝑜𝑡 𝑘𝑛𝑜𝑤 ℎ𝑜𝑤 𝑡𝑜 ℎ𝑎𝑛𝑑𝑙𝑒

Option A

Example 1.55: (Calculator allowed, but do not use the log keys)
If log 2 = .3010 and log 3 = .4771, the value of 𝑥 when 3𝑥+3 = 135 is approximately (AHSME 1954/38)

27 ∙ 3𝑥 = 135
Divide both sides by 27:
3𝑥 = 5
Take logs both sides:
log 3𝑥 = log 5
Use the power rule:
𝑥 log 3 = log 5
Solve for 𝑥:
10
log 5 log 2 log 10 − log 2 1 − log 2 1 − 0.3010 0.699
𝑥= = = = = = = 1.47
log 3 log 3 log 3 log 3 0.4771 0.4771

Example 1.56
A sum of money, when invested in a bank, doubles in 20 years under compound interest. Determine, to the closest
year, when will it treble.

Hint: Make use of the values of log 2 and log 3

𝑟 𝑛
Substitute 𝐴 = 2𝑃, 𝑛 = 20 in 𝐴 = 𝑃 (1 + ) :
100
𝑟 20 𝑟 20
2𝑃 = 𝑃 (1 + ) ⇒ 2 = (1 + )
100 100

P a g e 25 | 89
Get all the files at: https://bit.ly/azizhandouts
Aziz Manva (azizmanva@gmail.com)
𝑟
Use a change of variable. Let 𝑥 = 1 + 100:
log 2
𝑥 20 = 2 ⇒ 20 log 𝑥 = log 2 ⇒ log 𝑥 =
⏟ 20
𝑬𝒒𝒖𝒂𝒕𝒊𝒐𝒏 𝑰

𝑟 𝑛
Substitute 𝐴 = 3𝑃 in 𝐴 = 𝑃 (1 + ) :
100
𝑟 20
3𝑃 = 𝑃 (1 + ) ⇒ 3 = 𝑥 𝑛 ⇒ 𝑛 log 𝑥 = log 3
100
Solving for 𝑛, and substituting Equation I:
log 3 log 3 log 3 × 20 0.477 × 20
𝑛= = = = ≈ 31.8 ≈ 32 𝑦𝑒𝑎𝑟𝑠
log 𝑥 log 2 log 2 0.3
20

Example 1.57
log 125 equals:
A. 100 log 1.25
B. 5 log 3
C. 3 log 25
D. 3 – 3 log 2
E. (log 25)(log 5) (AHSME 1954/15)

1000
log 125 = log = log 1000 − log 8 = log 1000 − log 23 = 3 − 3 log 2
8
Option C

Example 1.58
1+𝑥 3𝑥+𝑥 3
Let 𝐹 = log 1−𝑥. Find a new function 𝐺 by replacing each 𝑥 in 𝐹 by 1+3𝑥 2
, and simplifying. The simplified expression
𝐺, in terms of 𝐹, is equal to: (AHSME 1963/30 Adapted, AHSME 1972/29 Adapted)

3𝑥 + 𝑥 3 1 + 3𝑥 + 3𝑥 2 + 𝑥 3
1+
𝐺 = log 1 + 3𝑥 2 = log 1 + 3𝑥 2
3𝑥 + 𝑥 3 1 − 3𝑥 + 3𝑥 2 − 𝑥 3
1−
1 + 3𝑥 2 1 + 3𝑥 2
Find the LCM to add fractions: Since the denominator is the same in both the
1 + 3𝑥 2 3𝑥 + 𝑥 3 fractions, we can cancel:
2+ 2
= log 1 + 3𝑥 2 1 + 3𝑥3 1 + 3𝑥 + 3𝑥 2 + 𝑥 3
1 + 3𝑥 3𝑥 + 𝑥 = log
− 1 − 3𝑥 + 3𝑥 2 − 𝑥 3
1 + 3𝑥 2 1 + 3𝑥 2 Note that the numerator is the expansion of (1 + 𝑥)3 ,
Add and rearrange:
and the denominator is the expansion of (1 − 𝑥)3 :
(1 + 𝑥)3 1+𝑥 3 1+𝑥
= log 3
= log ( ) = 3 log = 3𝐹
(1 − 𝑥) 1−𝑥 1−𝑥

B. Equations

Example 1.59: Equations


Find 𝑥 in terms of log 2 and log 5:
8𝑥−2 = 5𝑥

P a g e 26 | 89
Get all the files at: https://bit.ly/azizhandouts
Aziz Manva (azizmanva@gmail.com)

Substitute 8 = 23 :
23𝑥−6 = 5𝑥
Rearrange:
23𝑥
= 5𝑥
26
Take log both sides:
log 23𝑥 − log 26 = log 5𝑥
Use the power rule:
3𝑥 log 2 − 6 log 2 = 𝑥 log 5
Collate all terms on the LHS:
3𝑥 log 2 − 𝑥 log 5 = 6 log 2
Factor 𝑥:
𝑥(3 log 2 − log 5) = 6 log 2
Solve for 𝑥:
6 log 2
𝑥=
3 log 2 − log 5

Example 1.60: Equations


Solve for 𝑥:
2 log 𝑎 3 + log 𝑎 (𝑥 − 4) = log 𝑎 (𝑥 + 8)

Use the power rule:


log 𝑎 32 + log 𝑎 (𝑥 − 4) = log 𝑎 (𝑥 + 8)
Use the product rule:
log 𝑎 (9𝑥 − 36) = log 𝑎 (𝑥 + 8)
Exponentiate both sides:
9𝑥 − 36 = 𝑥 + 8
8𝑥 = 44
44 11
𝑥= = = 5.5
8 2

Example 1.61
Consider the graphs of 𝑦 = 2 log 𝑥 and 𝑦 = log 2𝑥. We may say that:
A. They do not intersect
B. They intersect at 1 point only
C. They intersect at 2 points only
D. They intersect at a finite number of points but greater than 2
E. They coincide (AHSME 1961/19)

The graphs will coincide when the 𝑦 values are equal:


2 log 𝑥 = log 2𝑥
log 𝑥 2 = log 2𝑥
Exponentiate both sides:
𝑥 2 = 2𝑥
Since 𝑥 ≠ 0, divide by 𝑥 both sides:
𝑥=2

We get one solution for the system of equations.


𝑂𝑝𝑡𝑖𝑜𝑛 𝐵

P a g e 27 | 89
Get all the files at: https://bit.ly/azizhandouts
Aziz Manva (azizmanva@gmail.com)

Example 1.62: Equations


A. If log 𝑥 − 5 log 3 = −2, then 𝑥 equals: (AHSME 1955/17)
3
B. Solve for 𝑥: 𝑥 ln 𝑥 = 𝑒 (ln 𝑥)
1 1
C. If 𝑚, 𝑛 ∈ ℝ+ , find the value of 𝑚 in terms of 𝑛: log ( (𝑚 + 2𝑛)) = (log 𝑚 + log 𝑛)
2√2 2
D. Solve for 𝑥 in terms of log 2 given that 22𝑥+1 = 3 × 2 1−2𝑥

Part A
log 𝑥 − log 35 = −2
𝑥
log = −2
243
𝑥 1
= 10−2 =
243 100
243
𝑥= = 2.43
100
Part B
Take the natural log of both sides:
3
ln(𝑥 ln 𝑥 ) = ln(𝑒 (ln 𝑥) )
Use the power rule for logarithms:
(ln 𝑥)(ln 𝑥) = (ln 𝑥)3 ln 𝑒
Simplify:
(ln 𝑥)2 = (ln 𝑥)3
If ln 𝑥 = 0, equation is satisfied. And hence:
ln 𝑥 = 0 ⇒ 𝑥 = 1
2
If ln 𝑥 ≠ 0, divide both sides by (ln 𝑥)
1 = ln 𝑥 ⇒ 𝑥 = 𝑒
𝑥 ∈ {1, 𝑒}
Part C
Use the product rule on the RHS:
𝑚 + 2𝑛 1
log ( ) = (log 𝑚𝑛)
2√2 2
Use the power rule on the RHS:
𝑚 + 2𝑛
log ( ) = log √𝑚𝑛
2√2
Exponentiate both sides:
𝑚 + 2𝑛
= √𝑚𝑛
2√2
Clear fractions:
𝑚 + 2𝑛 = 2√2√𝑚𝑛
Square both sides both to remove the radical:
𝑚2 + 4𝑚𝑛 + 4𝑛2 = 8𝑚𝑛
Collate all terms on one side
𝑚2 − 4𝑚𝑛 + 4𝑛2 = 0
Factor
(𝑚 − 2𝑛)2 = 0
Take square roots, and solve for 𝑚:
𝑚 = 2𝑛

P a g e 28 | 89
Get all the files at: https://bit.ly/azizhandouts
Aziz Manva (azizmanva@gmail.com)

Part D
Take log to the base 2 both sides:
log 2 (22𝑥+1 ) = log 2 (3 × 21−2𝑥 )
Use log rules on both sides:
(2𝑥 + 1) log 2 2 = log 2(3) + (1 − 2𝑥) log 2 (2)
Substitute log 2 2 = 1 and simplify:
(2𝑥 + 1) = log 2 (3) + (1 − 2𝑥)
4𝑥 = log 2 (3)
log 2 (3)
𝑥=
4

Example 1.63
log 𝑎 log 𝑏 log 𝑐 𝑏2
Given 𝑝
= 𝑞
= 𝑟
= log 𝑥, all logarithms to the same base and 𝑥 ≠ 1. If 𝑎𝑐 = 𝑥 𝑦 , then 𝑦, in terms of 𝑝, 𝑞 and 𝑟
is: (AHSME 1967/4)

Rewrite:
1 1 1
log 𝑎 = log 𝑏 = log 𝑐 = log 𝑥
𝑝 𝑞 𝑟
Use the power rule:
1 1 1
log 𝑎𝑝 = log 𝑏 𝑞 = log 𝑐 𝑟 = log 𝑥
Exponentiate throughout:
1 1 1
𝑎𝑝 = 𝑏𝑞 = 𝑐 𝑟 = 𝑥
Solve for 𝑎, 𝑏, 𝑐 in terms of 𝑥:
1
𝑎𝑝 = 𝑥 ⇒ 𝑎 = 𝑥 𝑝
1
𝑏𝑞 = 𝑥 ⇒ 𝑏 = 𝑥 𝑞
1
𝑐𝑟 = 𝑥 ⇒ 𝑐 = 𝑥𝑟
Substitute the above:
𝑏 2 (𝑥 𝑞 )2
𝑥𝑦 = = = 𝑥 2𝑞−𝑝−𝑟
𝑎𝑐 𝑥 𝑝 𝑥 𝑟
If bases are same, exponents are also same:
𝑦 = 2𝑞 − 𝑝 − 𝑟

Example 1.64
Make 𝑥 the subject of the equation:
1
𝑦=
1 + 𝑒 −𝑥

1 1 1−𝑦
1 + 𝑒 −𝑥 = ⇒ 𝑒 −𝑥 = − 1 =
𝑦 𝑦 𝑦
Take the natural log of both sides:
1−𝑦 𝑦
𝑥 = − ln ( ) = ln ( )
𝑦 1−𝑦
C. Exponential Equations leading to Logs

Example 1.65

P a g e 29 | 89
Get all the files at: https://bit.ly/azizhandouts
Aziz Manva (azizmanva@gmail.com)

Solve 5𝑥 = 7 in terms of:


A. Log to the base 10
B. Log to the base 𝑒

Take log to the base 10:


log 7
𝑥 log 5 = log 7 ⇒ 𝑥 =
log 5
Take log to the base e:
ln 7
𝑥 ln 5 = ln 7 ⇒ 𝑥 =
ln 5

Example 1.66: Solving Exponential Equations via Logarithms


Find 𝑥𝑦 given that 3 = 5𝑥 , 5 = 3𝑦

log 3
3 = 5𝑥 ⇒ 𝑥 =
log 5
log 5
5 = 3𝑦 ⇒ 𝑦 =
log 3
log 3 log 5
𝑥𝑦 = × =1
log 5 log 3

Example 1.67: Solving Exponential Equations via Logarithms


In an exponential equation, it may be necessary to take logarithms in order get the variable down from the
exponent.
𝑧
( )
A. Solve for 𝑧 in terms of log to the base 10: 5 × 10 4 = 32
3𝑥 1
B. Solve for 𝑥: 3𝑥 +3 = 3
C. Solve for 𝑥 in terms of log 2 and log 3: 8𝑥−1 = 63𝑥
D. Find the value of 𝑥 in terms of ln 2: ln 24𝑥−1 = ln 8𝑥+5 + log 2 161−2𝑥

Part A 3
𝑥 = log 3
Divide by 5 both sides: 2
𝑧
( )32 Part C
10 4 = 23𝑥−3 = 23𝑥 ∙ 33𝑥
5
Take log to the base 10 both sides: 2−3 = 33𝑥
𝑧
( ) 32 2−1 = 3𝑥
log 10 4 = log
5 − log 2 = 𝑥 log 3
Use the power rule on the LHS: log 2
𝑧 32 32 𝑥=−
= log ⇒ 𝑧 = 4 log log 3
4 5 5 Part D
Part B 24𝑥−1
Cross-multiply to eliminate fractions: ln = (1 − 2𝑥) log 2 24
23𝑥+15
3 ∙ 3𝑥 = 3𝑥 + 3 ln 2𝑥−16 = 4 − 8𝑥
Collate all 𝑥 terms on the LHS: 𝑥 ln 2 − 16 ln 2 = 4 − 8𝑥
3 ∙ 3𝑥 − 3𝑥 = 3 𝑥 ln 2 + 8𝑥 = 4 + 16 ln 2
𝑥
Factor 3 : 16 ln 2 + 4
3𝑥 (3 − 1) = 3 𝑥=
ln 2 + 8
3
3𝑥 =
2

P a g e 30 | 89
Get all the files at: https://bit.ly/azizhandouts
Aziz Manva (azizmanva@gmail.com)

Example 1.68: Reducible to Quadratic


The equation 22𝑥 − 8 ∙ 2𝑥 + 12 = 0 is satisfied by:
A. log 3
1
B. 2
log 6
3
C. 1 + log 2
log 3
D. 1 + log 2
E. none of these (AHSME 1969/17)

Use a change of variable. Let 𝑦 = 2𝑥 : 2𝑥 = 6


𝑦 2 − 8𝑦 + 12 = 0 log 2𝑥 = log 6
(𝑦 − 6)(𝑦 − 2) = 0 𝑥 log 2 = log 2 + log 3
𝑦 = 2𝑥 ∈ {2,6} log 3
𝑥 =1+
Consider Case I: log 2
2𝑥 = 21 ⇒ 𝑥 = 1 Hence,
Consider Case II: 𝑂𝑝𝑡𝑖𝑜𝑛 𝐷 𝑖𝑠 𝑐𝑜𝑟𝑟𝑒𝑐𝑡

D. More Equations

Example 1.69: Simulataneous Equations


Solve each system of equations. First treat them as simultaneous equations, and then solve by converting them to
exponential equations.
𝑥
A. log 𝑦 = 2, log 𝑥𝑦 = 1
𝑥
B. ln 𝑦 = 2, ln 𝑥𝑦 = 1

Part A
Standard Method
log 𝑥 − log 𝑦 = 2 ,
⏟ log 𝑥 + log 𝑦 = 1

𝑬𝒒𝒖𝒂𝒕𝒊𝒐𝒏 𝑰 𝑬𝒒𝒖𝒂𝒕𝒊𝒐𝒏 𝑰𝑰
Add the two equations:
2 log 𝑥 = 3
log 𝑥 2 = 3
𝑥 2 = 1000
𝑥 = ±10√10
Substitute the value of log 𝑥 in the equation given in the original question:
log 𝑥𝑦 = 1
log(±10√10)𝑦 = 1
(±10√10)𝑦 = 10
1
𝑦=±
√10
Convert to exponential form
Convert to exponential form:
𝑥 𝑥
log = 2 ⇒ = 102 ⇒ ⏟
𝑥 = 100𝑦
𝑦 𝑦
𝑬𝒒𝒖𝒂𝒕𝒊𝒐𝒏 𝑰𝑰𝑰
log 𝑥𝑦 = 1 ⇒ 𝑥𝑦 = 10
Substitute 𝑥 from Equation VI in the above:

P a g e 31 | 89
Get all the files at: https://bit.ly/azizhandouts
Aziz Manva (azizmanva@gmail.com)

1 1
(100𝑦)(𝑦) = 10 ⇒ 𝑦 2 = ⇒𝑦=±
10 √10
Substitute the value of 𝑦 from above in Equation III:
1
𝑥 = 100𝑦 = 100 × ± = ±10√10
√10
Hence, the final answer is:
1 1
(𝑥, 𝑦) = {(10√10, ) , (−10√10, −
)}
√10 √10
Note: Both the negative and the positive solutions work (as you can check by substitution), since
(−𝑣𝑒)(−𝑣𝑒) = +𝑣𝑒
Part B
Standard Method
ln 𝑥 − ln 𝑦 = 2 ,
⏟ ln 𝑥 + ln 𝑦 = 1

𝑬𝒒𝒖𝒂𝒕𝒊𝒐𝒏 𝑰𝑽 𝑬𝒒𝒖𝒂𝒕𝒊𝒐𝒏 𝑽
Add the two equations:
2 ln 𝑥 = 3
ln 𝑥 2 = 3
𝑥2 = 𝑒3
3
𝑥 = ±𝑒 2
Substitute the value of 𝑥 in the question given in the original equation:
ln 𝑥𝑦 = 1
3
ln [(±𝑒 2 ) 𝑦] = 1
3
(±𝑒 2 ) 𝑦 = 𝑒
1
𝑦=±
√𝑒
Convert to exponential form
Convert to exponential form:
𝑥 𝑥
ln = 2 ⇒ = 𝑒2 ⇒ 𝑥
⏟= 𝑒 2 𝑦
𝑦 𝑦
𝐸𝑞𝑢𝑎𝑡𝑖𝑜𝑛 𝑉𝐼
Convert to exponential form:
ln 𝑥𝑦 = 1 ⇒ 𝑥𝑦 = 𝑒
Substitute 𝑥 from Equation VI in the above:
1 1
(𝑒 2 𝑦)(𝑦) = 𝑒 ⇒ 𝑦 2 = ⇒𝑦=±
𝑒 √𝑒
Substitute the value of y in Equation VI:
1 3
𝑥 = 𝑒2𝑦 = 𝑒2 × ± = ±𝑒 2
√𝑒
Hence, the final answer is:
3 1 3 1
(𝑥, 𝑦) = {(𝑒 2 , ) , (−𝑒 2 , − )}
√𝑒 √𝑒

Example 1.70
A. The product of all real roots of the equation 𝑥 log10 𝑥 = 10 is (AHSME 1984/14)
3
B. The product of all positive real values of 𝑥 satisfying the equation 𝑥 (16(log5 𝑥) −68 log5 𝑥) = 5−16 is
(JEE Advanced, 2022/Paper-II/4)
C. Are the answers to both Part A and B the same? Why?

P a g e 32 | 89
Get all the files at: https://bit.ly/azizhandouts
Aziz Manva (azizmanva@gmail.com)

Part A −16 − 1:
Take logs to the base 10 both sides: 4𝑧 2 − 16𝑧 − 𝑧 + 4 = 0
log10(𝑥 log10 𝑥 ) = log10 10 4𝑧(𝑧 − 4) − 1(𝑧 − 4) = 0
Use the power rule: (4𝑧 − 1)(𝑧 − 4) = 0
(log10 𝑥)(log10 𝑥) = 1 Use the zero-product property:
(log10 𝑥)2 = 1 1 1
4𝑧 − 1 = 0 ⇒ 𝑧 = 𝑦 2 = ⇒ 𝑦 = ±
log10 𝑥 = ±1 4 2
𝑥 = 10−1 𝑂𝑅 𝑥 = 101 𝑧 − 4 = 0 ⇒ 𝑧 = 𝑦 2 = 4 ⇒ 𝑦 = ±2
The product of the real roots is: The four solutions that we get for 𝑦 are:
𝟏𝟎−𝟏 ∙ 𝟏𝟎𝟏 = 𝟏𝟎𝟏−𝟏 = 𝟏𝟎𝟎 = 𝟏
1 1
Part B 𝑦 = log 5 𝑥 ∈ {− , , −2,2}
Use a change of variable. Let 𝑦 = log 5 𝑥: 2 2
3 −68𝑦 Exponentiate the four solutions above to get the
𝑥16𝑦 = 5−16 value of 𝑥:
Take log to the base 5 both sides: 1 1
16𝑦 3 −68𝑦 𝑥 ∈ {5−2 , 52 , 52 , 5−2 }
log 5 (𝑥 ) = log 5 (5−16 )
Use the power rule: Finally, the product of all positive real values of 𝑥 is:
1 1
(16𝑦 3 − 68𝑦) log 5 𝑥 = −16 log 5 (5) 5−2+2+2−2 = 50 = 1
(16𝑦 3 − 68𝑦)𝑦 = −16 Part C
Expand and collate terms on LHS: The equations are of the form
16𝑦 4 − 68𝑦 2 + 16 = 0 (log 𝑏 𝑥)2 = 𝑎2
Divide by 4 both sides: log b 𝑥 = ±𝑎
4𝑦 4 − 17𝑦 2 + 4 = 0 Resulting in answer pairs of the form
This is a disguised quadratic. Again, use a change of 𝑥 = 𝑏 𝑎 𝑂𝑅 𝑥 = 𝑏 −𝑎
variable. Let 𝑧 = 𝑦 2 : And their product is:
4𝑧 2 − 17𝑧 + 4 = 0 𝑏 𝑎 𝑏−𝑎 = 1
Factor. 𝑃𝑟𝑜𝑑𝑢𝑐𝑡 = 16 = (−16)(−1), 𝑆𝑢𝑚 = −17 =

E. Inequalities

Example 1.71: Linear Inequalities


Solve the inequalities below:
1
A. log 𝑥 ≥ log 2 + 2 log 𝑥 (AHSME 1958/17)
B. −6 < 4 log 5 (𝑥) − 10 < −2

Part A Add 10:


1 4 < 4 log 5 (𝑥) < 8
Subtract 2 log 𝑥 from both sides:
1 Divide by 4:
log 𝑥 ≥ log 2 1 < log 5 (𝑥) < 2
2
log 𝑥 ≥ 2 log 2 = log 22 = log 4 Exponentiate to the base 5:
𝑥≥4 5 < 𝑥 < 25
Part B 𝑥 ∈ (5,25)

Example 1.72: Quadratic Inequalities


Solve the inequalities below:
A. (ln 𝑥)2 − (ln 2)(ln 𝑥) < 2(ln 2)2

P a g e 33 | 89
Get all the files at: https://bit.ly/azizhandouts
Aziz Manva (azizmanva@gmail.com)

Use a change of variable. Substitute 𝑦 = ln 𝑥 , 𝑧 = ln 2:


𝑦 2 − 𝑧𝑦 − 2𝑧 2 < 0
Use 𝑆𝑢𝑚 = −2𝑧 + 𝑧 = −𝑧, 𝑃𝑟𝑜𝑑𝑢𝑐𝑡 = (−2𝑧)(𝑧) = −2𝑧 2 :
(𝑦 + 𝑧)(𝑦 − 2𝑧) < 0
(𝑦 + 𝑧)(𝑦 − 2𝑧) is an upward parabola with roots:
1 1
𝑦 + 𝑧 = 0 ⇒ 𝑦 = −𝑧 ⇒ ln 𝑥 = − ln 2 ⇒ ln 𝑥 = ln ⇒ 𝑥 =
2 2
𝑦 − 2𝑧 = 0 ⇒ 𝑦 = 2𝑧 ⇒ ln 𝑥 = 2 ln 2 ⇒ ln 𝑥 = ln 4 ⇒ 𝑥 = 4
Negative region will be between the roots.
1
<𝑥<4
2

Example 1.73: Range


If one uses only the tabular information 103 = 1000, 104 = 10,000, 210 = 1024, 211 = 2048, 212 = 4096, 213 =
8192, then the strongest statement one can make for log10 2 is that it lies between:
3 4
A. 𝑎𝑛𝑑
10 11
3 4
B. 𝑎𝑛𝑑
10 12
3 4
C. 10
𝑎𝑛𝑑 13
3 40
D. 10
𝑎𝑛𝑑 132
3 40
E. 11
𝑎𝑛𝑑 132 (AHSME 1967/26)

We establish a lower and upper bound. 8192 < 10,000


First, note that: log10 8192 < log10 10,000
1000 < 1024 log10 213 < log10 104
log10 1000 < log10 1024 13 log10 2 < 4
log10 103 < log10 210 4
log10 2 <
3 < 10 log10 2 13
3 This gives us an upper bound.
< log10 2 Combine the two to get:
10
This gives us a lower bound. 3 4
< log10 2 <
Then: 10 13

1.74: Interchanging log and exponentiation (not valid)


In general:
log 𝑛 𝑥 𝑛 ≠ (log 𝑛 𝑥)𝑛

In general, the order in which we perform the operations is going to matter.

In log 𝑛 𝑥 𝑛 , we need to perform the exponentiation first, and take the log next.
In (log 𝑛 𝑥)𝑛 , we need to take the log first and perform the exponentiation next.

This also forms a basis of a set of questions in relation to interchanging the operations actually does not matter. For
what value of x does this hold. We look at these questions next.

Example 1.75
Solve log 𝑛 (𝑥 𝑛 ) = (log 𝑛 𝑥)𝑛 for 𝑛 = 2,3,4, …

P a g e 34 | 89
Get all the files at: https://bit.ly/azizhandouts
Aziz Manva (azizmanva@gmail.com)

±√3 = log 3 𝑥 ⇒ 𝑥 = 3±√3


𝑥 = 1 ⇒ log 𝑛 𝑥 = 0 is a solution for all 𝑛 > 0, 𝑛 ≠ 1 Case IV: 𝒙 ≠ 𝟏, 𝒏 = 𝟒
4 log 4 𝑥 = (log 4 𝑥)4
Case II: 𝒙 ≠ 𝟏, 𝒏 = 𝟐 Divide both sides by log 4 𝑥:
Use the property that log 𝑥 𝑛 = 𝑛 log 𝑥: 3 3
4 = (log 4 𝑥)3 ⇒ √4 = log 4 𝑥 ⇒ 𝑥 = 4 √4
2 log 2 𝑥 = (log 2 𝑥)2
General Case
Divide both sides by log 2 𝑥:
The above solutions show the pattern: even 𝑛 results
2 = log 2 𝑥 ⇒ 𝑥 = 22 = 4
in an odd root, while odd 𝑛 results in two solutions.
Case III: 𝒙 ≠ 𝟏, 𝒏 = 𝟑
𝑛 log 𝑛 𝑥 = (log 𝑛 𝑥)𝑛 ⇒ 𝑛 = (log 𝑛 𝑥)𝑛−1
3 log 3 𝑥 = (log 3 𝑥)3 𝑛−1 𝑛−1

Divide both sides by log 3 𝑥: Even 𝑛: √𝑛 = log 𝑛 𝑥 ⇒ 𝑥 = 𝑛 √𝑛


𝑛−1 𝑛−1
3 = (log 3 𝑥)2 Odd 𝑛: ± √𝑛 = log 𝑛 𝑥 ⇒ 𝑥 = 𝑛± √𝑛
Take the square root both sides:

F. Power Rule Extension


The power rule has a supercharged version that is useful.

1.76: Shortcut
𝑚 𝑚
log 𝑎𝑛 𝑥 𝑚 = log 𝑎 𝑥 𝑛 = log 𝑎 𝑥
𝑛

We can prove the power rule extension with using the change of base of rule (which is introduced later):
log 𝑥 𝑚 𝑚 log 𝑥 𝑚
log 𝑎𝑛 𝑥 𝑚 = = = log 𝑎 𝑥
log 𝑎𝑛 𝑛 log 𝑎 𝑛
Many questions can be solved faster with this shortcut.

Example 1.77: Basics


A. Simplify log 3 𝑥 4 − 4 log 9 𝑥
B. If 𝑎 = log 8 225 and 𝑏 = log 2 15, then find the relation between 𝑎 and 𝑏 (AHSME 1962/17, AHSME 1970/8)
1
C. Solve for 𝑁: log 3 𝑁 + log 9 𝑁 = 4
2

Part A
log 3 𝑥 4 − 4 log 32 𝑥
Use the power rule extension in the second term:
4
4 log 3 𝑥 − log 3 𝑥
2
4 log 3 𝑥 − 2 log 3 𝑥
2 log 3 𝑥
Part B
2 2
𝑎 = log 23 152 = log 2 15 = 𝑏
3 3
Part C
1 9
log 3 𝑁 + log 3 𝑁 =
2 2
3 9
log 3 𝑁 =
2 2
log 3 𝑁 = 3
𝑁 = 33 = 27

P a g e 35 | 89
Get all the files at: https://bit.ly/azizhandouts
Aziz Manva (azizmanva@gmail.com)

Example 1.78: Expressions


A. Simplify log 81 93 + log √𝑥 𝑥 3 + log 22√𝜋 𝜋 9
B. Evaluate log √2 2 × log √ 2 4 × log 8
√ √√√2

Part A Hence, the final answer is:


Each term follows a pattern: get log 𝑎 𝑎, and then 3
+ 6 + 198 = 205.5
replace it with 1: 2
6 3 Part B
log 34 36 = log 3 3 = log 1 2 × log 1 22 × log 1 23
4 2
22 24 28
3
log (1) 𝑥 3 = log 𝑥 𝑥 = 6 2 log 2 2 × 8 log 2 2 × 24 log 2 2
𝑥 2 1⁄
2 = 2 × 8 × 24 = 384
9
9
log ( 1 ) 𝜋 = log 𝜋 𝜋 = 198
𝜋 22 1⁄
22

Example 1.79: Equations


1
A. log 𝑥 2 √7 = log 3√5
√5
B. 2 − log 3 (𝑥 + 6) = log 1 4𝑥 2
9
C. log 𝑚 324 − log √𝑚 2𝑚 = 2
D. log 2 (𝑥 − 2) = log 4 (2𝑥 2 − 6𝑥 − 4)
E. log 𝑎 (𝑥 + 2) + log √𝑎 √𝑥 + 2 = log 𝑎2 900
F. 1 − log √3 (𝑥 + 6) = log 1 (−𝑥)
3

Part A 324
1 1 = 𝑚2 ⇒ 𝑚4 = 81 ⇒ 𝑚 = ±3
− 4𝑚2
log 𝑥 2 72 = log 15 2
53
1 3 𝑚 = −3 ⇒ log √𝑚 2𝑚 = log √−3 −6 ⇒ 𝑁𝑜𝑡 𝑉𝑎𝑙𝑖𝑑
log 𝑥 7 = − log 5 5
4 2 𝑚=3
log 𝑥 7 = −6 Part D
1 1 1
𝑥 = 7− 6 = 6 log 2(𝑥 − 2) = log 2(2𝑥 2 − 6𝑥 − 4)
√7 2
Part B 2 log 2 (𝑥 − 2) = log 2(2𝑥 2 − 6𝑥 − 4)
log 1 4𝑥 2 + log 3 (𝑥 + 6) = 2 log 2(𝑥 − 2)2 = log 2 (2𝑥 2 − 6𝑥 − 4)
9
𝑥 2 − 4𝑥 + 4 = 2𝑥 2 − 6𝑥 − 4
log 3−2 (2𝑥)2 + log 3 (𝑥 + 6) = 2
𝑥 2 − 2𝑥 − 8 = 0
− log 3 2𝑥 + log 3 (𝑥 + 6) = 2
𝑥+6 (𝑥 − 4)(𝑥 + 2) = 0
log 3 ( )=2 𝑥 ∈ {−2,4}
2𝑥
𝑥+6
=9 −2 𝑑𝑜𝑒𝑠 𝑛𝑜𝑡 𝑤𝑜𝑟𝑘
2𝑥
𝑥 + 6 = 18𝑥 4 𝑑𝑜𝑒𝑠 𝑤𝑜𝑟𝑘
6 Part E
𝑥= log 𝑎 [(𝑥 + 2)(𝑥 + 2)] = log 𝑎 30
17
Part C (𝑥 + 2)2 = 30
log 𝑚 324 − log 𝑚 (2𝑚)2 = 2 𝑥 + 2 = ±√30
324 𝑥 = ±√30 − 2
log 𝑚 =2
4𝑚2 √30 − 2 𝑤𝑜𝑟𝑘𝑠

P a g e 36 | 89
Get all the files at: https://bit.ly/azizhandouts
Aziz Manva (azizmanva@gmail.com)

−√30 − 2 𝑑𝑜𝑒𝑠 𝑛𝑜𝑡 𝑤𝑜𝑟𝑘 (𝑥 + 3)(𝑥 + 12) = 0


Part F 𝑥 ∈ {−12, −3}
log 1 (𝑥 + 6) + log 3−1 (−𝑥) = 1 Check −12:
32
LHS = 1 − log √3(−12 + 6) = 1 − log √3(−6)
2 log 3 (𝑥 + 6) − log 3(−𝑥) = 1
𝑁𝑜𝑡 𝑉𝑎𝑙𝑖𝑑
log 3 (𝑥 + 6)2 − log 3 (−𝑥) = 1
Check −3:
(𝑥 + 6)2
log 3 [ ]=1 LHS = 1 − log √3(−3 + 6) = 1 − log √3 3 = 1 − 2
−𝑥
= −1
(𝑥 + 6)2
=3 𝑅𝐻𝑆 = log 1 (−(−3)) = log 1 3 = 1 = 𝐿𝐻𝑆
−𝑥 3 3
𝑥 2 + 12𝑥 + 36 = −3𝑥 𝑉𝑎𝑙𝑖𝑑 𝑆𝑜𝑙𝑢𝑡𝑖𝑜𝑛

Example 1.80
1
(2𝑥)3
0.162 = −0.06 log10 [ ] , log10 2 = 0.3
0.1
If 𝑥 = 10𝑚 , 𝑚 ∈ ℝ and 𝑥 can be approximated as 4 × 10𝑛 , 𝑛 ∈ ℤ, then find 𝑚 + 𝑛.

Divide both sides by 0.06 and use the quotient rule on the RHS:
0.162 1
− = log10 [(2𝑥)3 ] − log10 [0.1]
0.06

Simplify the LHS, and use the power rule on the LHS:
1
−2.7 = log10[2𝑥] + 1
3

Subtract 1 from both sides and substitute log10 2 = 0.3:


1
−3.7 = (0.3 + log10 𝑥)
3
−11.4 = log10 𝑥

Exponentiate both sides:


𝑥 = 10−11.4 ≈ 4 × 10−12

𝑚 + 𝑛 = −11.4 − 12 = −23.41

Challenge 1.81: Advanced Systems


Determine the value of 𝑎𝑏 if log 8 𝑎 + log 4 𝑏 2 = 5 and log 8 𝑏 + log 4 𝑎2 = 7. (AIME 1984/5)

The two equations are cyclic. This gives us the idea of adding them:
log 8 𝑎 + log 4 𝑏 2 + log 8 𝑏 + log 4 𝑎2 = 5 + 7
Two of the terms have a base of 8, and the other two have a base of 4. We want to have a common base, so use the
1
property that log 𝑎𝑛 𝑥 = 𝑛 log 𝑎 𝑥:
1 2 1 2
log 2 𝑎 + log 2 𝑏 + log 2 𝑏 + log 2 𝑎 = 12
3 2 3 2
Add like terms:

1
Equations of this type are encountered in electrochemistry.

P a g e 37 | 89
Get all the files at: https://bit.ly/azizhandouts
Aziz Manva (azizmanva@gmail.com)

4 4
log 2 𝑎 + log 2 𝑏 = 12
3 3
4
Factor 3 and use log 𝑎 + log 𝑏 = log 𝑎𝑏
4
log 2 𝑎𝑏 = 12 ⇒ log 2 𝑎𝑏 = 9
3
Take anti-logs both sides:
𝑎𝑏 = 29 = 512

Example 1.82
Find (log 2 𝑥)2 if log 2(log 8 𝑥) = log 8 (log 2 𝑥)(AIME 1988/3)

Convert all bases to 2:


1 1
log 2 ( log 2 𝑥) = log 2 (log 2 𝑥)3
3
Take Anti-Log both sides:
1 1
log 2 𝑥 = (log 2 𝑥)3
3
Use a change of variable. Substitute 𝑦 = log 2 𝑥:
1 1 𝑦 2
𝑦 = 𝑦 3 ⇒ 1 = 3 ⇒ 𝑦 3 = 3 ⇒ 𝑦 2 = 27
3
𝑦3
Change back to the original variable:
(log 2 𝑥)2 = 27

Example 1.83: Inequalities


A. Solve: 5 < 6 + log 0.25 (𝑥) < 7

Subtract 6: Exponentiate to the base 4:


−1 < log 4−1 (𝑥) < 1 1
4>𝑥>
−1 < − log 4 (𝑥) < 1 4
Multiply by −1: 1
𝑥 ∈ ( , 4)
1 > log 4 (𝑥) > −1 4

G. Power to a Logarithm

1.84: Logs in the exponent cancel with the base


𝑎log𝑎 𝑥 = 𝑥

Start with log 𝑎 𝑥, and then recognize that 𝑥 = 𝑎𝑚 for some 𝑚:


log 𝑎 𝑥 = log 𝑎 𝑎𝑚 = 𝑚 log 𝑎 𝑎 = 𝑚 ⇒ 𝑎log𝑎 𝑥 = 𝑎𝑚 = 𝑥

1
2× =1
2
2
(√2) = 2

Example 1.85
2 )+log 1
52 log5 (3𝑥 5

P a g e 38 | 89
Get all the files at: https://bit.ly/azizhandouts
Aziz Manva (azizmanva@gmail.com)

Use the power rule:


2 2
5log5 (3𝑥 ) +log5 1
4
= 5log5 (9𝑥 )+log5 1
4
= 5log5 (9𝑥 )
= 9𝑥 4

Example 1.86
Simplify:
A. 10log10 7 (AHSME 1951/34)
B. 25log5 7
C. 7log√7 12
D. 𝑒 ln 𝑥+ln 𝑦+ln 𝑧

10log10 7 = 7
2
25log5 7 = 52 log5 7 = 5log5 7 = 72 = 49
2
7log√7 12 = 72 log7 12 = 7log7 12 = 144
𝑒 ln 𝑥+ln 𝑦+ln 𝑧 = 𝑒 ln 𝑥𝑦𝑧 = 𝑥𝑦𝑧

Example 1.87
2
Find the sum of the last three non-zero digits of the decimal representation of (9log3 √17 ) .

2 2 2 2
(9log3 √17 ) = ((32 )log3 √17 ) = (32×log3 √17 ) = (3log3 17 ) = (17)2 = 289
Hence, the sum of the last three digits is:
2 + 8 + 9 = 19

Example 1.88
Eliminate Logs
log10 𝑦 = 𝑎(log10 𝑥) + 𝑏

Exponentiate both sides to the power 10:


10log10 𝑦 = 10𝑎(log10 𝑥)+𝑏
Split
𝑎
𝑦 = 10(log 𝑥 ) × 10𝑏 = 10𝑏 𝑥 𝑎
𝑦 = 10𝑏 𝑥 𝑎

1.89: Interchanging
𝑥 log𝑎 𝑦 = 𝑦 log𝑎 𝑥

Let 𝑥 = 𝑎𝑚 , 𝑦 = 𝑎𝑛 :
𝑛 𝑚
𝑥 log𝑎 𝑦 = (𝑎𝑚 )log𝑎 𝑎 = 𝑎𝑚𝑛 log𝑎 𝑎 = (𝑎𝑛 )log𝑎 𝑎 = 𝑦 log𝑎 𝑥

Example 1.90
Evaluate 𝑥 log𝑦 𝑧 − 𝑧 log𝑦 𝑥

𝑥 log𝑦 𝑧 − 𝑧 log𝑦 𝑥 = 𝑧 log𝑦 𝑥 − 𝑧 log𝑦 𝑥 = 0

P a g e 39 | 89
Get all the files at: https://bit.ly/azizhandouts
Aziz Manva (azizmanva@gmail.com)

Example 1.91
3
𝑥 ln 𝑥 = 𝑒 (ln 𝑥)

Substitute 𝑥 = 𝑒 ln 𝑥 :
ln 𝑥 3
(𝑒 ln 𝑥 ) = 𝑒 (ln 𝑥)
2 3
𝑒 (ln 𝑥) = 𝑒 (ln 𝑥)
(ln 𝑥)2 = (ln 𝑥)3
If ln 𝑥 = 0, equation is satisfied. And hence:
ln 𝑥 = 0 ⇒ 𝑥 = 1
2
If ln 𝑥 ≠ 0, divide both sides by (ln 𝑥) :
1 = ln 𝑥 ⇒ 𝑥 = 𝑒
Final Solution Set:
𝑥 ∈ {1, 𝑒}
1.6 Change of Base
A. Basics
The change of base rule lets you split a logarithm and let you take it to any base you want.

1.92: Change of Base Rule


log 𝑏 𝑥
log 𝑎 𝑥 = , 𝑤ℎ𝑒𝑟𝑒 𝑏 > 0, 𝑏 ≠ 1
log 𝑏 𝑎

𝑘 = log 𝑎 𝑥
Convert to exponential form:
𝑎𝑘 = 𝑥
Take 𝑙𝑜𝑔 base 𝑏 both sides:
log 𝑏 𝑎𝑘 = log 𝑏 𝑥
Use the power rule:
𝑘 log 𝑏 𝑎 = log 𝑏 𝑥
Solve for 𝑘:
log 𝑏 𝑥
𝑘=
log 𝑏 𝑎
Substitute the original value of 𝑘
log 𝑏 𝑥
log 𝑎 𝑥 =
log 𝑏 𝑎

Example 1.93
4
log 1 √27
3

4
1 3 3
log √27 log(33 )4
= =
log 34
= 4 log 3 = − 3
1 log 3−1 log 3−1 − log 3 4
log
3

Example 1.94

Evaluate:

P a g e 40 | 89
Get all the files at: https://bit.ly/azizhandouts
Aziz Manva (azizmanva@gmail.com)

ln 27
A. ln 3
ln 8
B. ln 4
ln 144
C. ln 2√3
1
D. log 81 27
Write as a Fraction using Change of Base
E. log12 7

Parts A-C We could also have done this using the Power Rule
ln 27 log 𝑒 27 Extension that we already learnt. The method below
= = log 3 27 = 3
ln 3 log 𝑒 3 is usually faster:
ln 8 3 1 3 3
= log 4 8 = log 81 = log 34 3−3 = − log 3 3 = −
ln 4 2 27 4 4
ln 144 4 Part E
= log 2√3 24 32 = log 2√3 (2√3) = 4 log 7
ln 2√3 log12 7 =
Part D log 12
1
log 27 log 3−3 −3 log 3 3
= = =−
log 81 log 34 4 log 3 4

Example 1.95: Calculator


Calculators usually have a 𝑙𝑜𝑔 button which gives logarithms to the base 10. And they also have a 𝑙𝑛 button, which
gives the logarithm to the base 𝑒. However, calculators will not directly calculate the logarithm to an arbitrary base.
The change of base rule is useful here.

Use the change of base to find the approximate value of the following on a calculator.
A. log 7 3
B. log 3 45.2

log10 3
log 7 3 = =
𝑙𝑜𝑔10 7
log10 45.2
log 3 45.2 = =
𝑙𝑜𝑔10 3

Example 1.96
If log10 2 = 𝑎 and log10 3 = 𝑏, then
A. log 5 12 in terms of 𝑎 and 𝑏 =? (AHSME 1961/30)
B. then log125 24 in terms of 𝑎 and 𝑏 =? (AHSME 1961/30)

Part A
Use change of base:
log 12
=
log 5
Use the product rule
log 4 + log 3
=
log 10 − log 2
Substitute:

P a g e 41 | 89
Get all the files at: https://bit.ly/azizhandouts
Aziz Manva (azizmanva@gmail.com)

2𝑎 + 𝑏
=
1−𝑎
Part B
log 24 log 8 + log 3 3𝑎 + 𝑏
= = =
log 125 log 1000 − log 8 3 − 3𝑎

Example 1.97
𝑥 = log 𝑎 𝑦
A. Solve for 𝑦
B. Solve for 𝑎

𝑦 = 𝑎𝑥
1
Raise both sides to the power 𝑥:
1
𝑦𝑥 = 𝑎

Example 1.98
log𝑏(log𝑏 𝑎)
If 𝑎 > 1, 𝑏 > 1, and 𝑝 = log𝑏 𝑎
, then 𝑎𝑝 equals (AHSME 1982/13)

log𝑏(log𝑏 𝑎)
𝑎 = 𝑎 log𝑏 𝑎
𝑝

Use change of base:


= 𝑎log𝑎 (log𝑏 𝑎)
The power and the log cancel:
= log 𝑏 𝑎

1.99: Reciprocal of a Log


1
𝑥+ = 𝑐 ⇒ 𝑄𝑢𝑎𝑑𝑟𝑎𝑡𝑖𝑐
𝑥

If one term in an equation is the reciprocal of another term, use a change of variable, and this leads to a quadratic.

Example 1.100: Golden Ratio


𝑞
Suppose that 𝑝 and 𝑞 are positive numbers for which log 9 𝑝 = log12 𝑞 = log16(𝑝 + 𝑞). What is the value of 𝑝?
(AHSME 1988/26)

Use the change of base rule:


log 𝑝 log 𝑞 log(𝑝 + 𝑞)
= =
log 9 log 12 log 16
Cross-multiply in the first and the second equality:
log 12 log 𝑝 = 2 log 3 log 𝑞

𝑬𝒒𝒖𝒂𝒕𝒊𝒐𝒏 𝑰
Cross-multiply in the second and the third equality:
log 12 log(𝑝 + 𝑞) = 2 log 4 log 𝑞

𝑬𝒒𝒖𝒂𝒕𝒊𝒐𝒏 𝑰𝑰
Add Equation I and II:
log 12 log[𝑝(𝑝 + 𝑞)] = 2 log 12 log 𝑞

P a g e 42 | 89
Get all the files at: https://bit.ly/azizhandouts
Aziz Manva (azizmanva@gmail.com)

Divide both sides by log 12:


log[𝑝(𝑝 + 𝑞)] = log 𝑞 2

Exponentiate both sides:


𝑝(𝑝 + 𝑞) = 𝑞 2
Divide both sides by 𝑝𝑞:
𝑝 𝑞
+1=
𝑞 𝑝
𝑞
Use a change of variable. Let 𝑝 = 𝑥:
1
+1=𝑥
𝑥
Multiply both sides by 𝑥:
1 + 𝑥 = 𝑥2
Collate all terms on one side:
𝑥2 − 𝑥 − 1 = 0
Use the quadratic formula:
1 + √5
𝑥=
2

B. Cancelling Bases

1.101: Cancelling Bases


log 𝑥 log 𝑏 log 𝑥
log 𝑏 𝑥 ∙ log 𝑎 𝑏 = × = = log 𝑎 𝑥
log 𝑏 log 𝑎 log 𝑎

If we have:
➢ A term with 𝑏 in the base
➢ A term where we are finding the logarithm of 𝑏
Then we can use the change of base rule to eliminate 𝑏 from the expression.

Example 1.102: Expressions


Simplify
A. log 4 5 ∙ log 5 4
B. log 𝑎 𝑏 ∙ log 𝑏 𝑎 (AHSME 1953/39)
C. log 2 2 × log √2 × log √√2
√ √√2 √√√2

D. log 2 3 × log 3 4 × … × log127 128

Part A
log 5 log 4 log 5
log 4 5 ∙ log 5 4 = × = =1
log 4 log 5 log 5
Part B
log 𝑏 log 𝑎
× =1
log 𝑎 log 𝑏
Part C
Use the change of base rule:

P a g e 43 | 89
Get all the files at: https://bit.ly/azizhandouts
Aziz Manva (azizmanva@gmail.com)

log 2 log √2 log √√2


× ×
log √2 log √√2
log √√√2
Cancel and simplify:
log 2 log 2 log 2
= 1 = =8
1
log 28 log 2
log √√√2 8
Part D
log 2 128 = 7

Example 1.103: Expressions


If 𝑎 = log16 7, and 𝑏 = log 7 6, then find log 3 2 in terms of 𝑎 and 𝑏.

Wrong Approach
We could add the equations, but then we don’t have a way to add the logs since the bases are different:
𝑎 + 𝑏 = log16 7 + log 7 6
Correct Approach
The number in the first equation is the base in the second equation. Hence, multiply the two equations:
𝑎𝑏 = log16 𝟕 × log 𝟕 6
Simplify the RHS:
log 24 6
1
= log 2 6
4
1
= (log 2 2 + log 2 3)
4
1
= (1 + log 2 3)
4
Now, solve for the expression we want:
1
𝑎𝑏 = (1 + log 2 3)
4
log 2 3 = 4𝑎𝑏 − 1
1
log 3 2 =
4𝑎𝑏 − 1

Example 1.104
log 63 64 × log 62 63 × … × log 2 3

Use change of base:


log 64 log 63 log 3
× × …×
log 63 log 62 log 2

Telescope:
log 64
log 2

Use reverse change of base:


log 2 64 = 6

Example 1.105: Equations

P a g e 44 | 89
Get all the files at: https://bit.ly/azizhandouts
Aziz Manva (azizmanva@gmail.com)

A. Solve for 𝑥: log 𝑘 𝑥 ∙ log 5 𝑘 = 3 (AHSME 1958/25)


B. Which positive numbers 𝑥 satisfy the equation (log 3 𝑥)(log 𝑥 5) = log 3 5 (AHSME 1975/19)
C. Eliminate 𝑧 from the system of equations 𝑥 𝑧 = 𝑦, and 𝑧 = log 𝑦 𝑥 4 . Your answer should not be in terms of
logarithms.

Part A Substitute the above in 𝑥 𝑧 = 𝑦:


log 𝑥 log 𝑘 4
log 𝑥
× =3 𝑥 log 𝑦 =𝑦
log 𝑘 log 5
log 𝑥 Take log both sides:
=3 log 𝑥
log 5 4 log 𝑥 = log 𝑦
log 5 𝑥 = 3 log 𝑦
𝑥 = 53 = 125 log 𝑦 2
4=( )
Part B log 𝑥
log 𝑥 log 5 log 5 log 𝑦
× = = log 3 5 ±2 = = log 𝑥 𝑦
log 3 log 𝑥 log 3 log 𝑥
This is valid for any valid value of 𝑥, which is: Convert from logarithmic to exponential form:
𝐴𝑙𝑙 𝑝𝑜𝑠𝑖𝑡𝑖𝑣𝑒 𝑛𝑢𝑚𝑏𝑒𝑟𝑠 𝑒𝑥𝑐𝑒𝑝𝑡 1 1
𝑦 = 𝑥 2 𝑂𝑅 𝑦 = 2
Part C 𝑥
log 𝑥 Part D
𝑧 = log 𝑦 𝑥 4 = 4 log 𝑦 𝑥 = 4
log 𝑦

Example 1.106: Equations


Given that 2𝑥 = 3𝑥+1 , find the value of 𝑥 in the form log 𝑏 𝑎.

2𝑥 = 3 ∙ 3𝑥
2𝑥
=3
3𝑥
𝑥
2
( ) =3
3
𝑥 = log 2 3
3
Taking Logs First
Taking logs first is also valid, but far lengthier. 𝑁𝑜𝑡 recommended.
𝑥 log 2 2 = (𝑥 + 1) log 2 3
𝑥(1 − log 2 3) = log 2 3
log 2 3 log 2 3 log 2 3
𝑥= = = = log 2 3
1 − log 2 3 log 2 2 − log 2 3 log 2 3
23

1
A solution of the above equation obtained (log as a solution. Is the answer correct?
3 2)−1

1 1 1
= = = log 2 3 2
(log 3 2) − 1 (log 3 2) − log 3 3 2 3
log 3 3

Example 1.107

2
This step makes use of the reciprocal of a log property. See the next section for the property.

P a g e 45 | 89
Get all the files at: https://bit.ly/azizhandouts
Aziz Manva (azizmanva@gmail.com)

A. For what values of 𝑥 is log10 𝑥 = ln 𝑥

Use the change of base rule: log 𝑥 = 0 ⇒ 𝑥 = 1


log 𝑥 log 𝑥 Case II: 𝐥𝐨𝐠 𝒙 ≠ 𝟎
=
log 10 log 𝑒
⏟ If log 𝑥 ≠ 0, then we can divide both sides of Equation
𝑬𝒒𝒖𝒂𝒕𝒊𝒐𝒏 𝑰 I by log 𝑥:
We 𝑤𝑎𝑛𝑡 to divide both sides by log 𝑥 to eliminate it. 1 1
=
Apply casework. log 10 log 𝑒
Take the reciprocal:
Case I: 𝐥𝐨𝐠 𝒙 = 𝟎 log 𝑒 = log 10
Substitute log 𝑥 = 0 in Equation I: But this is never true. Hence, there are no solutions
0 from this case.
𝐿𝐻𝑆 = =0
log 10
0 Hence, the final answer is:
𝑅𝐻𝑆 = =0
log 𝑒 𝑥=1
Hence, log 𝑥 = 0 is a solution:

1.7 Change of Base: Reciprocals


A. Expressions

1.108: Reciprocal of a Log


1
log 𝑎 𝑥 =
log 𝑥 𝑎

Apply the change of base rule to get:


log 𝑥
log 𝑥 log 𝑎
log 𝑎 𝑥 = =
log 𝑎 1
log 𝑎
Multiply both numerator and denominator by log 𝑥 :
log 𝑥 log 𝑥 log 𝑎
× 1 1
log 𝑎 log 𝑎 log 𝑥
= = =
1 log 𝑎 log 𝑎 log 𝑥 𝑎

log 𝑥 log 𝑥

Example 1.109
1 1 1
Evaluate 1 – 1 − 1 (Purple Comet 2003/6)
log2 ( ) log3 ( ) log4 ( )
6 6 6

Move each term to the numerator:


log 1 2− log 1 3− log 1 4
( ) ( ) ( )
6 6 6
Combining the terms using the reverse of the quotient rule:
2 1
log 1 ( ) = log 1 ( ) = 1
( ) 3∙4 ( ) 6
6 6

Example 1.110
1 1 1 1
For all positive numbers 𝑥 distinct from 1, log + log + log equals 𝑛 . Find 𝑛. (AHSME 1978/21, Adapted)
3𝑥 4𝑥 5𝑥

P a g e 46 | 89
Get all the files at: https://bit.ly/azizhandouts
Aziz Manva (azizmanva@gmail.com)

Use the reciprocal rule:


1
= log 𝑥 3 + log 𝑥 4 + log 𝑥 5
𝑛
Use the product rule:
1
= log 𝑥 60
𝑛
Again, use the reciprocal rule:
𝑛 = log 60 𝑥

Example 1.111
For what values of 𝑎, 𝑏, 𝑐, 𝑥 is
1 1 1 1
+ + =
log 𝑎 𝑥 log 𝑏 𝑥 log 𝑐 𝑥 log 𝑎𝑏𝑐 𝑥

Use the reciprocal rule:


log 𝑥 𝑎 + log 𝑥 𝑏 + log 𝑥 𝑐
Use the product rule:
log 𝑥 𝑎𝑏𝑐
Again, use the reciprocal rule:
1
log 𝑎𝑏𝑐 𝑥
Hence, this is true for all valid values:
𝑎 > 0, 𝑏 > 0, 𝑐 > 0, 𝑥 > 0
𝑎 ≠ 1, 𝑏 ≠ 1, 𝑐 ≠ 1, 𝑥 ≠ 1

Example 1.112: Expressions


If log 8 3 = 𝑝 and log 3 5 = 𝑞, then, in terms of 𝑝 and 𝑞, log10 5 equals (AHSME 1974/18)

One of the bases is already 3. We take the reciprocal of the other to make its base also 3:
1 1
log 3 8 = ⇒ log 3 2 =
𝑝 3𝑝
Now, we can use the change of base rule on the expression we want:
log 3 5 log 3 5
log10 5 = =
log 3 10 log 3 2 + log 3 5
Substitute the values, and simplify:
𝑞 𝑞 3𝑝𝑞
= =
1 1 + 3𝑝𝑞 1 + 3𝑝𝑞
3𝑝 + 𝑞 3𝑝

Example 1.113: Expressions


Given that log10 5 = 𝑝, log 3 2 = 𝑞, find log 3 5 in terms of 𝑝 and 𝑞.

Use the change of base rule to rewire the required expression:


log10 5 𝑝
log 3 5 = =
log10 3 log10 3

The numerator is directly 𝑝. To find the denominator, we need a little more work. Note that
log10 2
𝑞=
log10 3

P a g e 47 | 89
Get all the files at: https://bit.ly/azizhandouts
Aziz Manva (azizmanva@gmail.com)

Rearrange to get:
10
log10 2 log10 5 log10 10 − log10 5 1 − 𝑝
log10 3 = = = =
𝑞 𝑞 𝑞 𝑞
Hence:
𝑝 𝑝 𝑝𝑞
= =
log10 3 1 − 𝑝 1 − 𝑝
𝑞

Example 1.114: Expressions


To the nearest thousandth, log10 2 = .301 and log10 3 is .477. Which of the following is the best approximation of
log 5 10?
8
A. 7
9
B. 7
10
C.
7
11
D. 7
12
E. (AHSME 1979/18)
7

Method I
log 5 10 = log 5 5 + log 5 2 = 1 + log 5 2
Let
𝑥 = log10 2
Use the reciprocal rule:
1
= log 2 10 = 1 + log 2 5
𝑥
Solve for log 2 5:
1 1−𝑥
log 2 5 = −1 =
𝑥 𝑥
Use the reciprocal rule one more time:
𝑥 0.301 0.3 3
log 5 2 = = ≈ =
1 − 𝑥 1 − 0.301 0.7 7

3 10
log 5 10 = 1 + log 5 2 = 1 + =
7 7
Method II
log10 2 log10 2 0.301
log10 2 = = = = 0.301
log10 10 log10 2 + log10 5 0.301 + log10 5

1
=1
0.301 + log10 5
1 = 0.301 + log10 5
log10 5 = 0.699
1 1
=
log10 5 0.699
Using 0.699 ≈ 0.7:
1 1 10
log 5 10 = = =
0.7 7 7
10

P a g e 48 | 89
Get all the files at: https://bit.ly/azizhandouts
Aziz Manva (azizmanva@gmail.com)

Example 1.115: Expressions


Simplify
1 1 1 log 𝑛 𝑎 log 𝑛 𝑏 log 𝑛 𝑐
+ + , 𝑥= , 𝑦= , 𝑧=
1 1 1 log 𝑛 𝑏 + log 𝑛 𝑐 log 𝑛 𝑎 + log 𝑛 𝑐 log 𝑛 𝑎 + log 𝑛 𝑏
1+𝑥 1+𝑦 1+𝑧

Simplify 𝑥:
log 𝑛 𝑎 log 𝑛 𝑎
𝑥= = = log 𝑏𝑐 𝑎
log 𝑛 𝑏 + log 𝑛 𝑐 log 𝑛 𝑏𝑐
Take the reciprocal:
1
= log 𝑎 𝑏𝑐
𝑥
Add 1 to both sides:
1
1+ = log 𝑎 𝑎 + log 𝑎 𝑏𝑐 = log 𝑎 𝑎𝑏𝑐
𝑥
Take the reciprocal:
1
= log 𝑎𝑏𝑐 𝑎
1
1+𝑥
Similarly:
1 1
= log 𝑎𝑏𝑐 𝑏 , = log 𝑎𝑏𝑐 𝑐
1 1
1+𝑦 1+𝑧
Hence, the required expression
1 1 1
= + +
1 1 1
1+𝑥 1+𝑦 1+𝑧
= log 𝑎𝑏𝑐 𝑎 + log 𝑎𝑏𝑐 𝑏 + log 𝑎𝑏𝑐 𝑐
= log 𝑎𝑏𝑐 𝑎𝑏𝑐 = 1

Example 1.116
10 𝑥+𝑦
If 𝑥, 𝑦 > 0, log 𝑦 𝑥 + log 𝑥 𝑦 = 3
and 𝑥𝑦 = 144,then 2
= (AHSME 1990/23)

1
Note that log 𝑦 𝑥 = . Substitute 𝑡 = log 𝑦 𝑥
log𝑥 𝑦
1 1 log 𝑥
𝑡+ =3 ⇒𝑡=3⇒ =3
𝑡 3 log 𝑦
Cross-Multiply:
log 𝑥 = 3 log 𝑦 = log 𝑦 3
3
Exponentiate to get 𝑥 = 𝑦 and substitute in 𝑥𝑦 = 144 to get:
𝑦 4 = 144 ⇒ 𝑦 = 2√3 ⇒ 𝑥 = 𝑦 3 = 24√3
And finally, the answer we want is:
𝑥 + 𝑦 24√3 + 2√3
= = 13√3
2 2
B. Equations

Example 1.117: Equations


If log 𝑀 𝑁 = log 𝑁 𝑀 , 𝑀 ≠ 𝑁, 𝑀𝑁 > 0, 𝑀 ≠ 1, 𝑁 ≠ 1, then 𝑀𝑁 equals: (AHSME 1966/24)

P a g e 49 | 89
Get all the files at: https://bit.ly/azizhandouts
Aziz Manva (azizmanva@gmail.com)

Use the reciprocal rule on the LHS of the given equality:


1
= log 𝑁 𝑀
log 𝑁 𝑀
Cross-multiply:
1 = (log 𝑁 𝑀)2
Take square roots both sides
log 𝑁 𝑀 = ±1
Case I:
log 𝑁 𝑀 = 1 ⇒ 𝑁 = 𝑀 ⇒ 𝑁𝑜𝑡 𝑉𝑎𝑙𝑖𝑑
Case II:
1
log 𝑁 𝑀 = −1 ⇒ 𝑁 −1 = 𝑀 ⇒ = 𝑀 ⇒ 𝑀𝑁 = 1
𝑁
Example 1.118
1 1 5
Find all integer solutions to log8 𝑛
+ 1 = − 2 (SMT 2021 Algebra Tiebreaker/1)3
log𝑛
4

Use the reciprocal rule to make all terms have 𝑛 in the number:
1 5
+ log 1 𝑛 = −
log 8 𝑛 4 2
Rewrite:
1 5
+ log 2−2 𝑛 = −
log 23 𝑛 2
Use the power rule, and power rule extension:
3 log 2 𝑛 5
− =−
log 2 𝑛 2 2
Substitute 𝑥 = log 2 𝑛, converting it into a quadratic:
3 𝑥 5
− =−
𝑥 2 2
Multiply both sides by 2𝑥:
6 − 𝑥 2 = −5𝑥
𝑥 2 − 5𝑥 − 6 = 0
(𝑥 − 6)(𝑥 + 1) = 0
𝑥 ∈ {6, −1}

Change back to the original variable:


log 2 𝑛 = 6 ⇒ 𝑛 = 26 = 64
1
log 2 𝑛 = −1 ⇒ 𝑛 = 2−1 = ⇒ 𝑁𝑜𝑡 𝑉𝑎𝑙𝑖𝑑
2

C. Systems of Equations

Example 1.119: Equations


Let 𝑥, 𝑦 and 𝑧 all exceed 1 and let 𝑤 be a positive number such that log 𝑥 𝑤 = 24, log 𝑦 𝑤 = 40 and log 𝑥𝑦𝑧 𝑤 = 12.
Find log z 𝑤. (AIME 1983/1)

1
Note that the number in each term is the same. Hence, use log 𝑎 𝑥 = log :
𝑥𝑎

3
Here is a Video Solution

P a g e 50 | 89
Get all the files at: https://bit.ly/azizhandouts
Aziz Manva (azizmanva@gmail.com)

1
log 𝑥 𝑤 = 24 ⇒ log 𝑤 𝑥 =
⏟ 24
𝑬𝒒𝒖𝒂𝒕𝒊𝒐𝒏 𝑰
1
log 𝑦 𝑤 = 40 ⇒ log 𝑤 𝑦 =
⏟ 40
𝑬𝒒𝒖𝒂𝒕𝒊𝒐𝒏 𝑰𝑰
1
log 𝑥𝑦𝑧 𝑤 = 12 ⇒ log 𝑤 𝑥𝑦𝑧 =
⏟ 12
𝑬𝒒𝒖𝒂𝒕𝒊𝒐𝒏 𝑰𝑰𝑰
Subtract Equations I and II from Equation III:
1 1 1
log 𝑤 𝑥𝑦𝑧 − log 𝑤 𝑥 − log 𝑤 𝑦 = − −
12 24 40
𝑥𝑦𝑧 1 1
log 𝑤 = −
𝑥𝑦 24 40
1
log 𝑤 𝑧 =
60
1
Apply log 𝑎 𝑥 = log 𝑎:
𝑥
log 𝑧 𝑤 = 60

Example 1.120: Manipulating an Equality


Very often the questions using the reciprocal rule require you to find an expression (rather than the value of a
variable). This requires a good understanding of the way the reciprocal rule works.

If log 𝑎 𝑎𝑏 = 𝑥, then find log 𝑏 𝑎𝑏 in terms of 𝑥.

We want to change the base from 𝑎 to 𝑏. If we take the reciprocal, we will get an extra a in the base, which we don’t
want. Thankfully, there is a way to eliminate the 𝑎 from the number:
𝑥 = log 𝑎 𝑎𝑏 = log 𝑎 𝑎 + log 𝑎 𝑏 = 1 + log 𝑎 𝑏
Subtract 1 from both sides:
𝑥 − 1 = log 𝑎 𝑏
Now we are in a position to get the base that we want. Take the reciprocal
1
= log 𝑏 𝑎
𝑥−1
And now we can add back the 𝑏 in the number by adding 1 to both sides:
1
+ 1 = log 𝑏 𝑎 + 1 = log 𝑏 𝑎 + log 𝑏 𝑏 = log 𝑏 𝑎𝑏
𝑥−1

D. Tripartite Equality

Example 1.121: Tripartite Equality


𝑥−𝑦
Given that 5𝑥 = 0.5𝑦 = 1000, find the value of 𝑥𝑦
.

Simplify the required expression:


𝑥−𝑦 𝑥 𝑦 1 1
= − = −
𝑥𝑦 𝑥𝑦 𝑥𝑦 𝑦 𝑥
Logarithms
Separate out the given information into two different equations and solve each equation for the respective variable:
log 1000 1 log 5
5𝑥 = 1000 ⇒ 𝑥 log 5 = log 1000 ⇒ 𝑥 = ⇒ =
log 5 𝑥 log 1000

P a g e 51 | 89
Get all the files at: https://bit.ly/azizhandouts
Aziz Manva (azizmanva@gmail.com)

log 1000 1 log 0.5


(0.5)𝑦 = 1000 ⇒ 𝑦 log 0.5 = log 1000 ⇒ 𝑦 = ⇒ =
log 0.5 𝑦 log 1000

0.5 1
1 1 log 0.5 log 5 log log 10 log 10−1 (−1) log 10 1
− = − = 5 = = = =−
𝑦 𝑥 log 1000 log 1000 log 1000 log 1000 log 10 3 (3) log 10 3
Exponents
3
5𝑥 = 1000 ⇒ 5 = 10𝑥
3 3
0.5𝑦 = 1000 ⇒ 0.5 = 10𝑦 ⇒ 5 = 10𝑦 × 10
3
3 3 10𝑦 1 3 3 3 3 1 1 1 1 1

10𝑦 × 10 = 10𝑥 ⇒ 3 = ⇒ 10𝑦 𝑥 = 10−1 ⇒ − = −1 ⇒ 3 ( − ) = −1 ⇒ − = −
10 𝑦 𝑥 𝑦 𝑥 𝑦 𝑥 3
10𝑥

E. Reducible to Quadratic

Example 1.122
Let 𝑎, 𝑏, and 𝑥 be positive real numbers distinct from one. Then 4(log 𝑎 𝑥)2 + 3(log 𝑏 𝑥)2 = 8(log 𝑎 𝑥)(log 𝑏 𝑥) for
what values of 𝑎, 𝑏 and 𝑥. (AHSME 1976/20, Adapted)

Use the change of base rule:


log 𝑥 2 log 𝑥 2 log 𝑥 log 𝑥
4( ) + 3( ) = 8( )( )
log 𝑎 log 𝑏 log 𝑎 log 𝑏
Combine into one term on both sides:
4(log 𝑏)2 (log 𝑥)2 + 3(log 𝑎)2 (log 𝑥)2 (log 𝑥)2
= 8
(log 𝑎 log 𝑏)2 log 𝑎 log 𝑏
(log 𝑥)2
Divide both sides by log 𝑎 log 𝑏
:
4(log 𝑏)2 + 3(log 𝑎)2
=8
log 𝑎 log 𝑏
Split the fraction:
4 log 𝑏 3 log 𝑎
+ =8
log 𝑎 log 𝑏
This is a disguised quadratic since the second term has the reciprocal of the first.
log 𝑏
Use a change of variable and let 𝑦 = log 𝑎:
3 1 3
4𝑦 + = 8 ⇒ 4𝑦 2 − 8𝑦 + 3 = 0 ⇒ 𝑦 ∈ { , }
𝑦 2 2
Change back to the original variable:
log 𝑏 1
= ⇒ log 𝑏 2 = log 𝑎 ⇒ 𝑏 2 = 𝑎
log 𝑎 2
log 𝑏 3
= ⇒ log 𝑏 2 = log 𝑎3 ⇒ 𝑏 2 = 𝑎3
log 𝑎 2
It is already stated in the question that
𝑥, 𝑎, 𝑏 𝑎𝑟𝑒 𝑝𝑜𝑠𝑖𝑡𝑖𝑣𝑒 𝑟𝑒𝑎𝑙 𝑛𝑢𝑚𝑏𝑒𝑟𝑠 𝑔𝑟𝑒𝑎𝑡𝑒𝑟 𝑡ℎ𝑎𝑛 1
We do not need to apply any further restriction on 𝑥. For 𝑎, 𝑏 we need the restriction that:
𝑏 2 = 𝑎 𝑂𝑅 𝑏 2 = 𝑎3

Example 1.123
P a g e 52 | 89
Get all the files at: https://bit.ly/azizhandouts
Aziz Manva (azizmanva@gmail.com)

What is the product of all the solutions to the equation


log 7𝑥 2023 ∙ log 289𝑥 2023 = log 2023𝑥 2023 ?
(AMC 12A 2023/19)

Take the reciprocal both sides:


log 2023 7𝑥 ∙ log 2023 289𝑥 = log 2023 2023𝑥

Split the logarithms using log properties:


(log 2023 7 + log 2023 𝑥)(log 2023 289 + log 2023 𝑥) = 1 + log 2023 𝑥

Use a change of variable. Let log 2023 𝑥 = 𝑦, log 2023 7 = 𝑎, log 2023 289
(𝑎 + 𝑦)(𝑏 + 𝑦) = 1 + 𝑦
𝑎𝑏 + (𝑎 + 𝑏)𝑦 + 𝑦 2 = 1 + 𝑦

Substitute 𝑎 + 𝑏 = log 2023 7 + log 2023 289 = log 2023 2023 = 1:


𝑎𝑏 + 𝑦 + 𝑦 2 = 1 + 𝑦
𝑦 2 + 𝑎𝑏 − 1 = 0

Let the solutions of the equation be 𝛼 = log 2023 𝑥1 and 𝛽 = log 2023 𝑥2 . Use Vieta’s Formulas, the sum of the
solutions is zero:
log 2023 𝑥1 + log 2023 𝑥2 = 0
log 2023 𝑥1 𝑥2 = 0

Exponentiate both sides:


𝑥1 𝑥2 = 1

And this is the required answer

F. A Factorization Identity
The reciprocal rule for change of base can be combined with the perfect square identity (𝑎 + 𝑏)2 = 𝑎2 + 2𝑎𝑏 + 𝑏 2
in an interesting way to factor logs under a square root.

Example 1.124
1 1
A. Under what conditions is it true that √𝑥 + = √𝑥 + 𝑥 + 2
√𝑥
B. Use the result from Part A to show that √log 𝑏 𝑎 + log 𝑎 𝑏 + 2 = √log 𝑏 𝑎 + √log 𝑎 𝑏

Part A
Square the LHS using (𝑎 + 𝑏)2 = 𝑎2 + 2𝑎𝑏 + 𝑏 2 :
2
2 1 1
(√𝑥) + (2)(√𝑥) ( )+( )
√𝑥 √𝑥
Simplify to get:
1
=𝑥+ + 2 = 𝑅𝐻𝑆 2
𝑥

Hence, this is true for all 𝑥 in the domain of the square root function:
𝑥>0
Part B

P a g e 53 | 89
Get all the files at: https://bit.ly/azizhandouts
Aziz Manva (azizmanva@gmail.com)

Note that the first and the second term in the LHS are reciprocals of each other.
Rewrite the second term using the reciprocal rule for logs:
1
𝐿𝐻𝑆 = √log 𝑏 𝑎 + +2
log 𝑏 𝑎
Use a change of variable. Let 𝑥 = log 𝑏 𝑎, and use the identity from Part A:
1 1
= √𝑥 + + 2 = √𝑥 +
𝑥 √𝑥
Change back to the original variable:
1
√log 𝑏 𝑎 +
√log 𝑏 𝑎
Write the second term with a radical around the entire fraction:
1
= √log 𝑏 𝑎 + √
log 𝑏 𝑎
Again, use the reciprocal rule for logs:
= √log 𝑏 𝑎 + √log 𝑎 𝑏

Challenge 1.125
Write √log 2 6 + log 3 6 as two separate radicals. (AMC 12B 2020/13, Adapted)

Use the product rule for logs: log 𝑎 𝑥𝑦 = log 𝑎 𝑥 + log 𝑎 𝑦:


√log 2 2 + log 2 3 + log 3 2 + log 3 3
Substitute log 2 2 = log 3 3 = 1:
= √log 2 3 + log 3 2 + 2
Use the reciprocal rule on the second term:
1
√log 2 3 + +2
log 2 3
1 1
Use the identity √𝑥 + 𝑥 + 2 = √𝑥 + to get:
√𝑥
1
= √log 2 3 + = √log 2 3 + √log 3 2
√log 2 3

P a g e 54 | 89
Get all the files at: https://bit.ly/azizhandouts
Aziz Manva (azizmanva@gmail.com)

2. APPLICATIONS
2.1 Graphs
A. Base > 𝟏

2.1: Graph of 𝒚 = 𝐥𝐨𝐠 𝒂 𝒙


𝑎>1

Intercepts
To find the 𝑦-intercept, substitute 𝑥 = 0:
𝑦 = log 𝑥 = log 0 ⇒ 𝑁𝑜𝑡 𝐷𝑒𝑓𝑖𝑛𝑒𝑑
𝑁𝑜 𝑦 − 𝑖𝑛𝑡𝑒𝑟𝑐𝑒𝑝𝑡

To find the 𝑥-intercept, substitute 𝑦 = 0:


0 = log 𝑥 ⇒ 𝑥 = 1
𝑥 − 𝑖𝑛𝑡𝑒𝑟𝑐𝑒𝑝𝑡 = (1,0)
Asymptotes
𝑉𝑒𝑟𝑡𝑖𝑐𝑎𝑙 𝐴𝑠𝑦𝑚𝑝𝑡𝑜𝑡𝑒: 𝑥 = 0
𝐻𝑜𝑟𝑖𝑧𝑜𝑛𝑡𝑎𝑙 𝐴𝑠𝑦𝑚𝑝𝑡𝑜𝑡𝑒: 𝐷𝑁𝐸 (𝐷𝑜𝑒𝑠 𝑁𝑜𝑡 𝐸𝑥𝑖𝑠𝑡)
Positive and Negative Intervals
𝑦 > 0 ⇒ log 𝑎 𝑥 > 0 ⇒ 𝑥 > 1 ⇔ 𝐺𝑟𝑎𝑝ℎ 𝑖𝑠 𝑎𝑏𝑜𝑣𝑒 𝑡ℎ𝑒 𝑥 − 𝑎𝑥𝑖𝑠
𝑦 < 0 ⇒ log 𝑎 𝑥 < 0 ⇒ 0 < 𝑥 < 1 ⇔ 𝐺𝑟𝑎𝑝ℎ 𝑖𝑠 𝑏𝑒𝑙𝑜𝑤 𝑡ℎ𝑒 𝑥 − 𝑎𝑥𝑖𝑠
Increasing and Decreasing Intervals
The function is increasing throughout its domain:
(0, ∞)
Domain and Range
𝐷𝑜𝑚𝑎𝑖𝑛: 𝑥 > 0, 𝑎 > 0, 𝑎 ≠ 1
𝑅𝑎𝑛𝑔𝑒: (−∞, ∞)

Example 2.2: Property (Memorize)


Draw 𝑓(𝑥) = log 2 𝑥 and 𝑔(𝑥) = log 𝑎 𝑥 , 𝑎 > 1. Hence, for each function, determine when it is:
A. Greater than 1
B. Between 0 and 1
C. Negative

For 𝒇(𝒙)
log 2 𝑥 > 1 ⇔ 𝑥 > 2
0 < log 2 𝑥 < 1 ⇔ 1 < 𝑥 < 2
log 2 𝑥 < 0 ⇔ 0 < 𝑥 < 1

P a g e 55 | 89
Get all the files at: https://bit.ly/azizhandouts
Aziz Manva (azizmanva@gmail.com)

For 𝒈(𝒙)
log 𝑎 𝑛 > 1 ⇔ 𝑛 > 𝑎
0 < log 𝑎 𝑛 < 1 ⇔ 1 < 𝑛 < 𝑎
log 𝑎 𝑛 < 0 ⇔ 0 < 𝑥 < 1

Example 2.3: Inequalities


A. If 𝑥 and log10 𝑥 are real numbers and log10 𝑥 < 0, then must lie in the interval: (AHSME 1963/5)
B. Answer the previous question if log10 𝑥 ≤ 0.

Part A
Convert from logarithmic form to exponential form:
𝑥 < 100 ⇒ 𝑥 < 1
Also, the number to which we find the logarithm must be positive. Hence:
0 < 𝑥 < 1 ⇒ 𝑥 ∈ (0,1)
Part B
𝑥 ≤ 100 ⇒ 𝑥 ≤ 1
Also, the number to which we find the logarithm must be positive. Hence,
0 < 𝑥 ≤ 1 ⇒ 𝑥 ∈ (0,1]

Example 2.4
The graph of 𝑦 = log 𝑥
A. Cuts the 𝑦-axis
B. Cuts all lines perpendicular to the 𝑥-axis
C. Cuts the 𝑥-axis
D. Cuts neither axis
E. Cuts all circles whose center is at the origin (AHSME 1950/44)

Option B
The graph cuts all lines 𝑥 = 𝑐, 𝑐 > 0, but does not cut 𝑥 = 𝑐, 𝑐 ≤ 0.
For example, the graph does not cut:
𝑥 = −1
𝐻𝑒𝑛𝑐𝑒, 𝑜𝑝𝑡𝑖𝑜𝑛 𝐵 𝑖𝑠 𝑖𝑛𝑐𝑜𝑟𝑟𝑒𝑐𝑡.
Option C
The graph of 𝑦 = log 𝑥 has 𝑥 − 𝑖𝑛𝑡𝑒𝑟𝑐𝑒𝑝𝑡:
𝑥=1
𝑂𝑝𝑡𝑖𝑜𝑛 𝐶 𝑖𝑠 𝑐𝑜𝑟𝑟𝑒𝑐𝑡

Example 2.5

P a g e 56 | 89
Get all the files at: https://bit.ly/azizhandouts
Aziz Manva (azizmanva@gmail.com)

A. If 𝑥 is real and positive and grows beyond all bounds, then log 3 (6𝑥 − 5) − log 3(2𝑥 + 1) approaches:
(AHSME 1967/23)
B. What is the horizontal asymptote of the expression in the above question?

Part A
Combine using the quotient rule:
6𝑥 − 5
log 3 ( )
2𝑥 + 1
We want to understand the behavior as 𝑥 grows very large. Divide both numerator and denominator by the highest
power of 𝑥 in the denominator:
6𝑥 5 5
− 6−
log 3 ( 𝑥 𝑥 ) = log 3 ( 𝑥)
2𝑥 1 1
𝑥 +𝑥 2+𝑥
5 1
Note that as 𝑥 becomes very large, 𝑥 and 𝑥 both become very small:
6
≈ log 3 ( ) = log 3 (3) = 1
2
Part B
The horizontal asymptote is the 𝑦-value when the 𝑥-value becomes very large:
𝑦=1
B. 𝟎 < 𝑩𝒂𝒔𝒆 < 𝟏

2.6: Reflection across the 𝒙 − 𝒂𝒙𝒊𝒔


The graph of
𝑦 = log 1 𝑥 ⇔
⏟ 𝑦 = log 𝑎 𝑥
𝑎 𝑹𝒆𝒇𝒍𝒆𝒄𝒕𝒊𝒐𝒏
𝒂𝒄𝒓𝒐𝒔𝒔 𝒕𝒉𝒆 𝒙−𝒂𝒙𝒊𝒔

➢ Reflecting the graph of log 𝑎 𝑥 is equivalent to taking the


reciprocal of the base.
➢ Taking the reciprocal of the base 𝑎 in log 𝑎 𝑥 is equivalent to
reflecting the graph across the 𝑥-axis.

Example 2.7
A. The graph of 𝑔(𝑥) is obtained after a transformation of the
graph of 𝑓(𝑥) = log 3 𝑥. Determine 𝑔(𝑥).

𝑔(𝑥) = log 1 𝑥
3

2.8: Graph of 𝒚 = 𝐥𝐨𝐠 𝒂 𝒙


0<𝑎<1

Intercepts
To find the 𝑦-intercept, substitute 𝑥 = 0:
𝑦 = log 𝑥 = log 0 ⇒ 𝑁𝑜𝑡 𝐷𝑒𝑓𝑖𝑛𝑒𝑑
𝑁𝑜 𝑦 − 𝑖𝑛𝑡𝑒𝑟𝑐𝑒𝑝𝑡
To find the 𝑥-intercept, substitute 𝑦 = 0:
0 = log 𝑥 ⇒ 𝑥 = 1

P a g e 57 | 89
Get all the files at: https://bit.ly/azizhandouts
Aziz Manva (azizmanva@gmail.com)

𝑥 − 𝑖𝑛𝑡𝑒𝑟𝑐𝑒𝑝𝑡 = (1,0)
Asymptotes
𝑉𝑒𝑟𝑡𝑖𝑐𝑎𝑙 𝐴𝑠𝑦𝑚𝑝𝑡𝑜𝑡𝑒: 𝑥 = 0
𝐻𝑜𝑟𝑖𝑧𝑜𝑛𝑡𝑎𝑙 𝐴𝑠𝑦𝑚𝑝𝑡𝑜𝑡𝑒: 𝐷𝑁𝐸 (𝐷𝑜𝑒𝑠 𝑁𝑜𝑡 𝐸𝑥𝑖𝑠𝑡)
Positive and Negative Intervals
log 𝑎 𝑥 > 0 ⇒ 0 < 𝑥 < 1
log 𝑎 𝑥 < 0 ⇒ 𝑥 > 1
Increasing and Decreasing Intervals
The function is decreasing throughout its domain:
(0, ∞)
Domain and Range
𝐷𝑜𝑚𝑎𝑖𝑛: 𝑥 > 0, 𝑎 > 0, 𝑎 ≠ 1
𝑅𝑎𝑛𝑔𝑒: (−∞, ∞)

Example 2.9: Property (Memorize)


Draw 𝑓(𝑥) = log 1 𝑥 and 𝑔(𝑥) = log 𝑎 𝑥 , 0 < 𝑎 < 1. Hence, for each function, determine when it is:
2
A. Greater than 1
B. Between 0 and 1
C. Negative

𝒇(𝒙)
1
log 1 𝑥 > 1 ⇔ 0 < 𝑥 <
2 2
1
0 < log 1 𝑥 < 1 ⇔ <𝑥<1
2 2
log 1 𝑥 < 0 ⇔ 𝑥 > 1
2
𝒈(𝒙)
log 𝑎 𝑥 > 1 ⇔ 0 < 𝑥 < 𝑎
0 < log 𝑎 𝑥 < 1 ⇔ 𝑎 < 𝑥 < 1
log 𝑎 𝑥 < 0 ⇔ 𝑥 > 1

Example 2.10
A. Find all points on the graph of 𝑦 = log 5 𝑥 at a distance of 3 units from the 𝑥 −axis.
B. Without solving an equation, find all points on the graph of 𝑦 = log 1 𝑥 that are a distance of 3 units from the
5
𝑥-axis.
C. Without solving an equation, find all points on the graph of 𝑦 = log 5 𝑥 that are a distance of 5 units from
the 𝑥-axis.

Part A 𝑥 = 5±3

For horizontal/vertical distance, recall that the 𝑬𝒙𝒑𝒐𝒏𝒆𝒏𝒕𝒊𝒂𝒍 𝑭𝒐𝒓𝒎
absolute value of the difference of two points gives 1
(𝑥, 𝑦) = ( , −3) , (125,3)
the distance between the points: 125
𝐷𝑖𝑠𝑡𝑎𝑛𝑐𝑒 𝑓𝑟𝑜𝑚 𝑥 − 𝑎𝑥𝑖𝑠 = 3 ⇒ |𝑦 − 0| = 3 Part B
|𝑦| = 3 The graph of 𝑦 = log 1 𝑥 is obtained by reflecting the
5
|log 5 𝑥| = 3 graph of 𝑦 = log 5 𝑥 across the 𝑥-axis.
log 5 𝑥 = ±3
⏟ Hence, reflect the points from Part A across the x-axis
𝑳𝒐𝒈𝒂𝒓𝒊𝒕𝒉𝒎𝒊𝒄 𝑭𝒐𝒓𝒎 by negating their 𝑦-coordinate.

P a g e 58 | 89
Get all the files at: https://bit.ly/azizhandouts
Aziz Manva (azizmanva@gmail.com)

1 1 1
(𝑥, 𝑦) = ( , 3) , (125, −3) (𝑥, 𝑦) = ( × , −5) , (125 × 52 , 5)
125 125 25
Part C
1
(𝑥, 𝑦) = ( , −3) , (125,3)
125

Example 2.11
A. Find all points on the graph of 𝑦 = log 𝑏 𝑥 , 0 < 𝑏 < 1 at a distance of 𝑐, 𝑐 > 0 units from the 𝑥 −axis.
B. Answer the previous question if 𝑏 > 1.
C. Find all points on the graph of 𝑦 = log 𝑏 𝑥 , 𝑏 > 1 that are a distance of 𝑐 + 1, 𝑐 > 0 units from the 𝑥-axis.

Part A 𝑥 ∈ {𝑏 𝑐 , 𝑏 −𝑐 }
For horizontal/vertical distance, recall that the (𝑥, 𝑦) = (𝑏 𝑐 , −𝑐), (𝑏 −𝑐 , 𝑐)
absolute value of the difference of two points gives Part B
the distance between the points: (𝑥, 𝑦) = (𝑏 𝑐 , 𝑐), (𝑏 −𝑐 , −𝑐)
𝐷𝑖𝑠𝑡𝑎𝑛𝑐𝑒 𝑓𝑟𝑜𝑚 𝑥 − 𝑎𝑥𝑖𝑠 = 3 ⇒ |𝑦 − 0| = 𝑐 Part C
|𝑦| = 𝑐 𝑏 −𝑐
(𝑥, 𝑦) = (𝑏 𝑐 × 𝑏, 𝑐 + 1), ( , −𝑐 − 1)
|log 𝑏 𝑥| = 𝑐 𝑏
log 𝑏 𝑥 = ±𝑐 = (𝑏 𝑐+1 , 𝑐 + 1), (𝑏 −𝑐−1 , −𝑐 − 1)
𝑥 = 𝑏 ±𝑐

2.12: Translation: Vertical and Horizontal Shift


Compared to the graph of 𝑓(𝑥):
𝑓(𝑥) + 𝑘 ⇒ 𝑀𝑜𝑣𝑒𝑠 𝑢𝑝 𝑓𝑜𝑟 𝑘 > 0, 𝑀𝑜𝑣𝑒𝑠 𝑑𝑜𝑤𝑛 𝑓𝑜𝑟 𝑘 < 0
𝑓(𝑥 + 𝑘) ⇒ 𝑀𝑜𝑣𝑒𝑠 𝑙𝑒𝑓𝑡 𝑓𝑜𝑟 𝑘 > 0, 𝑀𝑜𝑣𝑒𝑠 𝑟𝑖𝑔ℎ𝑡 𝑓𝑜𝑟 𝑘 < 0

Example 2.13
State the transformation applied to the graph of 𝑦 = log 2 𝑥 in each case:
A. 𝑦 = log 2 (𝑥) + 2
B. 𝑦 = log 2 (𝑥) − 𝜋
C. 𝑦 = log 2 (𝑥) + log 3 𝑥
D. 𝑦 = log 2 (𝑥 + 3)
E. 𝑦 = log 2 (𝑥 − 2)

Let
𝑓(𝑥) = log 2 𝑥
Parts A-B
𝑦 = log 2 (𝑥) + 2 = 𝑓(𝑥) + 2 ⇒ 𝑀𝑜𝑣𝑒 𝑢𝑝 𝑏𝑦 2 𝑢𝑛𝑖𝑡𝑠
𝑦 = log 2 (𝑥) − 𝜋 ⇒ 𝑀𝑜𝑣𝑒𝑠 𝑑𝑜𝑤𝑛 𝑏𝑦 𝜋 𝑢𝑛𝑖𝑡𝑠
Part C
𝑦 = log 2 (𝑥) + log 3 𝑥 ⇒ 𝑀𝑜𝑣𝑒𝑠 𝑢𝑝 𝑏𝑦 log 3 𝑥

This is better written as:


𝑥 > 1 ⇒ 𝑀𝑜𝑣𝑒𝑠 𝑢𝑝 𝑏𝑦 log 3 𝑥
0 < 𝑥 < 1 ⇒ 𝑀𝑜𝑣𝑒𝑠 𝑑𝑜𝑤𝑛 𝑏𝑦 |log 3 𝑥|
Part D
𝑆ℎ𝑖𝑓𝑡𝑠 𝑙𝑒𝑓𝑡 𝑏𝑦 3 𝑢𝑛𝑖𝑡𝑠
Part E
𝑆ℎ𝑖𝑓𝑡𝑠 𝑟𝑖𝑔ℎ𝑡 𝑏𝑦 2 𝑢𝑛𝑖𝑡𝑠

P a g e 59 | 89
Get all the files at: https://bit.ly/azizhandouts
Aziz Manva (azizmanva@gmail.com)

2.14: Vertical and Horizontal Scaling


Compared to the graph of 𝑓(𝑥):
𝑘𝑓(𝑥) ⇒ 𝑆𝑐𝑎𝑙𝑒𝑠 𝑡ℎ𝑒 𝑔𝑟𝑎𝑝ℎ 𝑣𝑒𝑟𝑡𝑖𝑐𝑎𝑙𝑙𝑦 𝑏𝑦 𝑎 𝑓𝑎𝑐𝑡𝑜𝑟 𝑜𝑓 𝑘
1
𝑓(𝑘𝑥) ⇒ 𝑆𝑐𝑎𝑙𝑒𝑠 𝑡ℎ𝑒 𝑔𝑟𝑎𝑝ℎ ℎ𝑜𝑟𝑖𝑧𝑜𝑛𝑡𝑎𝑙𝑙𝑦 𝑏𝑦 𝑎 𝑓𝑎𝑐𝑡𝑜𝑟 𝑜𝑓
𝑘

Example 2.15
State the transformation applied to the graph of 𝑦 = log 2 𝑥 in each case:
A. 𝑦 = 2 log 2 𝑥
B. 𝑦 = log 2 2𝑥

𝑉𝑒𝑟𝑡𝑖𝑐𝑎𝑙 𝑆𝑐𝑎𝑙𝑖𝑛𝑔 𝑏𝑦 𝑎 𝐹𝑎𝑐𝑡𝑜𝑟 𝑜𝑓 2

𝐻𝑜𝑟𝑖𝑧𝑜𝑛𝑡𝑎𝑙 𝑆ℎ𝑟𝑖𝑛𝑘 𝑏𝑦 𝑎 𝑓𝑎𝑐𝑡𝑜𝑟 𝑜𝑓 2


log 2 2𝑥 = log 2 2 + log 2 𝑥 = 1 + log 2 𝑥 = 𝑓(𝑥) + 1 ⇒ 𝑀𝑜𝑣𝑒 𝑢𝑝 𝑏𝑦 1 𝑢𝑛𝑖𝑡

Example 2.16: Transformations


A. Combining Transformations

2.17: Log of 1 to any valid base is zero


𝑎0 = 1, 𝑎 > 0, 𝑎 ≠ 1 ⇔ log 𝑎 1 = 0

The condition that 𝑎 ≠ 0 is very important because:


00 ⇒ 𝑁𝑜𝑡 𝐷𝑒𝑓𝑖𝑛𝑒𝑑

P a g e 60 | 89
Get all the files at: https://bit.ly/azizhandouts
Aziz Manva (azizmanva@gmail.com)

Example 2.18
Determine the value of the variable in each of the following:
A. Find 𝑎 = log 5 1 , 𝑏 = log 𝜋 1 , 𝑐 = log 27.5 1 , 0 = log 6 𝑥 , 0 = log 𝑦 1
𝑎=𝑏=𝑐=0
𝑥=1
𝑦 > 0, 𝑦 ≠ 1

Example 2.19
If log 𝑎 𝑥 = 0, log 𝑎 (𝑦 + 2) = 0 and log 𝑎 𝑧 2 = 0, then find the minimum value and the maximum value of 𝑥 + 𝑦 + 𝑧.

log 𝑎 𝑥 = 0 ⇒ 𝑥 = 1
log 𝑎 (𝑦 + 2) = 0 ⇒ 𝑦 + 2 = 1 ⇒ 𝑦 = −1
log 𝑎 𝑧 2 = 0 ⇒ 𝑧 2 = 1 ⇒ 𝑧 = ±1

Max(𝑥 + 𝑦 + 𝑧) = 1 − 1 + 1 = 1
Min(𝑥 + 𝑦 + 𝑧) = (1 − 1 − 1) = −1

Example 2.20
If y =log_a x, a>1, which of the following statements is incorrect?
A. If x=1,y=0
B. If x=a,y=1
C. If x=-1,y is imaginary (complex)
D. If 0<x<1,y is always less than 0 and decreases without limit as x approaches zero
E. Only some of the above statements are correct (AHSME 1950/37)

Option E

2.21: Log of a number to itself, and to its reciprocal


𝑎1 = 𝑎 ⇔ 𝑙𝑜𝑔𝑎 𝑎 = 1
1
𝑎−1 = ⇔ log 1 𝑎 = −1
𝑎 𝑎

Example 2.22
Simplify:
A. log 5 5
B. log 317 317
3𝜋
C. log 3𝜋
𝑒 𝑒
D. log 1 5
5
1
E. log 5 5
3
F. log 4 4
3
3𝜋
G. log 𝑒 𝑒
3𝜋
1
H. log 3−√8 (3+ 8)

P a g e 61 | 89
Get all the files at: https://bit.ly/azizhandouts
Aziz Manva (azizmanva@gmail.com)

3𝜋
log 5 5 = log 317 317 = log 3𝜋 =1
𝑒 𝑒
1 3 3𝜋
log 1 5 = log 5 = log 4 = log 𝑒 = −1
5 5 34 3𝜋 𝑒
1 1 3 − √8 3 − √8
log 3−√8 ( ) = log 3−√8 ( × ) = log 3−√8 ( )=1
3 + √8 ⏟ 3 + √8 3 − √8 9−8
𝑹𝒂𝒕𝒊𝒐𝒏𝒂𝒍𝒊𝒛𝒊𝒏𝒈 𝒕𝒉𝒆 𝑫𝒆𝒏𝒐𝒎𝒊𝒏𝒂𝒕𝒐𝒓

2.2 Counting and Probability


A. Approximation
If a number lies between two powers of the base, then the log of that number must lie between those powers.

2.23: Log of a number to itself, and to its reciprocal


For some valid base 𝑏, and some valid number 𝑥:
𝑏𝑝 < 𝑥 < 𝑏𝑞 ⇔ ⏟
⏟ 𝑝 < log 𝑏 𝑥 < 𝑞
𝐸𝑥𝑝𝑜𝑛𝑒𝑛𝑡𝑖𝑎𝑙 𝐹𝑜𝑟𝑚 𝐿𝑜𝑔𝑎𝑟𝑖𝑡ℎ𝑚𝑖𝑐 𝐹𝑜𝑟𝑚

4<5<8
22 < 5 < 23
2 < log 2 5 < 3

Example 2.24: Approximating logs


Find, for each term, the two integers between which it lies:

Integer Bases D. log10 243 Fractional Bases


A. log 2 9 Radical Bases Transcendental Bases
B. log 5 63 E. log √3 12
C. log 3 29

Integer Bases
log 2 8 < log 2 9 < log 2 16 ⇒ 3 < log 2 9 < 4
2 < log 5 63 < 3
3 < log 3 29 < 4
2 < log10 243 < 3
Radical Bases
4 5
(√3) = 9, (√3) = 9√3 ≈ 15 ⇒ 4 < log √3 12 < 5

Example 2.25: Approximating logs


Find, for each term, the two integers between which it lies:
A. log 1 12
3
1
B. log 1 2
3

Part A
−3 < log 1 12 < −2
3
Part B

P a g e 62 | 89
Get all the files at: https://bit.ly/azizhandouts
Aziz Manva (azizmanva@gmail.com)

From the graph:


1
0 < log 1 <1
32

Example 2.26: Approximating logs


Find, for each term, the two integers between which it lies:
A. log 𝜋 2𝜋
B. log 𝑒 3𝑒 , 𝑒 ≈ 2.71

Part A
3 < 𝜋 < 4 ⇒ log 𝜋 𝜋 < log 𝜋 2𝜋 < log 𝜋 𝜋 2 ⇒ 1 < log 𝜋 2𝜋 < 2

Part B
Since 𝑒 2 < 3𝑒 and 3𝑒 < 𝑒 3 :
log 𝑒 𝑒 2 < log 𝑒 3𝑒 < log 𝑒 𝑒 3
Hence:
2 < log 𝑒 3𝑒 < log 𝑒 𝑒 3

Example 2.27: Applying approximation to get a range


Find the sum of the values that satisfy 2 < log 𝜋 𝑛𝜋 < 3, 𝑛 ∈ ℕ

Solution I (Transforming Inequalities)


The given inequality has a log with the base 𝜋, but the other parts of the inequality have numbers.
Hence, convert the numbers into logs 𝜋:
log 𝜋 𝜋 2 < log 𝜋 𝑛𝜋 < log 𝜋 𝜋 3
Since all three parts have logs, we can “cancel” the logs:
(In other words, take anti-logs throughout the inequalities)
𝜋 2 < 𝑛𝜋 < 𝜋 3
Divide throughout by 𝜋:
𝜋 < 𝑛 < 𝜋2
Convert to numerical form
3.14 < 𝑛 < 9.86
Hence, the values that 𝑛 can take are:
𝑛 ∈ {4,5, 6, 7, 8, 9}
𝑆𝑢𝑚 = 4 + 5 + 6 + 7 + 8 + 9 = 39

Solution II (Working with endpoints of the inequality)


An alternate method of solving the inequality is to solve the corresponding equations:
log 𝜋 𝑛𝜋 = 2 ⇒ 𝑛 = 𝜋 ≈ 3.14
log 𝜋 𝑛𝜋 = 3 ⇒ 𝑛 = 𝜋 2 ≈ 9.86
∴ 3.14 < 𝑛 < 9.86 ⇒ 𝑛 ∈ {4,5, 6, 7, 8, 9}
B. Counting / Number Theory
We look at some ways in which logarithms can be combined with other concepts.

Example 2.28

P a g e 63 | 89
Get all the files at: https://bit.ly/azizhandouts
Aziz Manva (azizmanva@gmail.com)

Natural Numbers
Find the natural numbers that lie between:
A. log 2 3 and log 2 2048
1
B. log 3 81 and log 3 2087
Integers
Find the number of integers that lie between:
C. log10 0.005 and log10 1,000,000,000

{log 2 4 , … , log 2 1024} ⇒ {2, 3, … ,10} ⇒ 10 − 2 + 1 = 9


1
{log 3 , … , log 3 729} ⇒ 𝑛(−3, −2, −1,0,1,2, … ,6) = 𝑛(1, 2, … ,6) = 6
27
{log10 0.01, … , log10 100,000,000} = {−2, −1, … ,8} ⇒ 8 − (−2) + 1 = 11

Example 2.29: Natural Number Solutions


What is the sum of the values of 𝑥 that satisfy:
A. log 2 𝑥 = 𝑐, 𝑐 ∈ ℕ, 𝑥 < 1000
B. log 2 𝑥 = 𝑐, 𝑐 ∈ ℕ, 𝑐 < 1000

Part A
log 2 2 = 1
log 2 4 = log 2 22 = 2
log 2 8 = log 2 23 = 3
log 2 16 = log 2 24 = 4
.
.
.
log 2 512 = log 2 29 = 9
Hence, the numbers which work are:
{2,4,8,16, … ,512} = {21 , 22 , 23 , 24 , … , 29 }
The sum of the solutions is:
21 + 22 + ⋯ + 29 = ⏟
𝟐𝟎 + 21 + 22 + ⋯ + 29 − 𝟐𝟎 = 1022
𝟏𝟎𝟐𝟒−𝟏
Where we used the fact that:
1 + 2 + 22 + ⋯ 2𝑛 = 2𝑛+1 − 1
The above property hold it is a geometric series with 𝑎 = 1, 𝑟 = 2:
𝑎(𝑟 𝑛 − 1) 1(2𝑛+1 − 1)
𝑆= = = 2𝑛+1 − 1
𝑟−1 2−1
Part B
{21 , 22 , 23 , 24 , … , 2999 }
21 + 22 + ⋯ + 2999 = ⏟ 𝟐𝟎 + 21 + 22 + ⋯ + 2999 − 𝟐𝟎 = 21000 − 2
𝟐𝟏𝟎𝟎𝟎 −𝟏

Example 2.30: Integer Solutions


What is the sum of the values of 𝑥 that satisfy:
A. log 3 𝑥 = 𝑐, 𝑐 ∈ ℤ, 𝑥 < 1000
B. log 3 𝑥 = 𝑐, 𝑐 ∈ ℤ, |𝑐| < 1000

P a g e 64 | 89
Get all the files at: https://bit.ly/azizhandouts
Aziz Manva (azizmanva@gmail.com)

Part A
log 3 729 = 6
log 3 3 = 1
log 3 1 = 0
1
log 3 = −1
3
1
log 3 = −2
9
.
.
.
1
log 3 𝑛 = −𝑛
3

729 + 243 + 81 + ⋯
36 + 35 + 34 + ⋯

1
This is an infinite geometric series with 𝑎 = 36 , 𝑟 = 3 which has sum:
𝑎 36 36 729 × 3
𝑆= = = = = 1093.5
1−𝑟 1−1 2 2
3 3
Part B
1
3999 + 3998 + ⋯ +
3999
1
This is a finite geometric series with 𝑎 = 3999 , 𝑟 = 3 , 𝑛 = 1999:

1 1999
3999 [1 − (3)
𝑛) ]
𝑎(1 − 𝑟
𝑆= =
1−𝑟 1
1−3
3999 − 3999−1999
2
3
3999 − 3999−1999
=
2
3
3(3999 − 3−1000 )
=
2
1000 1
3 − 999
= 3
2

Example 2.31: Number of Solutions for the Base


Find the number of values 𝑏 that satisfy each equation below (answer separately for each):
A. log 𝑏 32 = 𝑛, 𝑛 ∈ ℕ
B. log 𝑏 1024 = 𝑛, 𝑛 ∈ ℕ
C. log 𝑏 248 = 𝑛, 𝑛 ∈ ℕ

Part A
Since 𝑛 is a natural number, 𝑏 must also be a natural number.

P a g e 65 | 89
Get all the files at: https://bit.ly/azizhandouts
Aziz Manva (azizmanva@gmail.com)

log 𝑏 32 = 𝑛
𝑏 𝑛 = 32 = 25
(𝑏, 𝑛) = {(2,5), (32,1)}
Part B
log 𝑏 1024 = 𝑛
𝑏 𝑛 = 1024 = 210
(21 )10 = (210 )1 = (22 )5 = (25 )2

(𝑏, 𝑛) = {(2, 𝟏𝟎)(4, 𝟓), (32, 𝟐)(1024, 𝟏)}

𝐹𝑎𝑐𝑡𝑜𝑟𝑠 𝑜𝑓 10 = {1,2,5,10}
Part C
log 𝑏 248 = 𝑛
𝑏 𝑛 = 248
The number of ordered pairs (𝑏, 𝑛) that will work is the same as the number of factors of 48:
(1,48)(2,24)(3,16)(4,12)(6,8) ⇒ 10 𝐹𝑎𝑐𝑡𝑜𝑟𝑠
10 𝑆𝑜𝑙𝑢𝑡𝑖𝑜𝑛𝑠

(21 )48 , (22 )24 , (23 )16 , (24 )12 , (26 )8


(248 )1 , (224 )2 , (216 )3 , (212 )4 , (28 )6

Example 2.32
Find the number of elements in the set
{(𝑎, 𝑏) ∈ ℕ: 2 ≤ 𝑎, 𝑏 ≤ 2023, log 𝑎 𝑏 + 6 log 𝑏 𝑎 = 5} (𝑰𝑶𝑸𝑴 𝟐𝟎𝟐𝟑/𝟐)

Using the reciprocal rule:


6
log 𝑎 𝑏 + =5
log 𝑎 𝑏

Use a change of variable. Let 𝑡 = log 𝑎 𝑏


6
𝑡+ = 5 ⇒ 𝑡 2 − 5𝑡 + 6 = 0 ⇒ 𝑡 ∈ {2,3}
𝑡

Case I: 𝐥𝐨𝐠 𝒂 𝒃 = 𝟐 ⇒ 𝒃 = 𝒂𝟐
(𝑎, 𝑏) = (2,4), (3,9), (4,16), … , (44,1936) ⇒ 43 𝑃𝑎𝑖𝑟𝑠
𝟑
Case II: 𝐥𝐨𝐠 𝒂 𝒃 = 𝟑 ⇒ 𝒃 = 𝒂
(𝑎, 𝑏) = (2,8), (3,27), … , (12,1728) ⇒ 11 𝑃𝑎𝑖𝑟𝑠

Total number of pairs


= 43 + 11 = 54

2.33: Number of Divisors4


The number of divisors 𝝉(𝒙) of a number 𝒙 with prime factorisation 𝒂𝒑 𝒃𝒒 𝒄𝒓 is given by:
𝒙 = 𝒂𝒑 𝒃𝒒 𝒄𝒓 ⇒ 𝝉(𝒙) = (𝒑 + 𝟏)(𝒒 + 𝟏)(𝒓 + 𝟏)

Example 2.34: Number of Solutions for the Base

4
The details of this formula can be found in the Note on NT-Basics

P a g e 66 | 89
Get all the files at: https://bit.ly/azizhandouts
Aziz Manva (azizmanva@gmail.com)

Find the number of values 𝑏 that satisfy each equation below (answer separately for each):
A. log 𝑏 5216 = 𝑛, 𝑛 ∈ ℕ
B. log 𝑏 75! = 𝑛, 𝑛 ∈ ℕ

Part A
216 = 63 = 23 × 33 ⇒ 𝜏(216) = (3 + 1)(3 + 1) = 16 ⇒ 16 𝑆𝑜𝑙𝑢𝑡𝑖𝑜𝑛𝑠
Part A

5! = 120 = 23 × 3 × 5 ⇒ 𝜏(5!) = (3 + 1)(1 + 1)(1 + 1) = 16 𝑆𝑜𝑙𝑢𝑡𝑖𝑜𝑛𝑠

2.35: Product of Divisors


The product of the distinct positive factors of a number N is given by:
𝜏(𝑁)
( )
𝑁 2

Example 2.36
Let 𝑆 be the sum of the base 10 logarithms of the proper divisors of 1,000,000. What is the integer nearest to 𝑆?
(AIME 1986/8)

If the divisors are 𝑑1 , 𝑑2 , … 𝑑𝑘 , then using the product rule for logarithms:
log 𝑑1 + log 𝑑2 + ⋯ + log 𝑑𝑘 = log 𝑑1 × 𝑑2 × … × 𝑑𝑘

The above is now the product of the divisors of 1,000,000, for which we have a formula:
𝜏(𝑁) 49
( )
𝑑1 × 𝑑2 × … × 𝑑𝑘 = (𝑁) 2 = (106 ) 2 = 10147
where:
𝑁 = 1,000,000 = 106 = 26 × 56
𝜏(𝑁) = number of divisors of 𝑁 = (6 + 1)(6 + 1) = 49

However, since we only need the proper divisors, we must exclude 1,000,000 from the calculations, and hence, we
get:
10147
= log = log 10141 = 141
106

C. Probability

Example 2.37
A. Li selects an integer 𝑥 from −1000 to 1000. What is the probability that log 3 𝑥 is defined?
B. Li selects an integer 𝑥 between −1000 and 1000. What is the probability that log 3 𝑥 is defined?

For log 3 𝑥 to be defined:


𝑥 𝑖𝑠 𝑎 𝑟𝑒𝑎𝑙 𝑛𝑢𝑚𝑏𝑒𝑟 𝑠𝑢𝑐ℎ 𝑡ℎ𝑎𝑡 𝑥 > 0
Part A
The integers that meet this condition are:
{1,2,3, … ,1000} ⇒ 1000 𝐼𝑛𝑡𝑒𝑔𝑒𝑟𝑠
The total number of integers to pick from is:
1000 − (−1000) + 1 = 2001
Hence, the probability is:

P a g e 67 | 89
Get all the files at: https://bit.ly/azizhandouts
Aziz Manva (azizmanva@gmail.com)

𝑆𝑢𝑐𝑐𝑒𝑠𝑠𝑓𝑢𝑙 𝑂𝑢𝑡𝑐𝑜𝑚𝑒𝑠 1000


=
𝑇𝑜𝑡𝑎𝑙 𝑂𝑢𝑡𝑐𝑜𝑚𝑒𝑠 2001
Part B
The integers that meet this condition are:
{1,2,3, … ,999} ⇒ 999 𝐼𝑛𝑡𝑒𝑔𝑒𝑟𝑠
The total number of integers to pick from is:
999 − (−999) + 1 = 1999
Hence, the probability is:
𝑆𝑢𝑐𝑐𝑒𝑠𝑠𝑓𝑢𝑙 𝑂𝑢𝑡𝑐𝑜𝑚𝑒𝑠 999
=
𝑇𝑜𝑡𝑎𝑙 𝑂𝑢𝑡𝑐𝑜𝑚𝑒𝑠 1999

Example 2.38
I select a number from 1 to 1000. What is the probability that the log of the number to the base 5 is an integer?

You can select any real number from the interval


(1,1000) ⇒ 𝐼𝑛𝑓𝑖𝑛𝑖𝑡𝑒 𝑁𝑢𝑚𝑏𝑒𝑟𝑠

The number of numbers such that the log of the number to the base 5 is an integer will be a finite number.
𝑆𝑢𝑐𝑐𝑒𝑠𝑠𝑓𝑢𝑙 𝑂𝑢𝑡𝑐𝑜𝑚𝑒𝑠 𝐹𝑖𝑛𝑖𝑡𝑒 𝑁𝑢𝑚𝑏𝑒𝑟
𝑃= = =0
𝑇𝑜𝑡𝑎𝑙 𝑂𝑢𝑡𝑐𝑜𝑚𝑒𝑠 ∞

Example 2.39
I select a natural number from 1 to 1000. What is the probability that the log of the number to the base 5:
A. is an integer
B. is a natural number
C. is an even integer
D. is an odd integer
E. is a whole number

Part A log 5 𝑥 = 𝑐, 𝑐 𝑖𝑠 𝑎𝑛 𝑒𝑣𝑒𝑛 𝑖𝑛𝑡𝑒𝑔𝑒𝑟 ⇒ 𝑥 ∈ {1,25,625}


log 5 𝑥 = 𝑐, 𝑐 ∈ ℤ ⇒ 𝑥 ∈ {1,5,25,125,625} 𝑆𝑢𝑐𝑐𝑒𝑠𝑠𝑓𝑢𝑙 𝑂𝑢𝑡𝑐𝑜𝑚𝑒𝑠 3
𝑆𝑢𝑐𝑐𝑒𝑠𝑠𝑓𝑢𝑙 𝑂𝑢𝑡𝑐𝑜𝑚𝑒𝑠 5 1 𝑃= =
𝑇𝑜𝑡𝑎𝑙 𝑂𝑢𝑡𝑐𝑜𝑚𝑒𝑠 1000
𝑃= = = Part D
𝑇𝑜𝑡𝑎𝑙 𝑂𝑢𝑡𝑐𝑜𝑚𝑒𝑠 1000 200
Part B log 5 𝑥 = 𝑐, 𝑐 𝑖𝑠 𝑎𝑛 𝑜𝑑𝑑 𝑖𝑛𝑡𝑒𝑔𝑒𝑟 ⇒ 𝑥 ∈ {5,125}
log 5 𝑥 = 𝑐, 𝑐 ∈ ℕ ⇒ 𝑥 ∈ {5,25,125,625} 𝑆𝑢𝑐𝑐𝑒𝑠𝑠𝑓𝑢𝑙 𝑂𝑢𝑡𝑐𝑜𝑚𝑒𝑠 2 1
𝑆𝑢𝑐𝑐𝑒𝑠𝑠𝑓𝑢𝑙 𝑂𝑢𝑡𝑐𝑜𝑚𝑒𝑠 4 1 𝑃= = =
𝑇𝑜𝑡𝑎𝑙 𝑂𝑢𝑡𝑐𝑜𝑚𝑒𝑠 1000 500
𝑃= = = Part E
𝑇𝑜𝑡𝑎𝑙 𝑂𝑢𝑡𝑐𝑜𝑚𝑒𝑠 1000 250
Part C Same as part A.

Example 2.40
I select a natural number from 1 to 10,000. What is the probability that the log of the number to the base 2:
A. is an integer
B. is a natural number
C. is an even integer
D. is an odd integer

Part A 𝑆𝑢𝑐𝑐𝑒𝑠𝑠𝑓𝑢𝑙 𝑂𝑢𝑡𝑐𝑜𝑚𝑒𝑠 14 7


𝑃= = =
log 5 𝑥 = 𝑐, 𝑐 ∈ ℤ ⇒ 𝑥 ∈ {20 1 13 }
,2 ,…,2 𝑇𝑜𝑡𝑎𝑙 𝑂𝑢𝑡𝑐𝑜𝑚𝑒𝑠 10,000 5000

P a g e 68 | 89
Get all the files at: https://bit.ly/azizhandouts
Aziz Manva (azizmanva@gmail.com)

Part B 𝑆𝑢𝑐𝑐𝑒𝑠𝑠𝑓𝑢𝑙 𝑂𝑢𝑡𝑐𝑜𝑚𝑒𝑠 7


𝑃= =
log 5 𝑥 = 𝑐, 𝑐 ∈ ℤ ⇒ 𝑥 ∈ {21
,…,2 13 }
𝑇𝑜𝑡𝑎𝑙 𝑂𝑢𝑡𝑐𝑜𝑚𝑒𝑠 10,000
𝑆𝑢𝑐𝑐𝑒𝑠𝑠𝑓𝑢𝑙 𝑂𝑢𝑡𝑐𝑜𝑚𝑒𝑠 13 Part D
𝑃= =
𝑇𝑜𝑡𝑎𝑙 𝑂𝑢𝑡𝑐𝑜𝑚𝑒𝑠 10,000 Out of 14 integers, 7 are even, so the remaining 7 are
Part C odd.
log 5 𝑥 = 𝑐, 𝑐 𝑖𝑠 𝑎𝑛 𝑒𝑣𝑒𝑛 𝑖𝑛𝑡𝑒𝑔𝑒𝑟 ⇒ 𝑥 𝑆𝑢𝑐𝑐𝑒𝑠𝑠𝑓𝑢𝑙 𝑂𝑢𝑡𝑐𝑜𝑚𝑒𝑠 7
𝑃= =
∈ {20 , 21 , … , 212 } 𝑇𝑜𝑡𝑎𝑙 𝑂𝑢𝑡𝑐𝑜𝑚𝑒𝑠 10,000

Example 2.41
I select a natural number 𝑛 at random from 1 to 1000. I calculate 𝑥 = log 2 𝑛 and 𝑦 = log 3 𝑛. What is the probability
that:
A. 𝑥 is an integer
B. 𝑦 is an integer
C. Both 𝑥 and 𝑦 are integers
D. At least one of 𝑥 and 𝑦 is an integer
E. Exactly one of 𝑥 and 𝑦 is an integer.

Part A 1
𝑃=
log 2 𝑛 = 𝑥, 𝑥 ∈ ℤ 1000
𝑛= {20 , 21 , … , 29 } ⇒ 10 𝑁𝑢𝑚𝑏𝑒𝑟𝑠
10 1 Part D (Union of Two Sets)
𝑃= = {1,2,4,8,16,32,64,128,256,512, 𝟏, 3,9,27,81,243,729}
1000 100
Part B {1,2,4,8,16,32,64,128,256,512, 3,9,27,81,243,729}
log 3 𝑛 = 𝑦, 𝑦 ∈ ℤ
𝑛= {3 0 1
, 3 , … , 36 } ⇒ 7 𝑁𝑢𝑚𝑏𝑒𝑟𝑠 10 7 1 16 2
+ − = =
7 ⏟
1000 ⏟
1000 ⏟
1000 1000 125
𝑃= 𝑉𝑎𝑙𝑢𝑒𝑠 𝑉𝑎𝑙𝑢𝑒𝑠 𝑉𝑎𝑙𝑢𝑒𝑠 𝑜𝑓
1000 𝑜𝑓 𝑥 𝑜𝑓 𝑦 𝑥∩𝑦
Part C (Intersection of two sets)
Part E (Exclusive OR)
We have already identified the values of 𝑛 for which 𝑥
{𝟏, 2,4,8,16,32,64,128,256,512, 𝟏, 3,9,27,81,243,729}
and 𝑦 are integers.
𝑥 ∈ ℤ ⇒ 𝑛 = {1,2,4,8,16,32,64,128,256,512} {2,4,8,16,32,64,128,256,512
⏟ , 3,9,27,81,243,729
⏟ }
𝑦 ∈ ℤ ⇒ 𝑛 = {1, 3,9,27,81,243,729} 𝑥 𝑖𝑠 𝑎𝑛 𝑖𝑛𝑡𝑒𝑔𝑒𝑟 𝑦 𝑖𝑠 𝑎𝑛 𝑖𝑛𝑡𝑒𝑔𝑒𝑟
For both the numbers to be an integer, it must belong
to both of the sets above. Hence, we want the 10 7 1 15 3
+ −2 = =
intersection of the two sets, which is: ⏟
1000 ⏟
1000 ⏟
1000 1000 200
{1} 𝑉𝑎𝑙𝑢𝑒𝑠 𝑉𝑎𝑙𝑢𝑒𝑠 𝑉𝑎𝑙𝑢𝑒𝑠 𝑜𝑓
𝑜𝑓 𝑥 𝑜𝑓 𝑦 𝑥∩𝑦

Example 2.42
I select a natural number 𝑛 at random from 1 to 1000. I toss a fair coin. If I get heads, I calculate 𝑥 = log 2 𝑛. If the
coins comes up tails, I calculate 𝑥 = log 3 𝑛. What is the probability that 𝑥 is an integer?

P a g e 69 | 89
Get all the files at: https://bit.ly/azizhandouts
Aziz Manva (azizmanva@gmail.com)

The probabilities when you toss the coin are:


1 1
𝐻𝑒𝑎𝑑𝑠 = , 𝑇𝑎𝑖𝑙𝑠 =
2 2
log 2 𝑛 = 𝑐, 𝑐 ∈ ℤ ⇒ 𝑛 = {20 , 21 , … , 29 } ⇒ 10 𝑁𝑢𝑚𝑏𝑒𝑟𝑠
log 3 𝑛 = 𝑐, 𝑐 ∈ ℤ ⇒ 𝑛 = {30 , 31 , … , 36 } ⇒ 7 𝑁𝑢𝑚𝑏𝑒𝑟𝑠

1 10 1 7 1 10 7 17
× + × = ( + )=
2 1000 2 1000 2 1000 1000 2000

Example 2.43
From a standard pack of playing cards, I discard the face cards and the Ace. I pick a card at random from the
remaining cards and note its face value 𝑓. I then choose a natural number 𝑛 at random from 1 to 1000. What is the
probability that log 𝑓 𝑛 is a whole number?

I will pick a single card with face value


𝑓 = {2,3, … ,10}

𝑓 = 2 ⇒ log 2 𝑛 = 𝑐, 𝑐 ∈ ℤ ⇒ 𝑛 = {20 , 21 , … , 29 } ⇒ 10 𝑁𝑢𝑚𝑏𝑒𝑟𝑠


𝑓 = 3 ⇒ log 3 𝑛 = 𝑐, 𝑐 ∈ ℤ ⇒ 𝑛 = {30 , 31 , … , 36 } ⇒ 7 𝑁𝑢𝑚𝑏𝑒𝑟𝑠
𝑓 = 4 ⇒ log 4 𝑛 = 𝑐, 𝑐 ∈ ℤ ⇒ 𝑛 = {40 , … , 44 } ⇒ 5 𝑁𝑢𝑚𝑏𝑒𝑟𝑠
𝑓 = 5 ⇒ log 5 𝑛 = 𝑐, 𝑐 ∈ ℤ ⇒ 𝑛 = {50 , … , 54 } ⇒ 5 𝑁𝑢𝑚𝑏𝑒𝑟𝑠
𝑓 = 6 ⇒ log 6 𝑛 = 𝑐, 𝑐 ∈ ℤ ⇒ 𝑛 = {60 , … , 63 } ⇒ 4 𝑁𝑢𝑚𝑏𝑒𝑟𝑠
𝑓 = 7 ⇒ log 7 𝑛 = 𝑐, 𝑐 ∈ ℤ ⇒ 𝑛 = {70 , … , 73 } ⇒ 4 𝑁𝑢𝑚𝑏𝑒𝑟𝑠
𝑓 = 8 ⇒ log 8 𝑛 = 𝑐, 𝑐 ∈ ℤ ⇒ 𝑛 = {80 , … , 83 } ⇒ 4 𝑁𝑢𝑚𝑏𝑒𝑟𝑠
𝑓 = 9 ⇒ log 9 𝑛 = 𝑐, 𝑐 ∈ ℤ ⇒ 𝑛 = {90 , … , 93 } ⇒ 4 𝑁𝑢𝑚𝑏𝑒𝑟𝑠
𝑓 = 10 ⇒ log 9 𝑛 = 𝑐, 𝑐 ∈ ℤ ⇒ 𝑛 = {100 , … , 103 } ⇒ 4 𝑁𝑢𝑚𝑏𝑒𝑟𝑠

1 10 + 7 + 5(2) + 4(5) 1 47 47
( )= ( )=
9 1000 9 1000 9000

Challenge 2.44
𝑁 = {log 2 𝑛 , log 3 𝑛 , … }
I pick a random natural number, 𝑛, from 1 to 1000, what is the probability that
A. at least one element of 𝑁 is a natural number
B. at least one element of 𝑁 is a natural number greater than 1.

Part A
log 𝑛 𝑛 = 1 ⇒ 𝑃 = 1
Part B
𝑛 = {4,8,16,32,64,128,256,512} = {22 , … , 29 } ⇒ 8
𝑛 = {32 , … , 36 } ⇒ 5 𝑁𝑢𝑚𝑏𝑒𝑟𝑠
𝑛 𝑖𝑠 4𝑥 ⇒ 0 𝑁𝑢𝑚𝑏𝑒𝑟𝑠
𝑛 𝑖𝑠 5𝑥 ⇒ {25,125,625} ⇒ 3 𝑁𝑢𝑚𝑏𝑒𝑟𝑠
𝑛 𝑖𝑠 6𝑥 ⇒ {36,216} ⇒ 2 𝑁𝑢𝑚𝑏𝑒𝑟𝑠
𝑛 𝑖𝑠 7𝑥 ⇒ {49,343} ⇒ 2 𝑁𝑢𝑚𝑏𝑒𝑟𝑠
𝑛 𝑖𝑠 8𝑥 ⇒ 0 𝑁𝑢𝑚𝑏𝑒𝑟𝑠
𝑛 𝑖𝑠 9𝑥 ⇒ 0 𝑁𝑢𝑚𝑏𝑒𝑟𝑠
𝑛 𝑖𝑠 10𝑥 ⇒ 2 𝑁𝑢𝑚𝑏𝑒𝑟𝑠

P a g e 70 | 89
Get all the files at: https://bit.ly/azizhandouts
Aziz Manva (azizmanva@gmail.com)

{11𝑥 , 12𝑥 , 13𝑥 , … , 312 } ⇒ 31 − 10 = 21 𝑁𝑢𝑚𝑏𝑒𝑟𝑠


{16𝑥 , 25𝑥 , 27𝑥 } ⇒ 3 𝑁𝑢𝑚𝑏𝑒𝑟𝑠
21 − 3 = 18 𝑁𝑢𝑚𝑏𝑒𝑟𝑠

8 + 5 + 3 + 2 + 2 + 2 + 18 = 40

Example 2.45
1 1
𝑥 is a random number that satisfies 16 < log 𝑥 5 < 2
A. What is the range of values that 𝑥 can take?
B. What is the probability that 𝑥 = 625, given that 𝑥 is a real number?
C. What is the probability that 𝑥 = 625, given that 𝑥 is a natural number?
1 1
D. Answer Part C assuming that other things remain unchanged and 16
≤ log 𝑥 5 ≤ 2

Part A 𝑇𝑜𝑡𝑎𝑙 𝑂𝑢𝑡𝑐𝑜𝑚𝑒𝑠 = ∞


Convert the inequality given in the question into 𝑃𝑟𝑜𝑏𝑎𝑏𝑖𝑙𝑖𝑡𝑦 = 0
exponential form: Part C
1 1 𝑆𝑢𝑐𝑐𝑒𝑠𝑠𝑓𝑢𝑙 𝑂𝑢𝑡𝑐𝑜𝑚𝑒𝑠 = 1
𝑥 16
<5< 𝑥2
𝑇𝑜𝑡𝑎𝑙 𝑂𝑢𝑡𝑐𝑜𝑚𝑒𝑠 = (516 − 1) − (52 ) = 516 − 26
Split the compound inequality into two inequalities:
1 1
𝑥 16 < 5 ⇒ 𝑥 < 516 𝑃𝑟𝑜𝑏𝑎𝑏𝑖𝑙𝑖𝑡𝑦 = 16
5 − 26
1 1 Part D
5 < 𝑥 2 ⇒ 𝑥 2 > 5 ⇒ 𝑥 > 52
𝑆𝑢𝑐𝑐𝑒𝑠𝑠𝑓𝑢𝑙 𝑂𝑢𝑡𝑐𝑜𝑚𝑒𝑠 = 1
Recombine the two inequalities:
𝑇𝑜𝑡𝑎𝑙 𝑂𝑢𝑡𝑐𝑜𝑚𝑒𝑠 = (516 ) − (52 ) + 1 = 516 − 24
52 < 𝑥 < 516
1
Part B 𝑃𝑟𝑜𝑏𝑎𝑏𝑖𝑙𝑖𝑡𝑦 = 16
5 − 24
𝑆𝑢𝑐𝑐𝑒𝑠𝑠𝑓𝑢𝑙 𝑂𝑢𝑡𝑐𝑜𝑚𝑒𝑠 = 1

D. Floor Function

2.46: Floor Function


⌊𝑥⌋ = 𝑐 ⇒ 𝑐 ≤ 𝑥 < 𝑐 + 1

Example 2.47
A. ⌊3.4⌋
B. ⌊−2.1⌋
C. ⌊𝑥⌋ = 3

⌊3.4⌋ = 3
⌊−2.1⌋ = −3
⌊𝑥⌋ = 3 ⇒ 3 ≤ 𝑥 < 4

Example 2.48
A. ⌊log 3 𝑥⌋ = 3
1
B. ⌊log 3 𝑥⌋ = 3
C. ⌊log 𝑏 𝑥⌋ = 𝑘
D. log 5 ⌊𝑥⌋ = 3

P a g e 71 | 89
Get all the files at: https://bit.ly/azizhandouts
Aziz Manva (azizmanva@gmail.com)

E. log 5 𝑥 = ⌊3⌋
F. log ⌊𝑥⌋ 25 = 2

Part A 𝑘 ≤ log 𝑏 𝑥 < 𝑘 + 1


Use the property of the floor function: 𝑏 𝑘 ≤ 𝑥 < 𝑏 𝑘+1
3 ≤ log 3 𝑥 < 4 Part D
Exponentiate throughout with respect to the base 3: log 5 ⌊𝑥⌋ = 3
33 ≤ 𝑥 < 3 4 ⌊𝑥⌋ = 53 = 125
Simplify: 125 ≤ 𝑥 < 126
27 ≤ 𝑥 < 81 𝑥 ∈ [125,126)
𝑥 ∈ [27,81) Part E
Part B log 5 𝑥 = 3
1 𝑥 = 53 = 125
⌊log 3 𝑥⌋ = ⇒ 𝑥 ∈ 𝜙
3 Part F
Part C log ⌊𝑥⌋ 25 = 2
If 𝑘 ∉ ℤ: 25 = ⌊𝑥⌋2
𝑥∈𝜙 25 ≤ 𝑥 2 < 26
If 𝑘 ∈ ℤ:
5 ≤ 𝑥 < √26

Example 2.49
⌊log 2 𝑥⌋ = 5
A. Find the integer solutions, and the number of solutions to the above equation.
B. Find the range of 𝑥 that satisfies the above equation.

Part A
5 ≤ log 2 𝑥 < 6
Exponentiate throughout:
32 ≤ 𝑥 < 64
Convert the inequality into set notation:
𝑥 ∈ {32,33, … ,63}
63 − 32 + 1 = 32 𝑆𝑜𝑙𝑢𝑡𝑖𝑜𝑛𝑠
Part B
32 ≤ 𝑥 < 64 ⇒ 𝑥 ∈ [32,64)

Example 2.50
Find the number of:
A. Even solutions to ⌊log 2 𝑥⌋ = 5
log2 𝑥
B. Solutions which are a perfect power of 2 to ⌊ + 7⌋ = 8
12
log2 𝑥
C. Even solutions to ⌊ 12
+ 7⌋ = 8

log 2 𝑥
8≤ +7<9
12
Part B log 2 𝑥
1≤ <2
𝑥 ∈ {32,34, … ,62} 12
Divide by 2: 12 ≤ log 2 𝑥 < 24
𝑥 ∈ {16,17, … ,31} 212 ≤ 𝑥 < 224
31 − 16 + 1 = 16 𝑆𝑜𝑙𝑢𝑡𝑖𝑜𝑛𝑠 {212 , 213 , … , 223 }
Part C 23 − 12 + 1 = 12 𝑆𝑜𝑙𝑢𝑡𝑖𝑜𝑛𝑠

P a g e 72 | 89
Get all the files at: https://bit.ly/azizhandouts
Aziz Manva (azizmanva@gmail.com)

Part D 212 212 + 2 224 − 2


Smallest number that works is: { , ,…, }
2 2 2
212 Simplify
Largest number that works is: {211 , 211 + 1, … , 223 − 1}
224 − 2 The number of numbers is:
Hence, the solution set is: 223 − 1 − (211 ) + 1
{212 , 212 + 2, … , 224 − 2} 223 − 211
Divide by 2:

Example 2.51
⌊log 𝐶 𝑥⌋ = 𝐾, 𝐾∈ℤ
A. Find the integer solutions, and the number of solutions to the above equation.
B. Find the range of 𝑥 that satisfies the above equation.

Part A
𝐾 < log 𝐶 𝑥 < 𝐾 + 1
Exponentiate:
𝐶 𝐾 ≤ 𝑥 < 𝐶 𝐾+1
𝑥∈ {𝐶 𝐾, 𝐶 𝐾 + 1, … , 𝐶 𝐾+1 − 1}
Part B
𝐶 𝐾 ≤ 𝑥 < 𝐶 𝐾+1 ⇒ 𝑥 ∈ [𝐶 𝐾 , 𝐶 𝐾+1 )

Example 2.52
Find the integer solutions to:
⌊log 2 𝑥⌋ = ⌊log 3 𝑥⌋

⌊log 2 𝑥⌋ = ⌊log 3 𝑥⌋ = 𝐾, 𝐾∈ℤ


The smallest value that 𝑥 can take is 1. If
𝑥≥1⇒𝐾≥0⇒𝐾∈ℕ
Hence, we do not need to consider negative values for 𝐾.
Case I: 𝑲 = 𝟎:
⌊log 2 𝑥⌋ = 0 ⇒ 𝑥 ∈ {𝟏}
⌊log 3 𝑥⌋ = 0 ⇒ 𝑥 ∈= {30 , 30 + 1} = {𝟏, 2}
Case II: 𝑲 = 𝟏:
⌊log 2 𝑥⌋ = 1 ⇒ 𝑥 ∈ {21 , 21 + 1} = {2, 𝟑}
⌊log 3 𝑥⌋ = 1 ⇒ 𝑥 ∈ {31 , 31 + 1, … , 32 − 1} = {𝟑, 4, … ,8}
Case III: 𝑲 = 𝟐:
⌊log 2 𝑥⌋ = 2 ⇒
𝑥 ∈ {2 , 2 + 1, … , 23 − 1} = {4,5, … ,7}
2 2

⌊log 3 𝑥⌋ = 2 ⇒ 𝑥 ∈ {32 , 32 + 1, … , 33 − 1} = {9,10, … ,26}


𝐼𝑛𝑡𝑒𝑟𝑠𝑒𝑐𝑡𝑖𝑜𝑛 𝑖𝑠 𝜙
We did not get any solutions for the case 𝐾 = 2. Note that
3𝑥 3 𝑥
= ( ) > 1, 𝑥 > 1
2𝑥 2
Hence, we will not get any solutions for 𝐾 ≥ 2 because
3𝑥 𝑔𝑟𝑜𝑤𝑠 𝑓𝑎𝑠𝑡𝑒𝑟 𝑡ℎ𝑎𝑛 2𝑥
Hence, the integer solutions are:
{1,3}

P a g e 73 | 89
Get all the files at: https://bit.ly/azizhandouts
Aziz Manva (azizmanva@gmail.com)

Example 2.53
Find the integer solutions to:
⌊log 3 𝑥⌋ = ⌊log 4 𝑥⌋

⌊log 3 𝑥⌋ = ⌊log 4 𝑥⌋ = 𝐾
Case I: 𝐾 = 0:
⌊log 3 𝑥⌋ = 0 ⇒ 𝑥 ∈ {30 , 30 + 1} = {𝟏, 𝟐}
⌊log 4 𝑥⌋ = 0 ⇒ 𝑥 ∈ {𝟏, 𝟐, 3}
𝑥 = {1,2}
Case II: 𝐾 = 1:
⌊log 3 𝑥⌋ = 1 ⇒ 𝑥 ∈ {31 , 31 + 1, … , 32 − 1} = {𝟑, 4, … ,8}
⌊log 4 𝑥⌋ = 1 ⇒ 𝑥 ∈ {41 , 41 + 1, … , 42 − 1} = {4,5, … ,15}
𝑥 = {4,5, … ,8}
Case III: 𝐾 = 2:
⌊log 3 𝑥⌋ = 2 ⇒ 𝑥 ∈ {32 , 32 + 1, … , 33 − 1} = {9,10, … ,26}
⌊log 4 𝑥⌋ = 2 ⇒ 𝑥 ∈ {42 , 42 + 1, … , 43 − 1} = {16,5, … ,63}
𝑥 = {16,17, … ,26}
Case III: 𝐾 = 3:
⌊log 3 𝑥⌋ = 3 ⇒ 𝑥 ∈ {27,28, … ,80}
⌊log 4 𝑥⌋ = 2 ⇒ 𝑥 ∈ {64,65, … ,255}
𝑥 = {64,65, … ,80}
The final answer is the union of the answers from each case above:
{1,2} ∪ {4,5, … ,8} ∪ {16,17, … ,26} ∪ {64,65, … ,80}

Example 2.54
Find the range of 𝑥 that satisfies each equation below:
⌊log 2 𝑥⌋ = ⌊log 3 𝑥⌋ = 𝑐, 𝑐 ≥ 0

⌊log 2 𝑥⌋ = ⌊log 3 𝑥⌋ = 𝐾, 𝐾∈ℤ


⌊log 𝐶 𝑥⌋ = 𝐾, 𝐾 ∈ ℤ ⇒ 𝐶 𝐾 ≤ 𝑥 < 𝐶 𝐾+1 ⇒ 𝑥 ∈ [𝐶 𝐾 , 𝐶 𝐾+1 )
Case I: 𝑲 = 𝟎:
⌊log 2 𝑥⌋ = 0 ⇒ 𝑥 ∈ [1,2)
⌊log 3 𝑥⌋ = 0 ⇒ 𝑥 ∈ [30 , 31 ) = [1,3)
𝑥 ∈ [1,2)
Case II: 𝑲 = 𝟏:
⌊log 2 𝑥⌋ = 1 ⇒ 𝑥 ∈ [21 , 22 ) = [2,4)
⌊log 3 𝑥⌋ = 1 ⇒ 𝑥 ∈ [31 , 32 ) = [3,9)
𝑥 ∈ [3,4)
Hence, the integer solutions are:
[1,2) ∪ [3,4)

Example 2.55
Find the range of 𝑥 that satisfies each equation below:
⌊log 3 𝑥⌋ = ⌊log 4 𝑥⌋ = 𝑐, 𝑐 ≥ 0

⌊log 3 𝑥⌋ = ⌊log 4 𝑥⌋ = 𝐾
Case I: 𝐾 = 0:
⌊log 3 𝑥⌋ = 0 ⇒ 𝑥 ∈ [30 , 31 ) = [1,3)
⌊log 4 𝑥⌋ = 0 ⇒ 𝑥 ∈ [40 , 41 ) = [1,4)

P a g e 74 | 89
Get all the files at: https://bit.ly/azizhandouts
Aziz Manva (azizmanva@gmail.com)

𝑥 = [1,3)
Case II: 𝐾 = 1:
⌊log 3 𝑥⌋ = 1 ⇒ 𝑥 ∈ [31 , 32 ) = [3,9)
⌊log 4 𝑥⌋ = 1 ⇒ 𝑥 ∈ [41 , 42 ) = [4,16)
𝑥 = [4,9)
Case III: 𝐾 = 2:
⌊log 3 𝑥⌋ = 2 ⇒ 𝑥 ∈ [32 , 33 ) = [9,27)
⌊log 4 𝑥⌋ = 2 ⇒ 𝑥 ∈ 𝑥 ∈ [42 , 43 ) = [16,64)
𝑥 = [16,27)
Case III: 𝐾 = 3:
⌊log 3 𝑥⌋ = 3 ⇒ 𝑥 ∈ [33 , 34 ) = [27,81)
⌊log 4 𝑥⌋ = 2 ⇒ 𝑥 ∈ [43 , 44 ) = [64,256)
𝑥 = [64,81)
The final answer is the union of the answers from each case above:
[1,3) ∪ [4,9) ∪ [16,27) ∪ [64,81)

Example 2.565
⌊log 𝑝 1947⌋ = ⌊log 𝑞 1947⌋, 𝑝, 𝑞 ∈ ℕ, 1 < 𝑝, 𝑞 < 1950
Find the number of
A. unordered pairs (𝑝, 𝑞)
B. ordered pairs (𝑝, 𝑞).

Part A .
Since 𝑝 and 𝑞 are not required to be distinct: 3 < log12 1947 < 4
𝑝 = 𝑞 ∈ {2,3,4, … ,1950} ⇒ 1949 𝑆𝑜𝑙𝑢𝑡𝑖𝑜𝑛𝑠
The number of numbers from 7 to 12 is
If 𝑝 ≠ 𝑞, then we analyze further: 12 − 7 + 1 = 6
10 < log 2 1947 < 11 Number of unordered pairs should be:
6 < log 3 1947 < 7 6 6×5
( )= = 15
5 < log 4 1947 < 6 2 2
4 < log 5 1947 < 5
4 < log 6 1947 < 5

3 < log 7 1947 < 4 2 < log13 1947 < 3


3 < log 8 1947 < 4
.
. Part C

Example 2.57
Let 𝑥 be chosen at random from the interval (0,1). What is the probability that ⌊log10 4𝑥⌋ − ⌊log10 𝑥⌋ = 0? Here
⌊𝑥⌋ denotes the greatest integer that is less than or equal to 𝑥. (AMC 12B 2006/20)

⌊log10 4𝑥⌋ = ⌊log10 𝑥⌋ = 𝑘


⌊log10 4𝑥⌋ = 𝑘
⌊log10 𝑥⌋ = 𝑘 𝑘 ≤ log10 4𝑥 < 𝑘 + 1
𝑘 ≤ log10 𝑥 < 𝑘 + 1 10𝑘 ≤ 4𝑥 < 10𝑘+1
10𝑘 ≤ 𝑥 < 10𝑘+1

5
15th Aug 1947 and Jan 26, 1950 are important dates in Indian history.

P a g e 75 | 89
Get all the files at: https://bit.ly/azizhandouts
Aziz Manva (azizmanva@gmail.com)

10𝑘 10𝑘+1 10−1


≤𝑥< 10−2 ≤ 𝑥 <
4 4 4
1 1
≤𝑥<
10𝑘 10𝑘+1 100 40
< 10𝑘 ≤ 𝑥 < < 10𝑘+1 1 1 6 3
4 4 𝐿𝑒𝑛𝑔𝑡ℎ 𝑜𝑓 𝑉𝑎𝑙𝑖𝑑 𝐼𝑛𝑡𝑒𝑟𝑣𝑎𝑙 = − = =
10𝑘+1 40 100 400 200
10𝑘 ≤ 𝑥 <
4
But note that since 𝑥 lies in the interval (0,1): 3 3
+ +⋯
0<𝑥<1 ⏟
20 ⏟
200
𝑘=−1 𝑘=−2
0 < 𝑥 < 100 3
This is a geometric series with 𝑎 = 20 , 𝑟 = 1/10 with
Suppose 𝑘 = −1: sum:
100 3
−1
10 ≤ 𝑥 < 𝑎 20 3 10 1
4 𝑆= = = ∙ =
1−𝑟 1− 1 20 9 6
1 1 10
≤𝑥<
10 4
1 1 6 3 1
𝐿𝑒𝑛𝑔𝑡ℎ 𝑜𝑓 𝑉𝑎𝑙𝑖𝑑 𝐼𝑛𝑡𝑒𝑟𝑣𝑎𝑙 = − = = 𝐿𝑒𝑛𝑔𝑡ℎ 𝑜𝑓 𝑉𝑎𝑙𝑖𝑑 𝐼𝑛𝑡𝑒𝑟𝑣𝑎𝑙 6 1
4 10 40 20 𝑃𝑟𝑜𝑏𝑎𝑏𝑖𝑙𝑖𝑡𝑦 = = =
Suppose 𝑘 = −2: 𝑇𝑜𝑡𝑎𝑙 𝐼𝑛𝑡𝑒𝑟𝑣𝑎𝑙 1 6

2.58: Conversions
log 𝑒 𝑎 = 2.303 ∙ log10 𝑎

log10 𝑎 = 0.434 ∙ log 𝑒 𝑎

2.3 Applications-II
A. Inequalities

Example 2.59: Inequality Proof


𝛼 𝛼 𝛼
If 𝑛 is a natural number such that 𝑛 = 𝑝1 1 ∙ 𝑝2 2 ∙ … ∙ 𝑝𝑘 𝑘 , and 𝑝1 , 𝑝2 , … 𝑝𝑘 are distinct primes, then show that ln 𝑛 ≥
𝑘 ln 2. When will equality hold? (IIT JEE 1984)

Is 𝜶 a natural number
Suppose we consider Note that the given equality has 𝑛, while the required
2 = 21 ∙ 30 ⇒ ln 2 = ln 2 inequality has ln 𝑛. Take the natural log of both sides
This gives us: of the given equality:
𝛼 𝛼 𝛼
𝑝1 = 2, 𝑝2 = 3, 𝛼1 = 1, 𝛼2 = 0, 𝑘 = 2 ln 𝑛 = ln(𝑝1 1 ∙ 𝑝2 2 ∙ … ∙ 𝑝𝑘 𝑘 )
ln 2 < 𝑘 ln 2 = 2 ln 2 Use the product rule:
Hence, even though the question does not apply the 𝛼 𝛼 𝛼
= ln(𝑝1 1 ) + ln(𝑝2 2 ) + ⋯ + ln(𝑝𝑘 𝑘 )
𝛼 𝛼 𝛼
restriction, we consider 𝑝1 1 ∙ 𝑝2 2 ∙ … ∙ 𝑝𝑘 𝑘 to be the Use the power rule
factorization of the natural number 𝑛, which then = 𝛼1 ln 𝑝1 + 𝛼2 ln 𝑝2 + ⋯ + 𝛼𝑘 ln 𝑝𝑘
enforces 𝑎𝑛 ≥ 1. Since 𝑝1 , 𝑝2 , … , 𝑝𝑘 are primes, 𝑝1 , 𝑝2 , … , 𝑝𝑘 ≥ 2,
Strategy ln 𝑛 ≥ 𝛼1 ln 2 + 𝛼2 ln 2 + ⋯ + 𝛼𝑘 ln 2
We have been given an equality, and we need to prove ln 𝑛 ≥ ln 2 (𝛼1 + 𝛼2 + ⋯ + 𝛼𝑘 )
an inequality. We will make the conversion, but first Since 𝑛 is a natural number, 𝛼𝑛 > 1:
get the equality into a form similar to what we want. ln 𝑛 ≥ ln 2 (1 + 1 + ⋯ + 1)

P a g e 76 | 89
Get all the files at: https://bit.ly/azizhandouts
Aziz Manva (azizmanva@gmail.com)

B. Diophantine Equations

Example 2.60: Diophantine Equations


log 2 𝑥 and log 4 𝑦 are natural numbers. The pairs (𝑥, 𝑦) that are a solution to log 2 𝑥 + log 4 𝑦 = 8 can be written
(𝑥1 , 𝑦1 )(𝑥2 , 𝑦2 ) … (𝑥𝑛 , 𝑦𝑛 )
Find 𝑎 given that the product:
𝑥1 𝑥2 … 𝑥𝑛 𝑦1 𝑦2 … 𝑦𝑛 = 2𝑎

Type equation here.

Example 2.61: Zero and One as Solutions


Find the number of distinct ordered pair solutions (𝑥, 𝑦) for:
1 1
log 𝑥 𝑦 = 0, log 𝑥 𝑦 = 1, log 𝑦 𝑥 = −1, 𝑥 ∈ {0,1,2, } , 𝑦 ∈ {0,1, , 3}
3 2

1
log 𝑥 𝑦 = 0 ⇒ (𝑥, 𝑦) = (2,1), ( , 1)
3
log 𝑥 𝑦 = 1 ⇒ (𝑥, 𝑦) = 𝑁𝑜 𝑆𝑜𝑙𝑢𝑡𝑖𝑜𝑛𝑠
1 1
log 𝑥 𝑦 = −1 ⇒ (𝑥, 𝑦) = ( , 3) ( , 2)
3 2

Example 2.62: Integer Solutions


Find integer values of 𝑥 and 𝑦 for which
𝑥 − 𝑦 log 3 2 = 10 log 9 6

Method I
𝑥 = log 3 65 + log 3 2𝑦 = log 3 35 ∙ 25 ∙ 2𝑦
25 ∙ 2𝑦 = 20 ⇒ 5 + 𝑦 = 0 ⇒ 𝑦 = −5
𝑥=5
Method II
𝑅𝐻𝑆 = 10 log 9 6 = log 3 35 ∙ 25 = log 3 35 + log 3 25 = 5 + 5 log 3 2
𝑥 − 𝑦 log 3 2 = 5 + 5 log 3 2
𝑥=5
−𝑦 = 5 ⇒ 𝑦 = −5
C. Classification of Numbers

Example 2.63
A. Determine, with proof, whether log 2 7 is rational or irrational. (IIT JEE 1990)
B. Is it possible to generalize your result?

Part A
Assume, if possible, to the contrary that:
log 2 7 ∈ ℚ
Then for some integers 𝑝 and 𝑞:
𝑝
log 2 7 =
𝑞
Exponentiate both sides:

P a g e 77 | 89
Get all the files at: https://bit.ly/azizhandouts
Aziz Manva (azizmanva@gmail.com)
𝑝
7 = 2𝑞
Raise both sides to the power 𝑞:
2𝑝 = 7𝑞
However, no power of 2 is ever equal to a power of 7.
Hence:
log 2 7 ∉ ℚ
log 2 7 𝑖𝑠 𝑎𝑛 𝑖𝑟𝑟𝑎𝑡𝑖𝑜𝑛𝑎𝑙 𝑛𝑢𝑚𝑏𝑒𝑟
Part B
log 𝑎 𝑏 for non-zero values of the expression will be rational only if the prime factorization of 𝑏 and the prime
factorization of 𝑎 both have exactly the same prime numbers.
log18 12 = log 2×32 22 × 3

log 𝑎 𝑏
𝑏 = (𝑝1 × 𝑝2 )𝑚
𝑎 = (𝑝1 × 𝑝2 )𝑛

log 2×32 22 × 34 = log 2×32 (2 × 32 )2 = 2 log 2×32 (2 × 32 ) = 2

D. Maximum and Minimum

2.64: AM-GM Inequality


𝑎+𝑏
≥ √𝑎𝑏

⏟2
𝐺𝑒𝑜𝑚𝑒𝑡𝑟𝑖𝑐
𝐴𝑟𝑖𝑡ℎ𝑚𝑒𝑡𝑖𝑐 𝑀𝑒𝑎𝑛
𝑀𝑒𝑎𝑛

Consider the product


𝑎𝑏, 𝑎 > 0, 𝑏 > 0
For a given value of 𝒂𝒃:
𝑎 + 𝑏 𝑤𝑖𝑙𝑙 𝑏𝑒 𝑚𝑖𝑛𝑖𝑚𝑢𝑚 𝑤ℎ𝑒𝑛 𝑎 = 𝑏
Suppose 𝑎𝑏 = 4, then:
𝑎 = 2, 𝑏 = 2 ⇒ 𝑎 + 𝑏 = 2 + 2 = 4
𝑎 = 1, 𝑏 = 4 ⇒ 𝑎 + 𝑏 = 1 + 4 = 5
For a given value of 𝒂 + 𝒃:
𝑎𝑏 𝑤𝑖𝑙𝑙 𝑏𝑒 𝑚𝑎𝑥𝑖𝑚𝑢𝑚 𝑤ℎ𝑒𝑛 𝑎 = 𝑏
Suppose 𝑎 + 𝑏 = 4, then:
𝑎 = 2, 𝑏 = 2 ⇒ 𝑎𝑏 = 4
𝑎 = 1, 𝑏 = 3 ⇒ 𝑎𝑏 = 3

Example 2.65
If it is known that log 2 𝑎 + log 2 𝑏 ≥ 6, then the least value that can be taken on by 𝑎 + 𝑏 is: (AHSME 1969/25)

Use the product rule:


log 2 𝑎𝑏 ≥ 6
Convert to exponential form:
𝑎𝑏 ≥ 26
If you substitute 𝑎 = 𝑏 = 23 , then you get the minimum value as

P a g e 78 | 89
Get all the files at: https://bit.ly/azizhandouts
Aziz Manva (azizmanva@gmail.com)

𝑎𝑏 ≥ 64
Take the square root both sides:
√𝑎𝑏 ≥ 8
𝑎+𝑏
If we combine the AM-GM inequality ( 2 ≥ √𝑎𝑏), with the above inequality, we get:
𝑎+𝑏
≥ √𝑎𝑏 ≥ 8
2
Multiply throughout by 2:
𝑎 + 𝑏 ≥ 2√𝑎𝑏 ≥ 16
And equality is achieved when
𝑎=𝑏=8

E. Number of Digits

Example 2.66 (Calculator)


Number of digits in 156324
Find first five digits of the number

Part A
𝑦 = 156324
Take logs to the base-10 both sides:
log 𝑦 = 6324 log 15 ≈ 7437.601122
𝑦 = 107437.601122 ⇒ 7438 𝐷𝑖𝑔𝑖𝑡𝑠
Part B
𝑦 = 107437.601122 = 100.601122 × 107437

100.601122 = 3.9913 … (𝑛𝑜𝑡 𝑟𝑜𝑢𝑛𝑑𝑒𝑑)


𝐹𝑖𝑟𝑠𝑡 𝑓𝑖𝑣𝑒 𝑑𝑖𝑔𝑖𝑡𝑠 = 39913

Example 2.67 (Calculator)


Number of digits in 125764
Find first five digits of the number

Part A
𝑦 = 125764
Take logs to the base-10 both sides:
log 𝑦 = 5764 log 12 ≈ 6220.40070
𝑦 = 106220.40070 ⇒ 6221 𝐷𝑖𝑔𝑖𝑡𝑠
Part B
𝑦 = 106220.40070 = 100.40070 × 106220

100.40070 = 2.5159 … (𝑛𝑜𝑡 𝑟𝑜𝑢𝑛𝑑𝑒𝑑)


𝐹𝑖𝑟𝑠𝑡 𝑓𝑖𝑣𝑒 𝑑𝑖𝑔𝑖𝑡𝑠 = 25159
F. Absolute Value

G. Trigonometric Equations

Example 2.68
P a g e 79 | 89
Get all the files at: https://bit.ly/azizhandouts
Aziz Manva (azizmanva@gmail.com)

H. Exponential and Logarithmic Modelling

2.69: Models

Exponential Growth:
𝑦 = 𝑎𝑏 𝑥 , 𝑏 =1+𝑟
Exponential Decay:
𝑦 = 𝑎𝑏 𝑥 , 𝑏 =1−𝑟

Example 2.70

50 = 100𝑒 −5730𝑘
1
= 𝑒 −5730𝑘
2
1
ln ( ) = −5730𝑘
2
1 1 1 −1
ln (2) −ln (2) ln (2) ln 2
𝑘= = = =
−5730 5730 5730 5730

Example 2.71
𝑉 = 𝑎𝑏 𝑡 , 𝑏 = 1 + 𝑟

𝑏 = 1 + 𝑟 = 1 + 50% = 1 + 0.5 = 1.5


Substitute 𝑏 = 1.5 in 𝑉 = 𝑎𝑏 𝑡
𝑉 = 𝑎(1.5)𝑡
At the start point, the price of the stock is $0.59
Substitute (𝑡, 𝑉) = (0,0.59):
0.59 = 𝑎(1.5)0 ⇒ 0.59 = 𝑎
Hence, the equation is:
𝑉 = 0.59(1.5)𝑡
If the value is 6 dollars, then 𝑉 = 6:
6 = 0.59(1.5)𝑡
6
= (1.5)𝑡
0.59
Take log to the base 10 on both sides:

6
log = log(1.5)𝑡
0.59

Example 2.72
Current value of lamp is 1000 dollars. Lamp appreciates 15% each year.
𝑉 = 𝑎𝑏 𝑡 , 𝑏 = 1 + 𝑟

Current price is
𝑉 = 1000 𝐷𝑜𝑙𝑙𝑎𝑟𝑠
Let the current time be

P a g e 80 | 89
Get all the files at: https://bit.ly/azizhandouts
Aziz Manva (azizmanva@gmail.com)

𝑡=0
Substitute 𝑉 = 1000, 𝑡 = 0:
1000 = 𝑎𝑏 0 ⇒ 1000 = 𝑎

Example 2.73
𝑁(𝑡) = 𝑁0 𝑒 −0.1155𝑡

𝑁(𝑡) = 𝑁0 (𝑒 −0.1155 )𝑡
𝑏 = 𝑒 −0.1155
1 − 𝑟 = 𝑒 −0.1155
1 − 𝑒 −0.1155 = 𝑟

Example 2.74
Richter Scale
2 𝐸
𝑀 = log ( 11.8 )
3 10

2 𝐸
9.2 = log ( 11.8 )
3 10
3 𝐸
9.2 × = log ( 11.8 )
2 10
𝐸
1013.8 = 11.8
10
1025.6 = 𝐸
3.98 × 1025 = 𝐸

2.75: Newton’s Law of Cooling


𝑇𝑡 − 𝑇𝑆 = (𝑇0 − 𝑇𝑠 )𝑏 𝑡
Where
𝑇𝑡 = 𝑇𝑒𝑚𝑝𝑒𝑟𝑎𝑡𝑢𝑟𝑒 𝑎𝑡 𝑇𝑖𝑚𝑒 𝑡
𝑇𝑠 = 𝑆𝑢𝑟𝑟𝑜𝑢𝑛𝑑𝑖𝑛𝑔 𝑇𝑒𝑚𝑝𝑒𝑟𝑎𝑡𝑢𝑟𝑒
𝑇0 = 𝐼𝑛𝑖𝑡𝑖𝑎𝑙 𝑇𝑒𝑚𝑝𝑒𝑟𝑎𝑡𝑢𝑟𝑒
𝑡 = 𝐸𝑙𝑎𝑝𝑠𝑒𝑑 𝑇𝑖𝑚𝑒
𝑏 𝑖𝑠 𝑎 𝑐𝑜𝑛𝑠𝑡𝑎𝑛𝑡

Example 2.76
A potato is taken from the oven, its temperature having reached 350°. After sitting on a plate in a 70° room for
twelve minutes, its temperature has dropped to 250°. In how many more minutes will the potato's temperature
reach 120°? Assume Newton's Law of Cooling, which says that the difference between an object's temperature and
the ambient temperature is an exponential function of time. (Phillips Exeter Math 4, 2022/5)

1 1 1
180 12 18 12 9 12
𝑏=( ) =( ) =( )
280 28 14
𝑇⏟𝑡 − 𝑇⏟𝑆 =[ 𝑇⏟0 − 𝑇𝑠 ] 𝑏 𝑡
𝑀𝑒𝑎𝑠𝑢𝑟𝑒𝑑 𝑅𝑜𝑜𝑚 𝐼𝑛𝑖𝑡𝑖𝑎𝑙 1
𝑇𝑒𝑚𝑝𝑒𝑟𝑎𝑡𝑢𝑟𝑒 𝑇𝑒𝑚𝑝𝑒𝑟𝑎𝑡𝑢𝑟𝑒 𝑇𝑒𝑚𝑝𝑒𝑟𝑎𝑡𝑢𝑟𝑒 9 12
12
Substitute 𝑇0 = 350, 𝑇𝑠 = 70, 𝑇𝑡 = 120, 𝑏 = (14) :
250 − 70 = (350 − 70)𝑏
180 = 280𝑏12

P a g e 81 | 89
Get all the files at: https://bit.ly/azizhandouts
Aziz Manva (azizmanva@gmail.com)

1 𝑡 5
9 12 𝑡 ln 28
120 − 70 = (350 − 70) [( ) ] =
14 12 ln 9
14
𝑡 5
9 12 ln
50 = 280 ( ) 𝑡 = 12 28 ≈ 46.789
14 9
𝑡 ln
5 9 12 14
=( ) 𝑀𝑜𝑟𝑒 𝑀𝑖𝑛𝑢𝑡𝑒𝑠 = 46.789 − 12 = 34.789
28 14
𝑡
5 9 12
ln = ln ( )
28 14

I. 𝒚 = 𝒍𝒐𝒈 𝒙 as a Function

2.4 Hyperbolic Functions


A. Hyperbolic Sine
The hyperbolic sine is called such because of its similarity to the trigonometric sine function.
It is defined in terms of exponentials, so its similarity is not immediately apparent, but we will see it as go along.

2.77: sinh x
𝑒 𝑥 − 𝑒 −𝑥
𝐻𝑦𝑝𝑒𝑟𝑏𝑜𝑙𝑖𝑐 𝑠𝑖𝑛𝑒 = sinh 𝑥 =
2

There are no restrictions on the inputs for the


function.
Hence,
𝐷𝑜𝑚𝑎𝑖𝑛 𝑜𝑓 sinh 𝑥 = ℝ
Also,
𝑅𝑎𝑛𝑔𝑒 𝑜𝑓 sinh 𝑥 = ℝ

sinh 𝑥 is a continuous function.

Example 2.78
Evaluate:
A. sinh 0
B. sinh 1
C. sinh(ln 3)

Example 2.79
Show that the 𝐷𝑜𝑚𝑎𝑖𝑛 𝑜𝑓 sinh 𝑥 = ℝ

There are no restrictions on the inputs to the function because of denominators, roots, etc.
Hence:
𝐷𝑜𝑚𝑎𝑖𝑛 𝑜𝑓 sinh 𝑥 = ℝ

Example 2.80
Show that the range of sinh 𝑥 is (−∞, ∞)

P a g e 82 | 89
Get all the files at: https://bit.ly/azizhandouts
Aziz Manva (azizmanva@gmail.com)

Calculate the limit as it goes to positive infinity:


𝑒 𝑥 − 𝑒 −𝑥 𝑒𝑥 1
lim = lim − lim − 𝑥 = ∞ − 0 = ∞
𝑥→∞ 2 𝑥→∞ 2 𝑥→∞ 2𝑒

Similarly, calculate the limit as it goes to negative infinity:


𝑒 𝑥 − 𝑒 −𝑥 𝑒𝑥 1
lim = lim − lim − 𝑥 = 0 − ∞ = −∞
𝑥→−∞ 2 𝑥→−∞ 2 𝑥→−∞ 2𝑒

Also, note that it is a continuous function with


𝐷𝑜𝑚𝑎𝑖𝑛 = ℝ

And hence, it will achieve all values between the above two limits. Hence,
𝑅𝑎𝑛𝑔𝑒 𝑜𝑓 sinh 𝑥 = ℝ

2.81: sinh x is an odd function


➢ sinh(−𝑥) = − sinh 𝑥
➢ sinh 𝑥 𝑖𝑠 𝑠𝑦𝑚𝑚𝑒𝑡𝑟𝑖𝑐 𝑎𝑏𝑜𝑢𝑡 𝑡ℎ𝑒 𝑜𝑟𝑖𝑔𝑖𝑛

Recall that a function is odd if and only if:


𝑓(−𝑥) = −𝑓(𝑥)
And also that
𝑂𝑑𝑑 𝑓𝑢𝑛𝑐𝑡𝑖𝑜𝑛𝑠 𝑎𝑟𝑒 𝑠𝑦𝑚𝑚𝑒𝑡𝑟𝑖𝑐 𝑎𝑏𝑜𝑢𝑡 𝑡ℎ𝑒 𝑜𝑟𝑖𝑔𝑖𝑛

We can now prove that sinh 𝑥 is odd:

𝑒 −𝑥 − 𝑒 −(−𝑥) 𝑒 −𝑥 − 𝑒 𝑥 𝑒 𝑥 − 𝑒 −𝑥
sinh(−𝑥) = = =− = − sinh 𝑥
2 2 2

And the symmetry directly follows from the fact that it is odd.
B. Hyperbolic Cosine
Just like define the hyperbolic sine, we define the hyperbolic cosine in terms of exponentials.

2.82: cosh x
𝑒 𝑥 + 𝑒 −𝑥
𝐻𝑦𝑝𝑒𝑟𝑏𝑜𝑙𝑖𝑐 𝑐𝑜𝑠𝑖𝑛𝑒 = cosh 𝑥 =
2

Note the similarity between the definition of the hyperbolic sine and the hyperbolic cosine: the only difference
between sinh x and cosh x is that the minus sign is replaced with a plus sign.

Like the hyperbolic sine, the hyperbolic cosine also has:


𝐷𝑜𝑚𝑎𝑖𝑛 𝑜𝑓 cosh 𝑥 = ℝ

However, the hyperbolic cosine is the sum of the two functions, making its minimum 1. Hence, it has
𝑅𝑎𝑛𝑔𝑒 𝑜𝑓 cosh 𝑥 = [1, ∞)

2.83: cosh x is an even function


➢ cosh(−𝑥) = cosh 𝑥
➢ cosh 𝑥 𝑖𝑠 𝑠𝑦𝑚𝑚𝑒𝑡𝑟𝑖𝑐 𝑎𝑏𝑜𝑢𝑡 𝑡ℎ𝑒 𝑦 − 𝑎𝑥𝑖𝑠

P a g e 83 | 89
Get all the files at: https://bit.ly/azizhandouts
Aziz Manva (azizmanva@gmail.com)

𝑒 −𝑥 + 𝑒 −(−𝑥) 𝑒 −𝑥 + 𝑒 𝑥 𝑒 𝑥 + 𝑒 −𝑥
cosh(−𝑥) = = = = cosh 𝑥
2 2 2

Example 2.84
Show that the 𝐷𝑜𝑚𝑎𝑖𝑛 𝑜𝑓 cosh 𝑥 = ℝ

There are no restrictions on the inputs to the function because of denominators, roots, etc.
Hence:
𝐷𝑜𝑚𝑎𝑖𝑛 𝑜𝑓 cosh 𝑥 = ℝ

Example 2.856
Show that the range of cosh 𝑥 is [1, ∞)

𝑒 𝑥 + 𝑒 −𝑥 𝑒𝑥 1
𝑦 = cosh 𝑥 = = +
2 ⏟
2 ⏟
2𝑒 𝑥
𝑃𝑜𝑠𝑖𝑡𝑖𝑣𝑒 𝑃𝑜𝑠𝑖𝑡𝑖𝑣𝑒

Since both the terms of the above are positive, we know that
𝑦>0

However, we do not know the minimum value.


Recall that we can find the range of 𝑦 by solving for 𝑥, and then finding the domain of 𝑦.
To find the minimum value, find the inverse function of
𝑦 = cosh 𝑥
And then find the domain of 𝑦.

𝑒 𝑥 +𝑒 −𝑥
Multiply 𝑦 = 2
by 2 both sides:
1
2𝑦 = 𝑒 𝑥 +
𝑒𝑥
Multiply by 𝑒 𝑥 to eliminate fractions:
2𝑦𝑒 𝑥 = 𝑒 2𝑥 + 1
Collate all terms on one side:
𝑒 2𝑥 − 2𝑦𝑒 𝑥 + 1 = 0
Substitute 𝑧 = 𝑒 𝑥 ⇒ 𝑧 2 = 𝑒 2𝑥 :
𝑧 2 − 2𝑦𝑧 + 1 = 0

This is a quadratic in 𝑧, with 𝑎 = 1, 𝑏 = −2𝑦, 𝑐 = 1:


−𝑏 ± √𝑏 2 − 4𝑎𝑐 2𝑦 ± √4𝑦 2 − 4(1)(1) 2𝑦 ± 2√𝑦 2 − 1
𝑧 = 𝑒𝑥 = = = = 𝑦 ± 2√𝑦 2 − 1
2𝑎 2 2

Since y is real, we must have:


𝑦 2 − 1 ≥ 0 ⇒ 𝑦 2 ≥ 1 ⇒ 𝑦 ≥ 1 𝑂𝑅 𝑦 ≤ −1

But we already know that 𝑦 > 0. Hence:


𝑦≥1

6
Copy to Disguised Quadratics, Range of Exponential Functions

P a g e 84 | 89
Get all the files at: https://bit.ly/azizhandouts
Aziz Manva (azizmanva@gmail.com)

Example 2.86
cosh(ln 𝑎)

1 1
𝑒 ln 𝑎 + 𝑒 − ln 𝑎 𝑒 ln 𝑎 + 𝑒 ln𝑎 𝑎 + 𝑎 𝑎2 + 1
cosh(ln 𝑎) = = = =
2 2 2 2𝑎

C. Decomposing 𝒆𝒙 to get the hyperbolic functions (Optional)


We now look at one way to think about how the hyperbolic functions are obtained. We already know that
➢ sinh x is odd
➢ cosh x is even

We first see that these two functions add up to 𝑒 𝑥 . That is:


sinh 𝑥 + cosh ℎ = 𝑒 𝑥

But, if we had 𝑒 𝑥 , and we wanted to get sinh 𝑥 and cosh ℎ, how would we do that?
That is a based on a property that we prove (and then use).

Example 2.87
Show that sinh 𝑥 + cosh ℎ = 𝑒 𝑥

𝑒 𝑥 − 𝑒 −𝑥 𝑒 𝑥 + 𝑒 −𝑥 2𝑒 𝑥
sinh 𝑥 + cosh ℎ = + = = 𝑒𝑥
2 2 2

Example 2.88
Any function 𝑓(𝑥) can be decomposed into an even function and an odd function by using the following definitions:
𝑓(𝑥) + 𝑓(−𝑥)
𝐸𝑣𝑒𝑛 𝐹𝑢𝑛𝑐𝑡𝑖𝑜𝑛 = 𝑓𝑒 =
2
𝑓(𝑥) − 𝑓(−𝑥)
𝑂𝑑𝑑 𝐹𝑢𝑛𝑐𝑡𝑖𝑜𝑛 = 𝑓𝑜 =
2
Show that
A. 𝑓(𝑥) = 𝑓𝑒 + 𝑓𝑜
B. 𝑓𝑒 is even
C. 𝑓𝑜 is odd

Example 2.89
Use the property stated in the previous example to decompose 𝑓(𝑥) = 𝑒 𝑥 into an even function and odd function.
Name the functions so obtained.

𝑓(𝑥) + 𝑓(−𝑥) 𝑒 𝑥 + 𝑒 −𝑥
𝑓𝑒 = = = cosh 𝑥
2 2
𝑓(𝑥) − 𝑓(−𝑥) 𝑒 𝑥 − 𝑒 −𝑥
𝑓𝑜 = = = sinh 𝑥
2 2
D. Hyperbolic tangent
We can now start exploiting the similarity between the trigonometric functions and the hyperbolic functions.
tanh x is defined just as tan x would be, making it easier to remember.

P a g e 85 | 89
Get all the files at: https://bit.ly/azizhandouts
Aziz Manva (azizmanva@gmail.com)

2.90: tanh x
sinh 𝑥
tanh 𝑥 =
cosh 𝑥

Hyperbolic tan has


𝐷𝑜𝑚𝑎𝑖𝑛 𝑜𝑓 tanh 𝑥 = ℝ

And it has
𝑅𝑎𝑛𝑔𝑒 𝑜𝑓 tanh 𝑥 = (1,1)

Example 2.91
Show that the 𝐷𝑜𝑚𝑎𝑖𝑛 𝑜𝑓 tanh 𝑥 = ℝ

sinh 𝑥
tanh 𝑥 =
cosh 𝑥

Both functions have


𝐷𝑜𝑚𝑎𝑖𝑛 = ℝ

The numerator is not an issue.


𝐷𝑒𝑛𝑜𝑚𝑖𝑛𝑎𝑡𝑜𝑟 = cosh 𝑥 ℎ𝑎𝑠 𝑟𝑎𝑛𝑔𝑒 [1, ∞)
And hence is never zero.

Hence,
𝐷𝑜𝑚𝑎𝑖𝑛 𝑜𝑓 tanh 𝑥 = ℝ

Example 2.92
Show that range of tanh x is (−1,1)

𝑒 𝑥 − 𝑒 −𝑥
sinh 𝑥 𝑒 𝑥 − 𝑒 −𝑥
tanh 𝑥 = = 𝑥 2 −𝑥 = 𝑥
cosh 𝑥 𝑒 + 𝑒 𝑒 + 𝑒 −𝑥
2
Property I: tanh x passes through the origin
𝑒 0 − 𝑒 −0 0
tanh 0 = 0 = =0
𝑒 + 𝑒 −0 2

Property II: tanh x is always less than 1:


𝑒 𝑥 − 𝑒 −𝑥 𝑒𝑥
tanh 𝑥 = < <1
𝑒 𝑥 + 𝑒 −𝑥 𝑒 𝑥 + 𝑒 −𝑥

Property III: As x increases without bound, the limit of tanh x is 1.


1
𝑒 𝑥 − 𝑒 −𝑥 𝑒 𝑥 − 𝑒𝑥
lim = lim =1
𝑥→∞ 𝑒 𝑥 + 𝑒 −𝑥 𝑥→∞ 𝑥 1
𝑒 + 𝑒𝑥

If we combine the above three properties, then by the Intermediate Value Theorem
𝑅𝑎𝑛𝑔𝑒 𝑜𝑓 tanh 𝑥 𝑜𝑣𝑒𝑟 𝑥 ∈ [0, ∞) 𝑖𝑠 𝑦 ∈ [0,1)

P a g e 86 | 89
Get all the files at: https://bit.ly/azizhandouts
Aziz Manva (azizmanva@gmail.com)

Also, tanh x is an odd function, and hence:


𝑅𝑎𝑛𝑔𝑒 𝑜𝑓 tanh 𝑥 𝑜𝑣𝑒𝑟 𝑥 ∈ (−∞, 0] 𝑖𝑠 𝑦 ∈ [0, −1)

If we had not used the fact that tanh x is odd, we would have had to repeat the entire process for 𝑥 ∈ (−∞, 0]

Consider 𝑥 < 0, and substitute 𝑦 = −𝑥:


𝑒 𝑥 − 𝑒 −𝑥 𝑒 −𝑦 − 𝑒 𝑦 𝑒 𝑦 − 𝑒 −𝑦
lim 𝑥 = lim = lim − = −1
𝑥→−∞ 𝑒 + 𝑒 −𝑥 𝑦→∞ 𝑒 −𝑦 + 𝑒 𝑦 𝑦→∞ 𝑒 −𝑦 + 𝑒 𝑦

E. Reciprocal Hyperbolic Functions


Like the trigonometric functions, the hyperbolic functions also have reciprocal counterparts. We define each of
these below.

2.93: Reciprocal Functions: csch x


1
csch 𝑥 =
sinh 𝑥

2.94: Reciprocal Functions: sech h


1
sech 𝑥 =
cosh 𝑥

2.95: Reciprocal Functions: coth x


cosh 𝑥
coth 𝑥 =
sinh 𝑥

F. Identities

2.96: “Pythagorean Identity”


The Pythagorean identity holds in a modified form, so be careful.
cosh2 𝑥 − sinh2 𝑥 = 1

2 2
2 2
𝑒 𝑥 + 𝑒 −𝑥 𝑒 𝑥 − 𝑒 −𝑥
cosh 𝑥 − sinh 𝑥 = ( ) −( )
2 2

This is possible to square, but would be messy, increasing the chances of making a mistake.
Instead exploit the symmetry by using a change of variables.

Use a change of variables. Let 𝑎 = 𝑒 𝑥 , 𝑏 = 𝑒 −𝑥 . Then:


𝑎+𝑏 2 𝑎 − 𝑏 2 𝑎2 + 2𝑎𝑏 + 𝑏 2 𝑎2 − 2𝑎𝑏 + 𝑏 2 4𝑎𝑏
𝐿𝐻𝑆 = ( ) −( ) = − = = 𝑎𝑏 = 𝑒 𝑥 𝑒 −𝑥 = 1
2 2 4 4 4

2.97: Alternate Version “Pythagorean Identity”


1 − tanh2 𝑥 = sech2 𝑥

We have just proved:


cosh2 𝑥 − sinh2 𝑥 = 1
Dividing the above by cosh2 𝑥 gives us:

P a g e 87 | 89
Get all the files at: https://bit.ly/azizhandouts
Aziz Manva (azizmanva@gmail.com)

cosh2 𝑥 sinh2 𝑥 1
− =
cosh 𝑥 cosh 𝑥 cosh2 𝑥
2 2
1 − tanh2 𝑥 = sech2 𝑥

G. Double Identity

2.98: Double Angle Identity


cosh 2𝑥 − 1
sinh2 𝑥 =
2

𝑒 2𝑥 + 𝑒 −2𝑥 𝑎2 + 𝑏 2 𝑎2 + 𝑏 2 − 2
( 2 ) − 1 ( 2 ) − 1
𝑅𝐻𝑆 = = = 2
2 2 2

Substitute 𝑎𝑏 = 𝑒 𝑥 𝑒 −𝑥 = 1
2
𝑎2 − 2𝑎𝑏 + 𝑏 2 𝑎−𝑏 2 𝑒 𝑥 − 𝑒 −𝑥
= =( ) =( ) = 𝐿𝐻𝑆
4 2 2

2.99: Double Angle Identity


cosh 2𝑥 + 1
cosh2 𝑥 =
2

𝑒 2𝑥 + 𝑒 −2𝑥 𝑎2 + 𝑏 2 𝑎2 + 𝑏 2 + 2
( 2 ) + 1 ( 2 )+1
𝑅𝐻𝑆 = = = 2
2 2 2

Substitute 𝑎𝑏 = 𝑒 𝑥 𝑒 −𝑥 = 1
2
𝑎2 + 2𝑎𝑏 + 𝑏 2 𝑎+𝑏 2 𝑒 𝑥 + 𝑒 −𝑥
= =( ) =( ) = 𝐿𝐻𝑆
4 2 2
H. Sum to Product Rules

2.100: Sum to Product for sinh x


sinh(𝑥 + 𝑦) = sinh 𝑥 cosh 𝑦 + cosh 𝑥 sinh 𝑦

2.101: Sum to Product for cosh x


cosh(𝑥 + 𝑦) = cosh 𝑥 cosh 𝑦 + sinh 𝑥 sinh 𝑦

𝑒 𝑥 + 𝑒 −𝑥 𝑒 𝑦 + 𝑒 −𝑦 𝑒 𝑥 − 𝑒 −𝑥 𝑒 𝑦 − 𝑒 −𝑦
𝑅𝐻𝑆 = ( )( )+( )( )
2 2 2 2

Use a change of variable. Let 𝑎 = 𝑒 𝑥 , 𝑏 = 𝑒 −𝑥 , 𝑐 = 𝑒 𝑦 , 𝑑 = 𝑒 −𝑦


𝑎+𝑏 𝑐+𝑑 𝑎−𝑏 𝑐−𝑑
=( )( )+( )( )
2 2 2 2
𝑎𝑐 + 𝑎𝑑 + 𝑏𝑐 + 𝑏𝑑 𝑎𝑐 − 𝑎𝑑 − 𝑏𝑐 + 𝑏𝑑
= +
4 4
2𝑎𝑐 + 2𝑏𝑑 𝑎𝑐 + 𝑏𝑑 𝑎𝑐 + 𝑏𝑑
= = =
4 2 2

P a g e 88 | 89
Get all the files at: https://bit.ly/azizhandouts
Aziz Manva (azizmanva@gmail.com)

𝑒 𝑥+𝑦 + 𝑒 −(𝑥+𝑦)
= = 𝐿𝐻𝑆
2

102 Examples

P a g e 89 | 89

You might also like